An MH370 Flight Path Ending Further North on 7th Arc

Runway on Cocos Island.

Introduction

Now that the recent search effort conducted by Ocean Infinity has ended without finding MH370’s debris field on the seabed, we continue to re-evaluate the evidence and consider other possibilities.

Many researchers that have reconstructed flight paths assume that the aircraft was on autopilot after 19:41. This leads to flight paths that cross the 7th arc at 26S or further south. Now that the 7th arc has been searched as far north as 25S and at a width of at least +/-22 NM, we have to consider the following possibilities:

  1. There are automated flight paths that end north of the 25S latitude that have not been previously considered.
  2. The aircraft was actively piloted after 19:41.
  3. After fuel exhaustion, the aircraft glided without pilot inputs and impacted further from the 7th arc than was searched.
  4. After fuel exhaustion, there was an actively controlled glide that ended outside of the areas searched.
  5. The debris field was scanned but not detected.
  6. The BTO data set was somehow corrupted, and we are not properly interpreting it.

Although we cannot completely dismiss any of these possibilities, and each should be further explored, this article addresses the first in the list.

The challenge in finding automated paths ending further north than 26S is that the reconstructed paths need to curve to the left and decelerate to satisfy the BTO. What follows is one way this can occur while the aircraft is navigating on autopilot with no pilot actions after 19:41.

The automated flight path assumes that the flight computers were programmed to pass near Car Nicobar Airport (ICAO: VOCX) and fly towards Cocos Island Airport (ICAO: YPCC) with an intention to land there. (A route that includes flying towards VOCX, YPCC, and other airports was previously considered by Richard Godfrey.) Here, we assume that after programming the flight computers for a landing at YPCC, there were no further pilot actions. Furthermore, we consider that approaching YPCC, the automated flight plan caused the aircraft to turn to the left to align with the runway, to decelerate, and to fly over and continue past YPCC. This combination of left turn and deceleration is required to match the BTO data.

There are several explanations for why the flight computers might have been programmed for a landing at Cocos Island and then that landing not completed. One explanation is the pilot became incapacitated.  Some possibilities for incapacitation include a physical challenge from crew or passengers, or the aircraft was hit by hostile gun fire leading to rapid decompression of the fuselage. The possibility that MH370 was pursued by a Malaysian fighter jet was the subject of a previous article, and may have relevance.

Assumptions and sequence of events

The reconstructed flight paths are based on the following:

  1. FMC was programmed for automated flight between Car Nicobar (VOCX) and a landing at Cocos Island (YPCC) using the LNAV and VNAV autopilot modes at cruise altitude.
  2. The FMC was programmed for landing on Runway 15 using the RNVZ15 standard approach with PCCNE selected as the transition waypoint. (The selection of a transition waypoint does not significantly change the results.) In the aircraft’s navigation database, the approach would be defined as: APPROACH RNVZ15 FIX PCCNI AT OR ABOVE 1500 FIX PCCNF AT OR ABOVE 1500 FIX OVERFLY PCCNM 55 RNW 15 FIX PCCNH TRK 152 UNTIL 1500; TRANSITION PCCNE FIX PCCNE AT OR ABOVE 1500 SPEED 210
  3. Flying between VOCX and YPCC, the VNAV target speed was either LRC, ECON, the last speed constraint from the flight plan, or a speed selected in a VNAV screen. There was no speed intervention, i.e., the MCP speed window was closed.
  4. At the Top of Descent (ToD) about 110 NM from PCCNE, the pilot did not reset the altitude to a lower altitude, which constrained the aircraft to continue at the existing cruise altitude.
  5. At about 38 NM from PCCNE, the descent path calculated by the FMC would have reached 10,000 ft. The target speed would have reduced to 240 KIAS in accordance with the FMC’s default speed transition at 10,000 ft, even though there was no change in altitude from the cruise altitude.
  6. Approaching PCCNE, the VNAV target speed was automatically reduced to 210 KIAS in accordance with the programmed speed restriction at PCCNE, or the minimum maneuver speed (MMS), whichever is greater, with the aircraft remaining at cruise altitude. MMS was about 210 KIAS.
  7. At PCCNE, the aircraft turns towards waypoint PCCNI, and aligns with the runway on a track of about 152°M.
  8. Upon passing the runway and the final waypoint PCCNH, the FMC reaches an END OF ROUTE, the plane continues at the cruise altitude on a constant magnetic heading until fuel exhaustion. As the speed constraint for the runway is less than the MMS, the MMS becomes the target in the MCP speed window, and this speed is maintained for the remainder of the flight until fuel exhaustion.

The input variables that were varied are:

  • The starting position at 19:41. Since we are constraining the path to a great circle between VOCX and YPCC, only the latitude at 19:41 needs to be specified.
  • The VNAV mode, i.e., whether in ECON, LRC, or constant airspeed. If in ECON mode, there is an associated Cost Index (CI), which is based on the cost of fuel and time. For ECON mode at a given CI, and for LRC mode, the optimum speed varies as a function of aircraft weight and altitude. VNAV also commands throttle and pitch so that the speed and flight path adhere to any speed and altitude constraints programmed in the flight plan or selected in the VNAV screen.
  • The cruise altitude, which is assumed to be constant until the flame out of the first engine.

As the aircraft passes YPCC on a constant magnetic heading, the magnetic variation tends to slightly curve the flight towards the east as the magnetic variation increases from about 2.4°W near YPCC to about 2.7°W near 22 S latitude on the 7th arc. On the other hand, after passing YPCC, the wind is initially towards the west at 19 knots and 266°T, but weakens and changes direction towards the east between 16S and 17S latitudes so that at 18S latitude, it is about 10 knots at 81°T.

Results

A range of paths can be generated by sampling the input space and incorporating the uncertainty in BTO values, BFO values, wind, temperature, and MMS. One solution that is shown below is at FL320 and M.819, with a position at 19:41 about 25 NM south of VOCX.

Automated flight path passing over YPCC.

At the time the aircraft reaches the approach waypoints for YPCC, the MMS is about 210 KIAS, and remains at this speed for the rest of the flight. The aircraft would cross the 7th arc at about 22.0S latitude, which places it well north of what was previously searched.

The following table summarizes the flight parameters after 19:41 for this case (M.819 at FL320). The RMS error for the BTO is 26.0 μs and the RMS error for the BFO is 6.4 Hz with a mean error of -5.1 Hz:

Discussion

Ocean Infinity has expressed an interest in continuing the subsea search for MH370 at some time in the future. Options include

  • Scanning along the 7th arc at latitudes north of 25S
  • Scanning along the 7th arc at previously searched latitudes, but at a greater distance perpendicular to the arc
  • Re-scanning areas where the detection of the debris field might have been missed

Ultimately, the decision where to search must consider other aspects such as end-of-flight dynamics, drift modeling, surface search efforts, and fuel consumption, none of which were considered here. As such, this article is not a recommendation as to where to search next. Rather, this article was meant to provoke discussion about the possibility of an automated flight ending much further north on the 7th arc than was previously considered. Also, the article provides additional data for scenarios in which the pilot intended to land on Cocos Island but did not take the actions required for landing.

Acknowledgement

I am grateful for comments received from Mike Exner, Richard Godfrey, and @Andrew.

Update on July 4, 2018

Here are the results for another path, including the results from a fuel analysis. The path assumes that after a hold at Car Nicobar at FL250, the aircraft proceeds towards YPCC at FL320 and M0.8, and crosses the 2nd arc about 53 NM south of Car Nicobar. The following table summarizes the flight parameters after 19:41 for this case (M.8 at FL320). The RMS error for the BTO is 25.3 μs and the RMS error for the BFO is 6.0 Hz with a mean error of -4.6 Hz:

The fuel model is an improved version of a model I developed over one year ago, and is based on the drag-lift curves for a B777-200 that was presented in Ed Obert’s textbook entitled “Aerodynamic Design of Transport Aircraft”, with fuel flow – thrust relationships developed from descriptions in Walt Blake’s Boeing textbook entitled “Jet Transport Performance Methods”. Previously, I found that the model predicted the tabulated fuel flow data for LRC and Holding speeds with an RMS error of about 1%. The present model improves the prediction by introducing a correction factor that forces the calculated fuel flow to the exact tabulated values at the LRC and Holding speeds, and linearly varies the correction factor as a function of Mach number between those speeds. As such, the accuracy of the model between the Holding and LRC speeds should be very high. Also added to the model are calculated flow rates for climbs and descents, which assume full thrust and idle thrust, respectively, with vertical speed and flight path angle (FPA) directly calculated from the thrust and drag models.

The results of the fuel analysis are tabulated in this Excel file, which includes the remaining fuel at one minute intervals from 17:07 UTC until fuel exhaustion. At each time, the fuel flow is calculated as a function of weight, altitude, speed, and temperature. Assuming both engines fail at exactly the same time, fuel exhaustion is predicted to occur at 00:14 UTC. If the right engine fails before the left, the final (left) engine will fail at about 00:17. The predicted time of fuel exhaustion is consistent with our assumed fuel exhaustion at 00:17, considering the uncertainty in the actual flight path, the engine PDAs, and the meteorological conditions.

549 Responses to “An MH370 Flight Path Ending Further North on 7th Arc”

  1. DennisW says:

    Two not very relevant comments:

    “further” North -> “farther” North 🙂

    Every flight path I have ever created needed a small ROD at 00:11 to match the BFO. The satellite is “coming at” the aircraft at too high a rate to be offset by ground speed alone.

  2. TBill says:

    @Victor
    Nice summary of the current status, and nice flight path.

    I agree we need to work out example northerly flight paths to see if they make sense. I have a very similar flight path to yours, mine going POLUM to UXORA to fall in the trench below Zenith. One thing we have not done in my opinion is handicap and locate potential deep water targets, in case that was a goal.

    Your end point is also very similar end-point to Ed Baker, who feels MH370 may have stopped going south at the thick cloud coverage below 21-22S. It would be fun to help Ed by putting some meat(BTOs) and bones(BFOs) on his concept, which includes some, shall we say, elective maneuvers.

    On the surface, I do not like the BFO match of these paths, which is why I stopped working them in favor of 180S cases. But there could be some elective maneuvers accounting for the BFOs. Besides Ed Baker’s elective maneuvers, which could possibly help to match BFOs, I have considered if a rogue pilot may have tried to swing clear of COCOS to avoid visual or jet engine sound over the Island, and I am generally descending when twilight happens when I fly the route in PSS777 (but I calculate twilight which is approx 22:51 at 93.5S but further east here it may be earlier).

  3. John Halabarec says:

    @victor
    Whilst drift analysis may be coarse at best, are they not accurate enough to preclude a location so far north?

  4. Richard Godfrey says:

    @Victor

    An interesting hypothesis.

    Even without being hit by hostile fire or other possibilities you have considered, the idea of trying to reach Australia or an Australian dependency such as the Cocos Islands (YPCC) is plausible, in my view. Your scenario is based on no pilot in control and a pre-programmed route to YPCC, but the scenario is equally valid even when the pilot incapacitation came later in the flight. Whether the motive was to reach an airport that was open and could take a B777, or a country which would offer asylum, or another motive, I can only speculate.

    As you mention, I already thought about a similar scenario in my paper “The Long Hunt for a Diversion Airport” dated 18th October 2016. My big problem with the hypothesis is, that I have to rethink the drift analysis. I appreciate that a MH370 End Point at 21.9666°S fits with the reverse drift analyses of Meteo France, GEOMAR as well as Henrik Rydberg’s excellent work, but it does not fit with CMCC, NOAA, CSIRO or the studies from Nesterov and myself. It is possible we were all suffering from confirmation bias with the forward drift studies as Dennis has pointed out.

    I decided to enter a start point of 21.9666°S 103.5739°E in my drift model and let it run for 380 days, recalculating the historic average drift current every day. The result was that floating debris is approaching Reunion after 380 days, about 34 NM east of St. Andre where the Flaperon was found. The Flaperon was found after 508 days, but with the disturbances caused by Severe Tropical Cyclone Gillian that passed through the MH370 End Point area between 6th to 27th March with winds up to 220 km/h, then the debris would have probably been thrown off course for a few days. However, I do not believe even Tropical Cyclone Gillian can explain the Flaperon arriving 128 days early in St. Andre, Reunion.

    The Drift Simulation from 21.9666°S 103.5739°E for 380 days:

    https://www.dropbox.com/s/kcufbkrbc8bhosv/21.9666S%20103.5739E%20380%20days.png?dl=0

    The path of Severe Tropical Storm Gillian versus the nominal Drift Simulation track with a satellite picture of the storm taken on 23rd March 2014:

    https://www.dropbox.com/s/b3waczn8h5sw102/Gillian.pdf?dl=0

  5. Victor Iannello says:

    @John Halabarec: Based on what little I know about drift models, I’d say an impact at 22S latitude would require a delay between the arrival of debris near the shores of Reunion Island, Madagascar, and Mozambique, and the discovery and reporting of that debris. On the other hand, a northern impact would better explain the timing of the (first) discovery of “Roy” in South Africa. That said, others with more knowledge are welcome to chime in.

  6. DennisW says:

    @VictorI

    Your path does check a lot boxes. I see no reasons why it is not plausible. I don’t know enough about aircraft instrumentation to comment on the 169 degree LNAV flight path auto change to 150 degrees to accomodate Cocos runway 15/33.

  7. Paul Smithson says:

    @Victor and collaborators. Thanks for sharing this latest hypothesis. While I applaud the persistence with searching for paths that terminate near 7th arc north of the searched area, I fear that we are increasingly clutching at straws. To my mind, the conclusion that must be drawn from search results to date is that “close to 7th arc” is most likely mistaken. And it that’s the case, you either go for rapid decent (BFO interpretation fine) followed by miraculous recovery and glide. Or you go for BFO interpretation wrong, and no rapid ROD at 7th arc. I incline toward the latter, not least because we seem to have a problem in reconciling BFO-modelled ROD with the timing of the flameout.

    I’d suggest we start looking for another reason for a power-cycle. And a scenario in which an “unmanned” aircraft could have flown significantly beyond the 7th arc. I can think of only two main categories of hypothesis if we assume no pilot inputs since 1941 or earlier:-

    1) The autopilot remained engaged after second flame-out, enabling a long, controlled, unmanned descent.
    2) One engine was still running at 00:19:27 and final flame-out didn’t occur until several minutes beyond the 7th arc.

    I don’t think that either of these scenarios is outlandish. It’s just that we haven’t quite pinned down the detail on the electrical configuration scenarios that would fit.

  8. Victor Iannello says:

    @DennisW: For the left turn to occur, the FMC had to be programmed for a standard approach for YPCC. When composing the post, I thought of describing how that would be done, but I wasn’t sure many here would find that interesting. (I did opt to include the specification of the standard approach that would be stored in the navigation database.) In any event, this is the one automated sequence that I could conceive that would result in the left turn and deceleration required for more northern paths.

  9. Victor Iannello says:

    @Paul Smithson: I find it interesting that you describe exploring paths ending north of the searched area as “clutching at straws”, yet you hold out for unmanned descent ending outside of the search area. I’m open to either or both, as well as the other items on my list. We have time before the next search to decide which avenue might be most fruitful. I tried to make that clear.

  10. Paul Smithson says:

    @VI. Sure, that’s understood. No disrespect intended – hence I begin with a genuine acknowledgement. I think there is a growing queue of reasons to think that the “close to 7th arc” must be wrong and that persisting with it is tenuous.

  11. Victor Iannello says:

    @Richard Godfrey: Thank you for your comments. Yes, considering an impact at 22S requires us to rethink the drift analyses. I appreciate your willingness to do so, whatever you conclude. And yes, I agree that the incapacitation could have occurred later in the flight, but before the planned descent to Cocos Island.

  12. TBill says:

    @Paul Smithson and @DrB
    I am not opposed to saying final BFO’s could be questionable, but give me a reason, why the IFE did not reboot this time?

  13. airlandseaman says:

    Paul S: re: “…we seem to have a problem in reconciling BFO-modelled ROD with the timing of the flameout…”, please explain what you mean by this statement. What problem?

  14. DennisW says:

    @Richard

    There are reports of the flaperon arriving on ReUnion earlier than July. I have no idea how reliable they are. I seem to recall getting some pushback relative to this from people here.

    http://www.therakyatpost.com/world/2015/08/02/mh370-was-flaperon-actually-found-in-may-on-reunion-island/

  15. Richard Godfrey says:

    @Victor

    I still have to run the drift analyses beyond 380 days to see the fit to other floating debris items in all the other locations.

  16. Victor Iannello says:

    @DennisW: The link to the story you provided references this earlier story in the Daily Mail, in which there is discussion of other items washing ashore, including a water bottle made in Jakarta, Indonesia, and a suitcase. But as you say, it’s hard to verify any of this.

  17. Nederland says:

    @Victor

    I think this is a really credible path. I have long ago considered a scenario like this, but was unable to come up with those approach waypoints. There are many possibilities to explain pilot incapacitation shortly before arriving at Cocos Islands. Even the peer reviewed drift study by Jansen et al. (CMCC), based on multiple debris finds, still assigns a reasonable probability to locations as far north as 22S.

    https://www.nat-hazards-earth-syst-sci.net/16/1623/2016/nhess-16-1623-2016.pdf

  18. Paul Smithson says:

    @alsm. I don’t pretend to have any particular insight re “rapid descent doesnt’ fit flame-out timing narrative”. I’m referring to stuff that @Gysbreght has been saying since long before most folks were prepared to listen. @VI has alluded to the same more recently re [some] boeing simulations achieving the “required” ROD to fit BFO, but the timing is wrong. I’m sure that you are familiar with the back-and-forth.

  19. Bruce Robertson says:

    @Victor

    Perhaps a better model of MH370’s flight into the SIO is that of a driverless car on the great American salt flats. With the throttle weighted down, the car follows no particular path but is, instead, governed by the physics of the car’s suspension geometries and that of the lakebed surface and surface winds. Where the car runs out of fuel does not depend upon any lines on maps nor how the radio is tuned.

    MH370 was likely flying without reference to GPS, airways, or much of anything else. It was just trying to stay aloft for the longest time possible.

  20. Bruce Robertson says:

    Oh, and welcome to the Zenith Plateau — I thought you’d never get here! It’s been a lonely four years camped out here . . .

  21. Kirill Prostyakov says:

    Yay IG is joining the Zenith camp. For the Australian citizens in the IG, you could always file a FOI request for disclosure of ping recordings and other related data from JACC. Also seeing the data of 13-Apr oil slick sample would be quite interesting. Though really there is enough data already to justify finishing up the 2014 UAV survey in the vicinity of 2-Apr/5-Apr HMS Echo/ADV Ocean shield signals.

  22. Victor Iannello says:

    @Bruce Robertson, @Kirill Prostyakov: First, the article was not meant to represent the views of other IG members, nor was the article meant as my recommendation about where to search next. I’m not yet ready to make that recommendation. Rather, it was meant to provoke discussion about an automated path not considered before and to think about scenarios in which there was the intention to land on Cocos Island.

  23. Kirill Prostyakov says:

    @Victor, thanks for repeating the usual disclaimer. Of course, a path via an airstrip is much better scenario logic-wise than a kamikaze dash to Antarctica. I think @DrB had a fitting fuel usage solution for 200ish KIAS. This still has the end smeared along the arc. The current narrative needs an ontology stretching to make search practical again. The ping saga has left many loose ends which you never covered, so now might be a good time to add the existing work to the discussion. I have acoustic ray tracing model on Github and a GIS project with that search progress. Cheers and sorry for intruding.

  24. David says:

    @ALSM. I agree with Paul Smithson that,”…we seem to have a problem in reconciling BFO-modelled ROD with the timing of the flameout.”
    You ask him, “What problem?”.

    I think three at least of us who have expressed doubts as to whether the Boeing simulations, portrayed and commented on by the ATSB, are consistent with that timing. I include Gysbreght and Victor.

    The problem is that the data supporting the ATSB conviction have not been disclosed.

    I have gone further in my posts above, after posting this:
    https://www.dropbox.com/s/h3vwosq3yfq5pd5/THERE%20IS%20REASON%20TO%20THINK%20THERE%20COULD%20HAVE%20BEEN%20A%20GLIDE.docx?dl=0

    Iterating, at the recent Senate hearing Peter Foley indicated that residual trim was a cause of the spiral though Andrew has said that IF the aircraft had been trimmed using standard procedure there would be none.

    Foley also said the BFOs were a consequence of the aircraft entering a less-stable phugoid. As I have posted that seems unlikely, including the timing.

    I do think that further work, or at least disclosure, is needed.

    Yes there is a problem with timing, at least.

  25. Sfojimbo says:

    Your proposed scenario highlights the issue I’ve been trying to bring to everyone’s attention.

    You propose that at 19:41 the aircraft was at 8.744845° 92.896667°

    This is 76 minutes after the known location of the plane at 18:25 (6.735953° 96.018519°).

    Yet the distance between those two points is only 222nm, which at a GS of 486kts would have only taken a little over 27 minutes.

    So where was the plane for the other 383nm it traveled in those 49 unaccounted for minutes of flight?

    The starting point you have chosen on the 19:41 ring has a huge impact on the predicted end of fuel point for 9M-MRO. I see no justification for the 8.744845° 92.896667° @ 19:41 starting point for all subsequent calculations. You have a huge hole of 49 minutes and 383nm in yours and apparently everyone else’s calculations.

  26. Mick Gilbert says:

    First of all, to echo previous comments, thank you to Richard G for the regular OI updates, to Richard C for the early reports and animations, to Don and Mike for the clarifications and, of course, to Victor for making any of it possible. Needless to say that it was disappointing that OI didn’t locate the debris field but, as Mike alluded to earlier, there are some positive take-aways from this effort. I think that the most significant of those is that OI’s multiple independent AUV approach should dramatically alter the search planning paradigm. The old deep-tow search vehicle technology meant that the time and cost to search a given area was respectively relatively long and expensive. That time/cost consideration may not have necessarily driven a focus on planning localised searches over more tightly defined areas but it undoubtedly contributed to that sort of thinking. Enter the ‘hot spot’. To pick up on a point that Dennis has made regularly there are simply too many variables and the limited data was almost certainly not anywhere near good enough to formulate anything any better than a luke warm spot, at best.

    Now we have OI with the capability of knocking over something in the order of 120,000 sq.km in a 4-5 month campaign. They may not have commoditised underwater searching but they have certainly gone a long way to industrialising it. Just as you treat your approach to buying commodities differently to how you buy boutique or bespoke goods consequently OI have ushered in a different, and what should be a much simpler, approach to planning.

    Whether the search effort to date has eliminated the rapid descent end-of-flight profiles is highly debatable. Mike’s recent efforts with the Kota Bharu PSR data seem to suggest that the target was very high. What if it was similarly very high at first engine flame-out? Back of the fag packet, the aircraft may have already been established in a 2,000 – 3,000 fpm descent well above 35,000 feet when the second engine flamed-out. What does that do possible impact distances from the arc? What about the scenario proposed by Victor some time back whereby because of the electrical configuration (Left Main and Back-up Generators selected OFF with the Left AC Bus being powered off the Right AC Bus) the first engine flame-out, rather than the second, triggers the SDU reboot? Just as there are too many variables and too little data to accurately plot the airplane’s route I get a sense that there are similarly too many variables and too little data to accurately plot an impact point in relation to the 7th arc.

    Leaving alternate scenarios aside, even if you were to accept that the search efforts to date do eliminate rapid descent end-of-flight profiles, what does that mean for defining a new search area? You’re not going to just up stumps and shift your focus to the ±100 NM outer edge and work your way back towards the arc, are you? that doesn’t seem to make a whole lot of sense. Surely you would pick up where you finished searching and progressively work your way out.

    To pick up on something ventus45 posted a while back, the approach now should be on setting reasonable north-south bounds to the 7th arc and continue searching from the arc out towards the outer 100 NM bound using campaigns that maximise the area searched rather than honing in map pins. That probably means searching one side of the arc per campaign rather than trying to do both sides of the street. It’s not fancy but it is likely to be as effective as anything else.

  27. DrB says:

    @TBill,

    The IFE messages were not transmitted beginning circa 00:21 for one of these reasons:
    1. The aircraft crashed prior to 00:21.
    2. The APU ran out of fuel between 00:19 and 00:21.
    3. The IFE had been turned off between 18:28 and 00:21.

  28. Mick Gilbert says:

    @DrB
    @TBill


    4. The airplane was oriented or manoeuvring such that LOS between the antennas and the satellite couldn’t be maintained.

  29. David says:

    @Dr B, TBill, Mick Gilbert.
    The ATSB has never explained why the 89 secs from LOR. It seems to have been copied from the delay after the earlier 18:25 log-on. Don Thompson has estimated 152 secs (I think) as an appropriated minimum after 00:19:37.

    I think further testing is required, not that I have said so many times before! It is more than just the SDU which might have been rebooted then. Rather different than some ground logs-on, when the rest may well have been already on-line. Maybe the 18:25 also.

    There is a possibility of load shedding also with no generators going of the five available other than the APU, if indeed the power did come from that.

  30. Jsrue says:

    Couple of things with the coco island programmed path:
    1. The plane will not slow down unless it reaches 10000ft, only on reaching the 210kt point.
    2. The plane will possibly start circling over the last flyable point on the missed approach than fly straight when no action is input.

  31. Victor Iannello says:

    @sfojimbo: There is no “issue”. I have been proposing the possibility of a loiter near Sumatra since July 2014.

  32. David says:

    On looking into it, after earlier discussions with Don Thompson I posted, “he leans to the 156 secs” (of the 12:50 log on, 7th March) as indicative of the minimum to be expected of a cold log-on.

  33. Victor Iannello says:

    @Jsrue: I’m sorry, but you are wrong on both points. The VNAV target speeds will be commanded even if the altitude is constrained by the MCP altitude, and a constant magnetic heading is maintained upon reaching a route discontinuity.

  34. David says:

    Also, ‘LOR’ first line should read ‘log-on’.

  35. Sfojimbo says:

    There is no “issue”. I have been proposing the possibility of a loiter near Sumatra since July 2014.

    I thought that you had given up on that years ago. It seems that you just decided to not talk about it so much.

    Victor, what is the basis of this “loiter”?

    The only tangible information we have about that portion of the flight is that the Malaysians say that they lost radar contact at MEKAR + 10 miles (which checks out in ball park terms.

    Three minutes later at 18:25, there was an Inmarsat transaction that provided six BFO indications over an eight tenths of a second period which indicated that 9M-MRO had a northerly component to its flight path.

    Three minutes after that, at 18:28 came an Inmarsat transaction that provided the 18:28 ping ring which lines up well with the “Malaysian last radar sighting”.

    The only other tangible piece of information for that period is an Inmarsat transaction that took place at 18:39:55.354 which was the result of an attempted phone call to 9M-MRO which provided a BFO reading but no BTO data. The BFO indicated that 9M-MRO was on a southerly course at that instant in time.

    There are no other messages, sightings, phone calls or anything else until the 19:41 ping ring to base any “loiter near Sumatra” theory on. Nothing, nada, zip!

    There are people who do random things without purpose, but Captain Zaharie Ahmad Shah was never one of those. Zaharie Ahmad Shah was purposeful and methodical to the extreme. There is no reason whatsoever to think he was operating his aircraft in the most efficient manner possible (as he had been doing from IGARI), and the Inmarsat data informs us that he was southbound towards the nether regions of the Southern Ocean.

    This “loiter” is based 100% on a figment of someone’s imagination with no data or logic to support it.

    BTW, this all means that his altitude (for the weight of his plane) would have been about 17,000 feet higher than the 32,000 that you projected in your chart above; therefore, it follows that his ground speed would have been higher than the 486kts predicted above. That means that there is more than just 383nm being swept under the rug above.

    That 400nm plus, places the potential resting place of 9M-MRO 400nm farther south than where the search has just taken place.

  36. Sfojimbo says:

    There is no reason whatsoever to think he was NOT operating his aircraft in the most efficient manner possible

  37. TBill says:

    @DrB @Mick @David
    Thank for the IFE list.
    I was thinking if MH370 had a long glide, consistent with 100nm off the arc, then we are talking something like 00:28 is the ditch time, if I recall from Rob’s scenario. But DrB is saying it could have been turned off anytime after 18:28. I was thinking it would come back on automatically.

  38. Sfojimbo says:

    What possible difference does it make whether he had a long glide or nosed it in, if you are throwing a 400nm error out the window?

    And some people here are worried that the ping rings might have been a couple of miles off!

  39. DrB says:

    @DennisW,

    You said: “Every flight path I have ever created needed a small ROD at 00:11 to match the BFO.”

    My 181.2 degree CMT route does not have this problem. It has a predicted BFO of 249 Hz at 00:11 with zero ROD compared with the measured 252 Hz.

    Victor’s Cocos route has a -12 Hz BFO error at 00:11, which seems to require a significant step in the BFO bias to have occurred at the 18:25 SDU restart. So far, we have no evidence this could or did occur, nor do we have proof it could not.

  40. DrB says:

    @TBill,

    My understanding is that there are IFE circuit breakers accessible on the flight deck. If they are turned off, the IFE can’t operate no matter what happens to the SDU.

  41. DennisW says:

    @DrB

    Yes, your error is similar to the errors I was referring to i.e. 4-5 Hz. I said it was a small ROD.

    You might want to refer to Figure 5.4 of “Bayesian Methods…” which shows BFO errors on the order of 20 Hz.

  42. DrB says:

    @David,

    If one uses 156 seconds for the SDU to transmit from a power-up, then MEFE would occur circa 16:54 instead of at 17:30 per ATSB. That difference in time in insignificant compared to other errors in calculating fuel flow and predicted endurance.

    It may matter in terms of the time required to reach BFO-compatible RODs, based on Boeing’s simulations. Going on memory, I think the issue was that BFO-compatible RODs were seen in a few cases, but then only several minutes later than 00:19.

    Suppose the L and R AC buses were both being powered by the right engine when it flamed out. That means the L engine was still running when the SDU was rebooting, using APU power, and it would continue to provide thrust for another 8 minutes, or until 00:25 if the R engine was fuel exhausted at 00:17. That obviously could explain flying well past the 7th Arc before a fast descent began. But level flight (with the L engine still thrusting) doesn’t fit our current interpretation of the 00:19 BFOs, and this inconsistency may doom this scenario.

  43. DrB says:

    @DennisW,

    You might want to refer to the Inmarsat paper, which says the BFO error is less than 7 Hz. Inmarsat built the system. They have more credibility on this issue than DSTG. We’ve been through this before. You can trust everything DSTG said if you want. I don’t.

  44. DennisW says:

    @DrB

    The figure 5.4 data is measured. It does not represent a DSTG opinion. Yes, I recall the Inmarsat paper 7Hz value. They did not say it was peak +/-7 Hz, RMS, or peak to peak. Their 7 Hz statement is vague and ambiguous. Plus that, no one responsible would quote a peak error. Life is a bell curve.

  45. airlandseaman says:

    Paul and David: I am still waiting to hear what you think the problem is with “timing”. It has all been explained many times. MEFE circa 00:17:30, APU on line circa 00:18:30 and LOR 00:19:29. What specifically do you disagree with in this timing?

    Foley never claimed to know precisely what happened. He only gave some examples of what could have happened to explain the descent and BFO values observed.

    In all our Level4 simulations back in Nov 2014, with any any rudder trim, there developed a spiral descent exactly like those described by Boeing. I reported this in November 2014. Paul M., the senior UAL 777 captain that arranged the simulator time, went through the SOP for trimming at altitude. Most of the UAL 777 fleet have a need for some trim at cruise to minimize fuel expense. But after fuel exhaustion, the TAC disengages automatically and the residual rudder trim remains, causing the spiral. Even when we set the trim to neutral and the tanks to run dry simultaneously, a spiral developed. It just took longer.

    In short, a spiral can develop rather rapidly if there is some residual trim. That appears to be what happened to MH370.

  46. DennisW says:

    @DrB

    We’ve been through this before.

    Yes, we have. The oscillator drift characteristics are neither stationary nor ergodic. That is what lead Dr. Allan to develop the Allan variance which is not the same concept as the standard variance we have all come to know and love (and misuse). The DSTG explanation of the drift characteristics is, indeed, laughable.

  47. Sfojimbo says:

    His altitude would have been 7,000 feet higher that your 32,000 feet.

    Please correct this when you finish your “moderation”.

  48. David says:

    @Dr B. The 156 secs is from log-on to first IFE connection, not time to transmit or log-on.

    In your last para’s scenario I take it you mean that the left back-up gen was inoperative also or the APU would not have auto-started. However without A/P and A/T the aircraft would be in a right spiral at log-on and that could be compatible with the BFOs though given 2 mins in that asymmetry maybe the descent rate at LOR could well have been higher than the BFO figure.

    Were there that extra fuel and the aircraft was in that configuration it would be likely that the aircraft would crash close to the 7th arc anyway, intact or having broken up. It would be reasonable I think to suppose it would not straighten from that spiral.

    Incidentally, going back to the difference I alluded to of an engine-less APU electrical supply when compared to engine running is that there would be no back-up generator in the former so the APU would be powering both transfer buses I believe, an extra load, and hence my comment about the increased likelihood of load shedding affecting IFE connection.

  49. Don Thompson says:

    David, Dr B,

    Considering the Log records for the final Log On at 00:19, another possible scenario it that the aircraft did not reliably receive the final Log On Acknowledge transmission from the GES (a forward channel transmission recorded by the GES at 00:19:38).

    The outcome from such a failure would be that the AES retransmis its Log On Acknowledge SU after a specific timeout. That timeout is 10 seconds (there are examples of this behaviour earlier in the Log).

    I am merely pointing out that a burst from the AES could have been expected as early as 00:19:48. Of course, that would reinforce a terminus close to the 7th arc.

  50. Jsrue says:

    Thanks for your response. I think the 10000ft speed constraint point is calculated realtime by the fmc based on the descent constraints set on the fmc page and is for notation only in the path display which varies as descent parameters change during a descent, the actual slow down by the autopilot is done based on reached altitude alone.

  51. Don Thompson says:

    Dr B,

    You wrote, “Inmarsat built the system”.

    That isn’t entirely correct. Inmarsat specified the system and many other parties interpreted Inmarsat’s specifications to build components of the system: Thales (née Racal) designed and manufactured an AES (SDU, RFU, HPAs), SED designed and integrated the 2013 delivered GES (using Square Peg’s tech), Inmarsat-3 F1 built by Lockheed Martin.

    A complex specification and test-validation process. Difficult environment in which to investigate ‘what-if’ scenarios, those possible ‘edge’ cases that weren’t set out in the test and integration programmes.

  52. David says:

    @ALSM. This problem with timing is not whether there was a descent at that rate but whether the descent rates of the 2 BFOs would match 2 mins, and 2 mins 8 secs, after loss of AC power.

    It is all very well to assert that there were high descent rates and even a high acceleration in those Boeing simulations. Indeed, doubtless there were: high descent rates were reported at least.

    But the timing to match has not been demonstrated. If for example those simulator descents were all seen much later after the power loss, they do not in fact demonstrate that residual trim could be the cause of the descent actually experienced, the same as if there were high descent rates but a lot more gradual acceleration between than has been assessed.

    As you may have read, I have gone further in saying that for the transmissions to be captured just the moment that the aircraft was in the high descent phase of a less-stable phugoid, one that developed at the right timing, would be extraordinary. It is not just a qualitative simulation that is necessary but one taking account of probabilities of actual realisation. After all, so many assumptions to date have been based on probabilities, including the high lift devices being housed, whether there was a pilot at the end and search width.

    I repeat that the information before us does not substantiate the claims that the diagrams and descriptions of the Boeing simulations emulate the actual descent the BFOs depict.

    As to your simulations, it remains reasonable to assume there may have been no trim, unless I have misunderstood @Andrew. If Boeing did not simulate that, that just joins other ingredients which could make a difference and which have not been simulated, including a relight. The possibility of that was acknowledged but then left aside with no accompanying explanation – such as it was improbable or would have little effect.

    Yet you seem to see simulations which can be seen as unrepresentative, and about which no data that could allay the above criticisms has been provided, as valid anyway.

    Of your simulations you say, “Even when we set the trim to neutral and the tanks to run dry simultaneously, a spiral developed. It just took longer.” If it takes longer does that mean compliance with the BFO timing of not?
    You then add, “In short, a spiral can develop rather rapidly if there is some residual trim. That appears to be what happened to MH370.” I point you at the remark I made above about a qualitative assessment and whether this is not in effect making the facts fit.

    I emphasise again that these simulations have limitations, non-replication of outside-the-envelope flight being another, which undo their veracity but still you accept that they are satisfactory when even qualitatively, currently they do have serious weaknesses.

    Finally about your account of ‘all’ your simulations, one thing you overlook is that when you simulated a left-engine-first failure and the aircraft then banked, the bank later flattened and the aircraft pulled out for a while.

    Hopefully you will not see need to re-iterate your, “I am still waiting to hear what you think the problem is with “timing” since I can add no more. Maybe @Paul Smithson can.

  53. Andrew says:

    @David

    Finally, my take on the engine restart/APU autostart problem:

    MH370 – The Left Engine Restart/APU Auto-start Problem

  54. David says:

    @Don Thompson. So what you are saying is that another AES transmission could have been expected at 00:19:48 and since that was not received the SDU was no longer transmitting even then?

    @Andrew. Thanks Andrew. Will look forward to taking a look tomorrow, now being cross-eyed. (Besides there is the Australia vs Ireland rugby…)

  55. Gysbreght says:

    @Andrew: Thank you for an excellent paper that discusses many factors that need to be considered in relation to the engine restart/APU autostart problem.

    However, there is one aspect that you seem to have overlooked. On page 2 you write:

    “According to the ATSB, “the difference in location between the left engine fuel inlet and the APU fuel inlet would result in approximately 30 lb of fuel being available to the APU after a left engine fuel exhaustion”.7”

    The ATSB statement is at best misleading, certainly when read without the qualification that the ATSB adds:

    Auxiliary Power Unit (APU) fuel inlet

    The APU fuel inlet is located in the left main tank. The APU is estimated to consume (when electrically-loaded) approximately 2 lb of fuel in 55 seconds. In a standard flight attitude (1° pitch), the difference in location between the left engine fuel inlet and the APU fuel inlet would result in approximately 30 lb of fuel being available to the APU after a left engine fuel exhaustion. From this information, the APU had a maximum operating time of approximately 13 minutes and 45 seconds. The pitch attitude would have an effect on the usable fuel for the APU; an aircraft not under control may exhibit dynamic changes in pitch attitude (i.e. phugoid[2] motion) which could have limited the APU’s ability to receive fuel. In-flight acceleration forces could also affect the distribution of fuel in the tanks.

    The quantity of approximately 30 lb of fuel, determined in a standard flight attitude (1° pitch), is not available to the APU after the flame-out of both engines, because the loss of propulsion wil result in changes of attitude and longitudinal acceleration of the airplane that will move the fuel forward in the tank. Since the APU fuel inlet is at the rear of the tank, that movement of fuel will reduce the quantity available to the APU, possibly to nothing at all. In that case the only fuel available to the APU would be that contained in the fuel line between the tank and the APU.

  56. Don Thompson says:

    David,

    Yes, in the range of possibles, the AES may have failed to decode to the LOA transmitted by the GES, in which case it would have retransmitted its LOA.

    Ireland vs Australia, disappointing result, but close! The Irishmen were perhaps confused & poorly aclimatised: today, it’s warmer here than in Brisbane.

  57. David says:

    Hooray. The vortex generator to be collected and hopefully, examined.

    https://www.airlineratings.com/news/malaysia-finally-collect-crtical-mh370-debris/

  58. David says:

    @Don Thompson. Thanks.
    But disappointing result be blowed.

  59. airlandseaman says:

    Andrew: Thanks for your detailed analysis of the engine restart/APU autostart problem. I think your analysis is consistent with my interpretation of the simulations. I would add a couple of points.

    1. We do not know the altitude at 00:19. If the altitude remained very high, as observed at KB and BU (>40,000 ft), it is possible that the APU took more than the assumed 60 seconds to start up, in which case the BFO implied descent rates would be more consistent with the simulator timing.
    2. We do not know the rudder trim position. In all our simulations in the Level D simulator, the trim position had a significant effect on the how soon after MEFE the spiral descent started. It is certainly possible that the rudder trim was set such that the descent started more quickly than what we observed in the simulator.
    3. If there was a brief restart, it is very likely that the attitude changed significantly, causing a high rate of descent to begin sooner.
    4. Regardless of the fuel available in the left tank, there was more than enough fuel in the APU fuel line to power the APU for at least a few minutes after MEFE, even if there was a restart.

    Given all that, I see no reason to dwell on the timing questions as David and Paul seem inclined to do. They do not matter. What we know is that the BFO values indicate a high rate of descent, even allowing for some OCXO offset error in those values.

  60. Victor Iannello says:

    @sfojimbo: You obviously are not aware that the possibility of a descent and loiter has been well-discussed on this blog. You also do not understand the difference between presenting a possibility and being stubbornly certain about a scenario. More northern paths require some delay before the turn to the south. We can only guess what occurred between 18:28 and 19:41, so some of us have generated paths that start at 19:41. You seem quite sure of what occurred and what did not occur in the time interval. Yet, you offer zero evidence, and you provide no complete path to study.

    Your comments will continue to be subject to moderation because you are repetitive and combative. I’ve certainly given you the floor to present your case, and you have presented nothing of value other than being sure of what the captain had in mind.

  61. airlandseaman says:

    Gysbreght: The estimated fuel in the APU fuel line was sufficient to power the APU even if the tank was empty. Moreover, the attitude had no effect on the fuel in the line. So all the discussion about the 30 lbs of fuel in the tank and attitude are irrelevant.

  62. Julia says:

    @Victor
    This is from your blog of 3/2/2017 about a possible Malaysian military fighter jet
    “Another interesting aspect occurs when we extrapolate the path of the second aircraft backwards in time. Doing so, we find that the path crosses a point just to the south of Runway 18 of Butterworth Air Field near Penang. This raises the possibility that the second aircraft departed from from Butterworth and chased MH370. Richard Godfrey discovered that this point to the south of Butterworth falls very close to waypoint UPTOP.”
    Do you think the new Malaysian Government might clear this up/reveal details of this? The presence of a jet at this location seems pretty vital information. I am astonished that this detail is still kept secret from presumably the ATSB, IG (definitely), and all the other official bodies involved in the search.

  63. Gysbreght says:

    @airlandseaman: “So all the discussion about the 30 lbs of fuel in the tank and attitude are irrelevant.”

    Unless you want to throw engine relight into the mix.

  64. Victor Iannello says:

    @DrB said:Victor’s Cocos route has a -12 Hz BFO error at 00:11, which seems to require a significant step in the BFO bias to have occurred at the 18:25 SDU restart.

    The BFO error at 00:11 is higher than the others. It’s possible that the aircraft was not maintaining steady altitude and attitude at that time. @sk999 has shown that there is considerable variation in the BFO during a descent. That would be even more pronounced if the speed was close to stick shaker speed, at which time I think the BFO could vary in either direction. Just a thought.

  65. Victor Iannello says:

    @Julia: There have been rumors for some time that jets were scrambled from Butterworth. However, it is just a rumor. There might be nothing to reveal.

  66. TBill says:

    @Victor
    can we move the holding pattern to after Arc3? I’ll check.

  67. Julia says:

    @Victor
    Just seems an easy rumour to dispel

  68. airlandseaman says:

    Gysbreght says:
    June 9, 2018 at 8:58 am “Unless you want to throw engine relight into the mix.” Wrong again. Are you ignoring what I wrote? A relight has no effect on the fuel in the APU fuel line.

  69. Gysbreght says:

    @airlandseaman: Where do you get the fuel required to relight the main engine?

  70. Godfrey says:

    First time post so be gentle…

    “a constant magnetic heading is maintained upon reaching a route discontinuity”

    I think what Jsrue was referring to is that most approaches have a subsequent missed approach path and holding fix. In the case of Helios the aircraft flew to the holding fix and then flew around in circles until fuel exhaustion.

    I don’t know about the Malaysian database and I don’t know about the approach into the Cocos islands.

  71. Victor Iannello says:

    @Godfrey: Some approaches do have a hold. However, I provided the specification for approach RNVZ15, and there is no holding fix, so an END OF ROUTE would occur. We also know that MH370 did not hold near Cocos Island because it is far from the 7th arc.

  72. Gysbreght says:

    @Andrew: Maybe a dumb question:

    One of the conditions you list for the APU fuel feed system supplying fuel to the left engine is “The left engine N3 is less than the engine run speed.”

    Is that fuel supply shut off after relight when the left engine N3 exceeds the engine run speed?

  73. airlandseaman says:

    Gysbreght says:
    June 9, 2018 at 9:35 am
    @airlandseaman: Where do you get the fuel required to relight the main engine?

    You know exactly where is comes from. Certainly not from the fuel in the APU fuel line.

  74. Gysbreght says:

    @airlandseaman: Can I take that as an admission that you were wrong when you said “So all the discussion about the 30 lbs of fuel in the tank and attitude are irrelevant.”?

  75. Sfojimbo says:

    Victor

    Once again you respond to a technical argument with a personal attack. The only information available between the 18:28 ping ring and the 18:41 ping ring comes from a single BFO reading at 18:39:55.354 which indicated that 9M-MRO was on a southerly course at that instant in time.

    Thus the “loiter” is based 100% on a figment of someone’s imagination with no data or logic to support it.

    You say that you’ve discussed the reasoning behind the “loitering” hypothesis; when was that, July of 2014? I can’t track down everything you’ve said over the last four years.

    So far in this thread, you’ve refused to own up to holding a belief in a sortie to the north and now that I’ve forced your hand, you accept that you are basing your calculations on the premise that 9M-MRO flew in circles for 49 minutes, but are unwilling to discuss your reasoning for this technically unsupported belief.

    This is in spite of the fact that the search for the wreckage that you have supported and very possibly provided input for, has come up empty time and again.

    You and this “independent group” have an obligation for truth. There is no way for a layman, or even a relative of one of the victims, to know that all the technical mathematics discussion here is based on a goofy, unsupported theory that has been kept in the background and beyond discussion.

    You and the other participants here, present yourselves as experts above reproach and in doing so you are very possibly suppressing the opportunity for someone else to see a need for an independent, open investigation of what happened to the Malaysian airliner.

    It’s time for you to lay your argument on the table.

  76. airlandseaman says:

    Gysbreght: You are clearly in troll mode again, so I have nothing to add. I trust most people here understand what I have said.

  77. Victor Iannello says:

    @sfojimbo said: The only information available between the 18:28 ping ring and the 18:41 ping ring comes from a single BFO reading at 18:39:55.354 which indicated that 9M-MRO was on a southerly course at that instant in time.

    That’s one possibility. The possibility of a descent at 18:40 has been thoroughly discussed here. In fact, I wrote an article on it back in March 2017. The last paragraph in that article states:

    The timing of MH370’s final turn to the south has an important impact on the estimated end point along the 7th arc. The later the timing of the turn, the further north the end point is predicted to be. Although the BFO values at 18:40 UTC recorded during a 1-minute interval suggest the plane was flying at constant altitude and had already turned south, an alternative interpretation is the plane was still traveling northwest but was descending. Here, we find that the combination of groundspeed and vertical speed that is required to match the BFO at 18:40 also corresponds to a typical descent rate of around 3°. We also find that over the 1-minute interval, the change in BFO that is expected due to this descent is small compared to the noise in the BFO values that were recorded. An autopilot mode that minimizes the variation in BFO is V/S at -2600 fpm.

    As for references to a loiter in the Andamans, this has been discussed MANY times by me and others. For instance, two places are here (links included):

    1. Yes, the plane descended for a reason. As people here know, I lean towards the guilt of the captain, so I don’t put a lot of weight on explaining the descent with a scenario involving a failure. If the pilot intended to loiter before he turned south, the descent might have been related to lowering fuel flow by holding at FL200. (Comments from others on this page also discussed the loiter.)

    2. There was an unanswered satellite phone call to MH370 at 18:40 UTC which produced BFO values, and these can be used to infer the track of the aircraft at this time. If the flight was level at 18:40 UTC, the track would be towards to the south. For this reason, the BFO values at 18:40 UTC have been used in the past as justification for a turn to the south preceding the call at 18:40 UTC. However, recognizing that vertical speed has a strong effect on the BFO of the received signal, it is possible that the plane was still traveling to the northwest at 18:40 UTC and descending at about 2900 ft/min as it approached Car Nicobar Airport. Understanding that fuel flow would have been minimized if MH370 flew at a holding speed of around 293 KTAS and at a pressure altitude of around 20,000 ft, it is possible that MH370 reached Car Nicobar around 18:53 UTC and entered a holding pattern, as shown by the racetrack pattern near Car Nicobar…

    @sfojimbo said: You and the other participants here, present yourselves as experts above reproach and in doing so you are very possibly suppressing the opportunity for someone else to see a need for an independent, open investigation of what happened to the Malaysian airliner.

    With that statement, you demonstrate what a buffoon you are. The participants here do not suppress others from independently investigating the disappearance, as you claim. In fact, I STRONGLY encourage you or anybody else that feels similarly to form an independent group to investigate the disappearance of MH370. To be truly independent, your group you should self-assemble, as the IG did.

  78. @David
    You said: Hooray. The vortex generator to be collected and hopefully, examined.
    https://www.airlineratings.com/news/malaysia-finally-collect-crtical-mh370-debris/

    I took a look at the photo showing a sideview of the engine. Is anybody thinking that the Rolls Royce emblem (either that found earlier or another), might have ripped off the fin? If so, the emblem travelled about 25 degrees upward from the picture horizontal. Might this provide any clues as the attitude of the aircraft when it entered the water?

  79. HD says:

    @All

    My thanks to Victor for creating and maintaining this search blog forum, and the free flow of search information about MH370.

    I tip my hat to all of the independent contributors and IG members, who have labored tirelessly the past four years, but were definitely hindered by the limited amount of data and information to work with. I commend you all for generously sharing your analytical expertise.

    Everyone wanted closure for this long search, and hoped for OI’s success. But since most of the priority areas along the 7th arc have been searched, the question on everyone’s mind seems to be:
    “Are there any viable locations for a future search effort?”

    I believe the answer to that question is “YES”, and I hope this research information can be of some assistance for a future search effort.

    I want to thank several contributors on this blog, and other sources, who unknowingly provided key pieces of information for a flight path I have been working on the past 4 years. Some of the information was vitally important, which helped me construct a plausible flight path, and fit together several missing pieces of this complex puzzle.

    The 2 PART – Comprehensive Report about MH370 was recently finished, but until OI’s search was complete, I was hesitant about posting the details, since some of the basic assumptions are different from those used for plotting previous flight paths.

    PART 1 is posted below and PART 2 will be posted here in a couple of days.

    The title for PART 1 is:
    “WHY THEY HAVE NOT FOUND MH370”,
    and is divided into three Dropbox files:

    #1: “PRIMARY RADAR”
    #2: “FLIGHT PATH & AIR SPEED”
    #3: “DEBRIS DRIFTING and SEARCH SUMMARY”

    #1: “PRIMARY RADAR”
    https://www.dropbox.com/s/6gmv0ekwpn0btpi/%231MH370%20–%20Primary%20Radar%20-%20Pgs%201-4.pdf?dl=0

    #2: “FLIGHT PATH & AIR SPEED”
    https://www.dropbox.com/s/hodhaablvncpcgi/%232MH370%20–%20Flight%20Path%20%26%20AirSpeed%20-%20Pgs%205-10.pdf?dl=0

    #3: “DEBRIS DRIFTING and SEARCH SUMMARY”
    https://www.dropbox.com/s/4crq8pvhsk7l40b/%233MH370%20–%20Debris%20Drift%20%26%20Summary%20Facts%20-%20Pgs%2011-16.pdf?dl=0

  80. airlandseaman says:

    HD: I know of no analysis by anyone that suggests a speed of only 160 kts circa 18:25. The average derived from 7 BTO values was 332 kts. Details here: https://goo.gl/yDErGY

  81. DrB says:

    @DennsW,

    You said that DSTG’s Figure 5.4 was measured. It is not. It is the difference between a measured BFO and DSTG’s predicted BFO. I suspect some of the outliers are large because DSTG did not use the correct aircraft parameters for the predicted BFO.

    @DonT,

    My statement that Inmarsat “built the system” is correct in the traditional aerospace usage of “built”. Nobody manufacturetes every component of any electronics assembly. They design, buy some components, make some components, assemble, and test. That’s what Inmarsat did. A more important consideration is that Inmarsat collects and processes years worth of data from thousands of aircraft. That experience cannot be replicated by DSTG with limited data access and a relatively short time frame”

  82. DrB says:

    @Victor Iannello,

    You said “@sk999 has shown that there is considerable variation in the BFO during a descent. That would be even more pronounced if the speed was close to stick shaker speed, at which time I think the BFO could vary in either direction. Just a thought.”

    Could be, but it’s a bit inconsistent to claim the BFO variation during a minute-long 2600 fpm descent at 18:40 is small but at 00:11 it could be 12 Hz.

  83. DrB says:

    @DennisW,

    I don’t think there is any ambiguity in Inmarsat’s BFO peak error. Their figures clearly show +7 Hz and -7 Hz error bounds.

  84. DennisW says:

    @DrB

    You said that DSTG’s Figure 5.4 was measured. It is not. It is the difference between a measured BFO and DSTG’s predicted BFO. I suspect some of the outliers are large because DSTG did not use the correct aircraft parameters for the predicted BFO.

    Good grief. You think the DSTG is incapable of calculating a BFO based on a known ground track, ground speed, and location? Plus that, they highlighted that result so it was given special and careful attention.

  85. DennisW says:

    @DrB

    I don’t think there is any ambiguity in Inmarsat’s BFO peak error. Their figures clearly show +7 Hz and -7 Hz error bounds.

    So, you think it makes sense to specify a peak BFO error? In the case of oscillator physics the cognoscenti don’t even specify an RMS error.

  86. DrB says:

    @DennisW,

    I would not do it that way, but oscillator drift is not the only contributor to BFO error. I consider Inmarsat’s simplified approach a reasonable choice.

  87. DrB says:

    @DennisW,

    How well known were the aircraft parameters used by DSTG for the BFO predictions? The ACARS data is one sample per 5 minutes, and these times are asynchronous with SATCOM messages yielding BTO and BFO. There will be some errors, possibly large if a maneuver occurs between a pair of ACARS position reports. due to interpolating ACARS data to BTO/BFO epochs.

  88. DennisW says:

    @DrB

    My assumption is that the subject flight was on autopilot. The data in Figure 5.4 spans several hours. The histogram of errors from 20 flights shown in Figure 5.5 shows a standard deviation of 5.5 Hz with many samples exceeding the Inmarsat peak value of 7 Hz.

  89. David says:

    @Neville Macaulife. A part of the intake cowl could have hit the vortex generator, the cowl evidently having disintegrated judging by the RR size. However the less exposed fan cowling to its rear on which the vortex generator is mounted apparently broke up, quite possibly being buckled by compression, though there was a quite large section of a right hand fan cowling recovered (item 6 of the Malaysian listing). This might have been behind the damage the generator exhibits also.

    I think it is equally possible that part of the cowling with this right engine vortex generator attached hit something and that both broke the cowl into small pieces and took off the vortex generator’s fin. What you see is part of the base to which the fin had been mounted. A like vortex generator recovered from MH17 wreckage had not lost its fin.
    Besides, of interest is how this one was detached so cleanly.

    Also of as much interest to me is the missing paint on the inside of the cowl fragment to which it is attached, when compared to the finish on the inside of item 6 fan cowl and the appearance of the surface under some missing washers.

  90. Victor Iannello says:

    @DrB said: Could be, but it’s a bit inconsistent to claim the BFO variation during a minute-long 2600 fpm descent at 18:40 is small but at 00:11 it could be 12 Hz.

    It’s not inconsistent when you consider that the descent at 18:40 might have been in V/S mode and the descent at 00:11 would not be.

  91. Andrew says:

    @Gysbreght

    RE: “However, there is one aspect that you seem to have overlooked.”

    Thanks – point taken. The ATSB refers to a ‘standard’ pitch attitude’ of 1° in relation to the 30 lb fuel figure. It’s worth noting that in the simulator the pitch attitude at the time of the second engine flameout is about 5°, due to the reducing speed after the first flameout. About 90 seconds later, the pitch attitude had only dropped to about 4°, the speed was much the same and the aircraft was in a 30° banked turn to the right, with a rate of descent of about 800 ft/min.

    RE: “One of the conditions you list for the APU fuel feed system supplying fuel to the left engine is “The left engine N3 is less than the engine run speed.”

    Is that fuel supply shut off after relight when the left engine N3 exceeds the engine run speed?”

    The system is designed to allow the APU DC fuel pump to supply the left engine via the APU isolation valve. That’s to allow a quick relight if there’s a dual flameout and the boost pumps stop working due to the loss of AC power. The manuals don’t state what happens to the DC pump and isolation valve if AC power is subsequently restored by an engine restart (however brief) or APU autostart. In short: I don’t know!

  92. Andrew says:

    @ALSM

    RE: “1. We do not know the altitude at 00:19. If the altitude remained very high, as observed at KB and BU (>40,000 ft), it is possible that the APU took more than the assumed 60 seconds to start up, in which case the BFO implied descent rates would be more consistent with the simulator timing.”

    I agree, but the vertical acceleration also needs to be greater than was observed in the simulator.

    RE: “2. We do not know the rudder trim position. In all our simulations in the Level D simulator, the trim position had a significant effect on the how soon after MEFE the spiral descent started. It is certainly possible that the rudder trim was set such that the descent started more quickly than what we observed in the simulator.”

    If the aircraft is trimmed correctly, it should not roll when the autopilot disengages. The point of trimming the rudder in the cruise is to remove any rolling tendency by zeroing the lateral control input. That ensures the spoilers are not deflected, reducing drag. The Boeing recommended technique is to zero the index on the top of the control column, but some pilots do it more accurately by bringing up the flight controls maintenance page on the lower EICAS and adjusting the rudder trim until the spoilers have zero deflection (I think that’s the technique Paul demonstrated to you?).

    As I see it, the high rate of descent needs to develop less quickly than was observed in the simulator. It’s already been mentioned that the B777 seems to need about 1-2 units of right rudder trim in the cruise. If the aircraft is not trimmed correctly, then it’s highly likely it wasn’t trimmed at all (some pilots don’t bother), in which case the rudder trim would be set at zero, where it should have been for take-off. If the trim is set at zero and the aircraft needs right rudder trim, the autopilot will provide a right roll input to keep the aircraft flying straight. If the autopilot disengages, the aircraft will then roll left. Such a scenario might reduce the right yaw/roll rate that would be expected if the left engine were to relight and accelerate to a high thrust setting, and delay the onset of a high rate of descent.

    RE: “3. If there was a brief restart, it is very likely that the attitude changed significantly, causing a high rate of descent to begin sooner.”

    As above, the high rate of descent needs to begin later than was observed in the simulator.

    RE: “4. Regardless of the fuel available in the left tank, there was more than enough fuel in the APU fuel line to power the APU for at least a few minutes after MEFE, even if there was a restart.”

    That point is probably worth further discussion. We don’t know the capacity of the APU fuel line, but I’ve been told the outside diameter of the line (not the shroud!) is ‘about one inch’, or 2.54 cm. As @David pointed out, the line is made of rubber and kevlar. We don’t know the internal diameter, but let’s say it’s 1.0 cm and the length of the fuel line is 100 ft or 3.05m. That would make the internal volume about 2400 cm^3, or 2.4 L. The SG of Jet-A1 fuel varies, but is typically around 0.8, so the line could hold about 1.9 kg of fuel.

    The APU fuel line will obviously be full when the APU is running. During a normal APU shutdown on the ground, the APU shutoff valve closes when the APU is selected OFF. The APU keeps running at 100% for 15 secs after the APU is selected OFF and at a reduced RPM for a further 80 seconds to cool down before it shuts down. During that period, the APU continues to draw fuel from the APU fuel supply line, but the shutoff valve at the tank end is closed.

    Using the ATSB’s figures, the APU fuel flow is about 1 kg/min (2 lb in 55 sec) when electrically-loaded; let’s say 0.75 kg/min at the reduced RPM. At that rate, the APU would consume about [(1 x 15/60) + (0.75 x 80/60)] = 1.25 kg of fuel from the APU fuel supply line during the shutdown. If the APU fuel shut off valve is closed, there would be about 0.65 kg of fuel remaining in the fuel line the next time the APU is started. The centrifugal boost pump in the APU fuel cluster can’t ‘suck’ that fuel out of the line to start the APU. The fuel line must first be filled with fuel drawn from the left main tank and supplied by the main tank boost pumps or the APU DC fuel pump.

    RE: “…the discussion about the 30 lbs of fuel in the tank and attitude are irrelevant.”

    I don’t think that’s correct. The APU will need to draw some fuel from the left main tank, as discussed above. Moreover, if we start to throw an engine restart into the mix as a way of explaining the high rate of descent, then the fuel in the left main tank (and its availability) becomes very relevant. That’s the point I think @Gysbreght was trying to make.

  93. Andrew says:

    @ALSM

    Oops, the length of the APU fuel line in point 4 of the above comment should be 30.5m, not 3.05 m!

  94. DrB says:

    @DennisW,

    I would expect numerous cases of noticeable BFO prediction errors per flight based on the ACARS positions and flight data using a temporal interpolation scheme (DSTG did not specify how they estimated the aircraft truth data for predicting BFOs). For instance, the MH370 flight plan has 10 intermediate waypoints. So there are typically course changes every half hour or so that cause steps in the BFO. These small steps could cause the higher errors present in the DSTG plot if DSTG Interpolated the ACARS data time sequences. It’s possible DSTG did something more complex, but I don’t recall any description being given in their book.

  95. Irthe turner says:

    It has always been my belief that ZS chose the end point for a deeper reason. The Zenith Plateau was formerly known as the Zenith Seamount. ZS. The zenith being opposite the Nadir. Nadir in Arabic has an emotional meaning: ‘ the lowest point in a person’s spirits or quality of an activity or professionan’. The man was cunning and it has crossed my mind that his simulator endeavors were created to mislead and were purposely deleted knowing it would be found and leading us to think the aircraft is further south.

  96. After flight paths were reconstructed using the BTO data, the predicted values of BFO were compared to the measured values to ensure that match was within an acceptable tolerance of 20 Hz.

  97. Gysbreght says:

    @Andrew: Thanks for your detailed reply.

    RE the 30 lb fuel quantity – clearly condititions other than 1 degree pitch need to be considered. Since that information is available for the fuel quantity indication system, not considering those conditions it is a strange omission by ATSB and Boeing.

    RE the N3 threshold – after loosing both engines due to fuel exhaustion, I would want the APU running for an emergency landing (just as Sully on the Hudson). I wouldn’t want to waste the few drops of fuel remaining on futile relight attempts. Perhaps the system designers thought of that?

  98. David says:

    @ Andrew. Some most useful work done on fuel availability and use at the end of flight thanks.

    The accelerations you deduced from the airline simulations, which included relights, show a maximum descent acceleration of 0.41g. I have derived an average of the acceleration over the 8 secs before, which comes to 0.29 g. This is less than half MH370’s average of 0.68 g. As you say the timing is awry also and even the maximum descent rate would fall short of the DSTG minimum.

    You may need to adjust your fuel consumptions to account for the APU maximum delivery rate of 3150 lb/hr (1430 kg/hr). That might not be the actual maximum since the engine might attempt to draw more, decreasing delivery pressure so increasing flow rate. Even so it would not reach flows like the 4760 kg/hr of your case 1 or the 3000 kg/hr of case 2.

    Also, an adjustment in maximum fuel flow might extend relights while reducing thrust.

    If it could not support the intermediate level thrust you mention, the possibility of a relight and immediate flameout, “after reaching the intermediate thrust setting” which might bring the right timing at least (your last paragraph), would not arise.

    It is possible that the engine fuel draw could reduce the pump delivery pressure below ambient, which could constrain APU engine supply. I have written already about my serious doubts the APU pump could draw fuel from the line at the low ambient pressure at altitude without vapour lock. There is a chance that would be even more likely despite this being nominally a pressurised supply.

    One other point about fuel flow is whether on APU DC pump start-up the flow rate to the engine will be high. The scene would be that the left engine had run out of fuel, meaning that its fuel intake and engine manifolds would be dry. The pump probably will deliver at its full 1430 kg/hr rate until these are refilled and that might take a few seconds and an increased quantity.

    On the possibility that there will be a delay between fuel arrival to the igniters and relight, if there is to be a successful relight I would expect it to be at first try. In a conventional start fuel is delivered when ignition conditions are rising, including air pressure, temperature and flow rate. In a relight the conditions are deteriorating, with engine speed and also temperature dropping but also the ignition-assisting hot combustion chamber would be cooling. I think that would be the reason why the RR engine would be shut down automatically at 35% N3.

    A final point is about idle fuel flows at sea level which I understand you to say is a maximum of 850 lb/hr. For your interest my figure of 1080 kg/hr came from the “Emissions Databank” download , line 567, column CF.
    https://www.easa.europa.eu/easa-and-you/environment/icao-aircraft-engine-emissions-databank#group-easa-downloads

    My conclusion is that the airline simulator did not meet with the BFO related descent characteristics and so the Boeing simulations are even less likely to have done so. If not, there should be a lot more work done of this topic, including in my view fuel access by the APU.

    That said, data on those might be persuasive if disclosed. In the meantime the ATSB/SSWG conclusion that the Boeing simulations indicate that a MH370 descent to match the BFOs was likely should not be the basis for decisions on future searching.

    Thank you for a comprehensive and clear paper on the topic.

  99. Victor Iannello says:

    @Andrew: Thank you for composing this impressive paper. I am still digesting the implications on end-of-flight scenarios. Unfortunately, I suspect that we will never understand the system interactions of the 777 well enough to declare that our interpretation of the BFO values and the associated 0.67g downward acceleration is wrong. So then the question is whether it’s possible to reach this increasingly steep descent with no pilot inputs and yet impact the ocean some tens of miles further. The answer to that question has a large bearing on where to search next.

  100. DennisW says:

    @DrB

    I would expect numerous cases of noticeable BFO prediction errors per flight based on the ACARS positions and flight data using a temporal interpolation scheme (DSTG did not specify how they estimated the aircraft truth data for predicting BFOs).

    Figure 5.4 does not support that point of view. I suspect Figure 5.4 is representative (but simply a “worst case”) of all the flights.

  101. DennisW says:

    @Victor

    The answer to that question has a large bearing on where to search next.

    Yes, it certainly does. Our analytics to date have been characterized by an over-estimation of the accuracy (and in some cases the meaning) of the data available to us.

  102. Niels says:

    There is something I do not understand (probably due to my lack of knowledge about aircraft control systems and flight dynamics):
    The 00:19:27 BTO has exactly the value one would expect based on the previous four (extrapolation of a third order polynomial fit). The simplest interpretation seems to be that the aircraft maintained speed, track and altitude most of time between 00:11 and 00:19. How can that be reconciled with the typical (“uncontrolled”) fuel exhaustion scenarios, where there would be several minutes flight without autopilot control ?

  103. Paul Smithson says:

    That’s a good question @Niels. I guess some might suggest that the aircraft was slowing down and losing altitude after ~0012. So what it lost in horizontal distance was made up for by change in altitude. In any event, the error margins on the BTO are too large for us to conclude that there wasn’t a deceleration over the final minutes if you assume straight line travel.

    However, I think consensus has it that the steep descent requires advanced development of banked turn. So I the default expectation for final BTO ought to be substantially lower than extrapolation from the previous four because you might easily have spent 1.5 of the 8.5 minutes doing a 180 turn. I can’t say how much lower because I haven’t done the sums. Perhaps @Gysbreght can offer us a scenario of duration of banked turn and AOB to culminate in the 0.67G and final ROD. Then we could see whether the BTOs are compatible with that.

  104. DennisW says:

    @Niels

    Section D. of the Holland paper spends a bit of time relative to your question.

    D. BFO Trend During the MH370 Flight

  105. TBill says:

    @Victor
    Your path proves several points in favor of searching north to 20S. The heading of the COCOS airstrip is a valid point, and an excellent contribution. Also you have demonstrated a heading change (dogleg left) to approx. 150S can remediate BFOs. I seem to recall (but I do not have reference) Australia sent a search vessel to COCOS or Xmas almost immediately after the loss of MH370.

    >>I offer an alternate path to 22S sans loiter: go 180S at ISBIX, and then make the heading change to 150S just before Arc3 to head towards BR. Phone call comes in at 23:14, time for another misdirection, heading change to approx 85-90 deg East to Zenith.

    @Irthe Turner
    “It has always been my belief that ZS chose the end point for a deeper reason. The Zenith Plateau was formerly known as the Zenith Seamount. ZS…”

    Excellent observation. I would give that some possible merit. 3 points for Zenith.

  106. Gysbreght says:

    @Niels: “The 00:19:27 BTO has exactly the value one would expect based on the previous four (extrapolation of a third order polynomial fit).”
    @Paul Smithson:

    Yes, that is an interesting observation. All scenarios involve at least one engine failure, and possibly two 15 minutes apart. If the airplane was flying on autopilot the engine failure(s) would result in a lower average speed between arcs 6 and 7 compared to the previous intervals.

    Simulations of end-of-flight involving loss of AC power at the second flame-out invariably show the airplane turning left at low bank angle after the loss of the autopilot. That turn reduces the distance between the arc crossings (the track being closer to perpendicular to the arcs). Apparently the reduction of track distance approximately compensates the reduction of groundspeed.

    In the simulations Boeing conducted in 2016, the track angle reduced by typically 40 – 50 degrees in the two minutes between loss of A/P and 7th arc.

    Your observation would exclude the Boeing simulations that involved loss of AC power at the first engine flame-out, since all involved a turn initially to the right.

  107. Victor Iannello says:

    @TBill: Thank you, Bill. A scenario that includes a diversion with the intent to land on Cocos Island but not carried through answers some questions, but also raises some new ones. At this point, I remain open about where to search next.

  108. Gysbreght says:

    @Nieis: On reflection, I should have said that I was thinking of typical cruise altitudes. If altitude and initial speed are low enough, the airplane after an engine failure would be able to increase thrust of the remaining engine to maintain the initial speed.

  109. DennisW says:

    @Victor

    An older version involving the Cocos by Mikhail P. The same author as the wave height paper indicating a splash down near Christmas Island.

    https://sites.google.com/site/mh370tibet/updates/31may2014

  110. ST says:

    @Victor – Thank you for this interesting and thought provoking analysis similar to the previous blogs.The end point of an Australian territory seems very likely from a motive perspective.
    The simulator according to several articles was used to land at airports in remote islands and some strips in Srilanka, Andaman etc. Further Cocos island seems to be one without any active Military or defense personnel on the island according to Wikipedia.
    Also wondering if the likely incapacitation that prevented a landing could be from an earlier de-pressurization or above envelope flight altitude as discussed in your previous posting regarding the civilian radar data analysis.

  111. Victor Iannello says:

    @ST: I don’t know what to make of early reports like this one that states the captain had plotted a course to a remote island in the SIO. If the basis of the story is the simulator data that we now have, then the story is false. Perhaps there is other data that we don’t have. Without more evidence, it’s hard to accept the story on its face value.

  112. Victor Iannello says:

    @DennisW: Thanks, Dennis. That path ends a bit further north at 17.5S. The changes in speed are not explained.

  113. DennisW says:

    @Victor

    Yes, your version is much more likely, IMO. The PIC would have known he did not have enough fuel to reach Learmouth. There is no sensible way to explain the Learmouth turn at the Cocos. A continuation from the Cocos on a constant magnetic heading is much more defensible.

  114. David says:

    @Paul Smithson. “However, I think consensus has it that the steep descent requires advanced development of banked turn.”

    I think that was the consensus but for my part anyway no longer is. What Peter Foley made clear recently was that the ATSB conclusion from the Boeing simulations was that the BFOs resulted from a less stable phugoid. That is different. It dispenses with the need for accompanying bank and tight turns.

    The descent was caused by bank, that caused by trim he says. As in the level D simulator descents that bank might indeed straighten and there be a less stable phugoid without bank extremes and high turn rates.

    At play is whether trim alone could cause that since the Boeing simulations did not include relight. The level D simulations do not display less stable phugoids of the extent or timing needed, even with relight thrown in.

    There is also the issue which Gysbreght alludes to as to which way the bank was. The level D’s were to the right, some of Boeings to the left. Left engine relights obviously will reinforce the rights and oppose the lefts.

    What the trim was exactly is important but tends to be led by outcome. If the more the trim the more extreme the phugoid resulting then it ‘follows’ that there must have been more trim. That is turning assessment into a circularity.

  115. David says:

    @Andrew. I didn’t draw out engine suction enough. If it exceeds the APU DC pump flow rate and in the extreme could contribute to the risk of APU vapour lock, the same can be said of the engine, which might draw an embolism and flame out again. The level D simulations emulate this. Commonly the relight is brief and it is followed by a higher EGT than cold. Noticeable have been flickers of relights following also.

    We have no idea how the simulator might assume there to be fuel, or how much, particularly with the aircraft nose down and not all the 30 lbs of the Boeing assessment being available. Indeed the basis for that supply must be weak or Boeing would have utilised a simulator programmed like that.

    Clearly though in those level D simulations the relight is being starved of fuel when its demand rises. That does look like the embolism to me. If so, the APU supply is under that added risk also.

  116. Niels says:

    @Paul Smithson
    Agreed that both possible BTO error and the option to “trade” horizontal distance for altitude gives some space for manoeuvring in the final minutes.

    @Gysbreght
    I had to think a while about: “That turn reduces the distance between the arc crossings (the track being closer to perpendicular to the arcs). Apparently the reduction of track distance approximately compensates the reduction of groundspeed”. I think I get it. That could indeed work, especially for the near 180 degrees tracks in the lower to mid-thirties.

    You raise the point that on one engine lower and slower than typical cruise conditions should be assumed. Can you give an indication by how much?

    @DennisW
    For BTO trend discussion the Holland paper refers to the DSTG book.I haven’t found the particular section yet.

  117. airlandseaman says:

    David: Foley did not claim the high rate of descent WAS due to a phugoid. He simply noted that a phugoid could be in the mix.

    BTW…the notion that spiral descents and phugoids are mutually exclusive is false. In fact, most or all of the phogoids we observed took place after a turn started. You can see phugoids in a 45 degree bank.

  118. DennisW says:

    @Niels

    I thought section D was spot on your question. I must have misunderstood.

  119. Gysbreght says:

    @Niels: RE YR question: “Can you give an indication by how much?”

    The difficulty is that in normal cruise the thrust is limited to the CLB rating, and FCOM ENGINE INOP data are for Max Continuous thrust. Changing the thrust rating limit from CLB to CON requires a pilot action.

    According to the FCOM the ENGINE INOP Driftdown Speed/Level Off Altitude at Max Continuous Thrust and 174000 kg is 219 kt IAS/29500 ft at ISA+10C&Below, and 27200 ft at ISA+20C. I would guess that at some altitude below 27000 ft the airplane might achieve that speed at CLB thrust and ISA+10C&Below.

  120. TBill says:

    @Niels
    Brian Anderson has pointed out that for straight (passive) flight paths (south) that Arc7 is quite far from Arc6 so that makes it relatively hard to get to Arc7 as that implies MH370 had to keep the speed fairly high at higher altitude. Since I have historically considered active pilot, it is not so hard for me as I usually turning toward the east and slowing down and descending.

    Not to say I am convinced passive is wrong, I am quite open at the moment.

  121. Gysbreght says:

    @ALSM: The ATSB observed rates of descent consistent with the final BFO values in 5 of the 10 simulations that Boeing conducted in 2016. In four cases (#7, 8, 9 and 10) that occurred in spiral dives when AC power was lost at the first engine flame-out. In the remaining case (#3) there was no large bank angle but the late part of the trajectory showed distinct signs of an extreme phugoid, that were not present in any other of the ten simulations. All uncontrolled descents will show some phugoid motion, but not with the amplitude indicated by the final BFOs.

  122. David says:

    @ALSM. You said, “Foley did not claim the high rate of descent WAS due to a phugoid. He simply noted that a phugoid could be in the mix.”

    That’s not my interpretation.

    Here are some quotes:
    • First the initiation: “After the first engine flamed out you get the rudder correction, the thrust asymmetry compensation. When the second engine flames out, it spools down, the rudder comes back and just a little rudder stayed on. That’s what established that left hand turn.”
    • Then the phugoid’s place: “The Boeing simulations also showed that the behaviour of the aircraft was pretty stable. It was likely to be in what we call a phugoid motion. This is where the aircraft pitches over, picks up speed and picks up lift and starts to climb a little on the way down. The analysis that was latterly done by the Defence Science and Technology Group on the last couple of communications, to try to ascertain or bound the rates of descent, showed us that the aircraft was in a high and increasing rate of descent at some point in the context of that phugoid.”
    • Again: “The phugoids that Boeing looked at in the final series of simulations, the advice from Boeing is that the rates of descent determined by DSTG were consistent with a phugoid that was less stable.”
    • Again: “When those two final transmissions occurred the aircraft was in a high and increasing rate of descent, and likely to be in a phugoid.”

    I add as relevant to another part of this discussion:
    “What we’ve never been able to simulate is the effect of one or both the engines spooling up again—which, of course, they’re trying to do; once they flame out they’re continually trying to restart. It can’t be simulated because once the simulator sees zero fuel it doesn’t allow the engine to start.”

    Also you said, “BTW…the notion that spiral descents and phugoids are mutually exclusive is false.” I did not say that, if that is what you imply.

  123. David says:

    @ALSM. Of further interest in the above is the remark in the second last paragraph,”What we’ve never been able to simulate is the effect of one or both the engines spooling up again—which, of course, they’re trying to do; once they flame out they’re continually trying to restart”.

    Two things:
    One, the mention of a right engine attempted relight: I saw none in your simulations and was watching for it. Had there been I would have concluded that in those the fuel was not coming from the APU DC pump but from the fuel by-pass suctions not labled in @Andrew’s fig 1 but at the arrow top left, the inwards check valve. In both tanks residual fuel could have sloshed forward and covered those. If Foley reckons on a right engine relight, I think he is straying beyond the assumption it was fed by fuel from the APU DC pump, the bedrock of the ATSB’s case.

    Two, “continually trying to restart” is what was observed of the left engine in your simulations. Again, that indicates the software is not saying the restarts fail from residual fuel exhaustion. I see fuel gaps.

    It is interesting to see how both EGT and higher-than-windmilling revs remain on the left engine post the brief relights.

  124. David says:

    Second paragraph amendment please. In replace, “….beyond the assumption it was fed….”,
    with, “……beyond the assumption that a left engine relight would be fed like the APU (from the APU DC pump…..).”

  125. mash says:

    Theorem: Capt Zaharie innocent
    Method: Proof by Zigzagation
    Proof: If Capt Zaharie were a Rouge Pilot+, he would choose a Direct flight.
    But this is a Zigzag flight.

    glossary:
    Proof by Zigzagation – Proof by Contradiction + Principle of Least Action
    Rouge Pilot Plus – Rouge Pilot + a destination
    Zigzag – the ugly zig from KL and zag back to Peneng
    Direct – west bound MH flight …

  126. airlandseaman says:

    David: Frankly, I’m not clear on what you are trying to claim/prove. Your APU fuel analysis is more speculative than factual IMO. I believe the ATSB/Boeing analysis (and my own) which show that there was enough fuel in the line (and the APU had means to draw on that fuel) to explain the 00:19 events.

    I met with Foley 3 times in the last 6 months. We have discussed the EOF scenarios in some detail. I won’t try to recall specific quotes, but I am pretty sure your all too narrow interpretation of Foley’s Senate statements do not accurately reflect his braod viewer views.

    Sure, phugoids were likely. I have been saying that since Nov 2, 2014. But they were not likely to be the sole cause of the high descent rates observed. The BFO values may have been sampled during a phugoid event, or during a steep banked turn, but that is not particularly important to the overall descent path. The increasing turn rate probably had more to do with the POI.

    Note also, all of the publicly available Boeing descent tracks (Nov 2016) involved a turning descent path, typically in tight turns (thus high bank angles). There was not a single straight path with phugoids.

    I sat in the LevelD simulator for 4 hours in 2014. I know what I observed. It was not consistent with what you are claiming. Regarding engine restarts…I observed several attempts, on several sim’s, and one successful attempt that lasted about a second. I observed phugoids at various bank angles. Sometimes they came and went…not a continuous phenomenon. I observed bank angles in excess of 90 degrees. I observed the airspeed indicator pegged at 500 kts IAS. I observed impacts at very high speeds. My overall assessment was that it was very likely that MH370 crashed close to the 7th arc.

  127. Andrew says:

    @David

    To be clear, my paper was intended to analyse what happened in the simulator and to determine if the estimated residual fuel would actually support an engine restart or APU autostart. I fully accept that cavitation of the engine fuel pump might prevent the engine accelerating to a high RPM if it did relight. The APU I’m not so sure about, given the much lower fuel demand. I still believe that power restoration by the APU is the most likely cause of the final SATCOM transmissions.

    I’m not sure what you’re seeing during the engine restarts in the simulator videos viz. ’embolisms’ and ‘flickers of relights’. All I can see is an engine that briefly restarts and then flames out again, presumably due to fuel exhaustion. Frankly, I doubt that a simulator could emulate the scenario you described.

    RE: “We have no idea how the simulator might assume there to be fuel, or how much, particularly with the aircraft nose down and not all the 30 lbs of the Boeing assessment being available. Indeed the basis for that supply must be weak or Boeing would have utilised a simulator programmed like that.”

    I agree that we don’t know how the simulator modelled the residual fuel, as I stated in the paper. I doubt that any simulator could accurately model the movement of fuel within the tank. Note the aircraft only pitched down in one of the two simulator trials where the autopilot disengaged, when both engines flamed out at the same time. In the trial where the right engine failed first, the pitch attitude was about 5° nose up initially and was still at about 4° nose up some 90 seconds later after the left engine failed. The attitude remained above the horizon until after the engine restarted and again failed. I think it’s possible the engine restarted if the nose remained above the horizon during the relight. The maximum thrust it might have developed and its effect on the flight path is questionable.

  128. Andrew says:

    @Victor

    Thank you for your comments. Unfortunately, that’s a question I can’t answer.

  129. DrB says:

    @Gysbreght & Niels,

    My 181.2 degrees mag track route also continuously turns to the left. It fits the 00:19 BTOs using the expected airspeed decelerations at engine flame-outs, without any additional left turn due to the aircraft banking left after MEFE. I’m not saying (yet?) that a left bank did not occur, but I do think now the aircraft crashed more than 22 NM from the 7th Arc. The question is, how could that happen, with or without an active pilot?

  130. DrB says:

    @All,

    Does anyone know the waypoints used by MAS for their Mumbai to Kuala Lumpur flight?

    That flight on 2 March 2014 was used in Bayesian Methods in Figure 5.4 to illustrate the worst case of unexplained BFO errors as calculated by DSTG. I want to see if there is any apparent correlation of DSTG BFO error with maneuvers along the route.

  131. DrB says:

    @All,

    Consider this scenario which has the aircraft continuing to fly on one engine after 00:19. It has one big problem, but I was trying to think outside the box, and I thought maybe this would trigger someone else to suggest modifications.

    17:22 L AC Bus is isolated (L IDG is off and L AC Bus is not tied to R AC Bus)
    18:24 L AC Bus is tied to R AC Bus; SDU boots up
    xx:xx IFE is turned off
    00:17 R engine flames out, causing loss of power to both L and R AC Buses; L engine is still running
    00:18 APU starts and powers L and R AC Buses
    00:19 SDU boots up and transmits LOR/LOA
    00:21 No IFE messages sent because IFE was turned off
    00:25 L engine flames out
    00:27? APU flames out

    This biggest problem here is the 00:19 BFOs, which in level flight would have to affected by a large negative warm-up error to match the data. I don’t know if that is possible, but this reboot is unique (being in flight and after a short power interruption), so maybe we shouldn’t assume we know what the warm-up errors would be for this case. Of course, if this were true, then you could no longer interpret the 00:19 BFOs as indicating a rapid descent, and the ATSB scenario would need to be re-examined.

    There may be other things I have got wrong. Let me know what they are. Could be just another dead end.

  132. ST says:

    @Victor – Thanks for your earlier note. It definitely is hard to isolate false news and data vs accurate reporting – particularly in the context of this search. Some of the reports in the last few days make you wonder where the ethics of journalism is at particularly considering the sensitivity of families and folks that are impacted.

  133. TBill says:

    @Victor
    If the 38S route makes sense, alternatively MH370 could have changed heading for Dordrecht Hole after 22:41, and probably could have made it given the mighty tail winds.

    That route, had it been completed, would have given us a different (further away) Arc6 and Arc7. Instead (perhaps due to active pilot reaction to 23:14 the sat call) MH370 wound up at a spot that gives us 38S as a mirror image solution…a little like @Ventus45 saying there is a mirror image path solution at Penang.

    More later…

  134. David says:

    @ALSM.”I believe the ATSB/Boeing analysis (and my own) which show that there was enough fuel in the line (and the APU had means to draw on that fuel) to explain the 00:19 events.”

    As you might have read I agree it is likely there would be enough fuel in the line if it was of 20mm id as Andrew estimated. If in fact it is 10mm id as he has said above that drops the fuel in there by three quarters, which makes a difference particularly if, as I researched and posted, the APU fuel consumption is higher than the ATSB expects. This was not speculative but supported by APU test bed and other fuel consumption data, all of which suggests that it would be a good deal higher. Yes that ‘suggests’ that the ATSB estimate is understated and I do not claim that to be fact since I do not know how the ATSB came to its conclusion. For example, in the aircraft there are no fuel meters on the APU or its fuel line. Besides, the consumption also depends on the electrical loading. The ATSB has not said what loading they assumed.

    Besides that I have done an analysis, also posted, which indicates it is quite possible that fuel in the line would not be available in sufficient quantity, even at 20mm id due to the low ambient pressure at altitude. The APU would not suck it if unpresurised. You keep asking questions about what I have addressed already, needing unnecessary and voluminous replies like this.

    “Sure, phugoids were likely. I have been saying that since Nov 2, 2014. But they were not likely to be the sole cause of the high descent rates observed.” and, “There was not a single straight path with phugoids.”

    You seem to have the idea that I reckon the phugoids must be in unbanked flight. What I have said is not that there could not have been steep banks. What I have said is not that the phugoids must have been without bank. What I have asserted above is that the less stable phugoids need not be ACCOMPANIED by STEEP banks. They might be PRECEDED and FOLLOWED by them. By steep I mean over 45 deg. I had provided amplification in my next paragraph.
    It is common sense anyway that a very sideways phugoid will yield a lesser climb and descent and in the major phugoids I have seen in your simulations there was some levelling from a steep bank before.

    You say, “Frankly, I’m not clear on what you are trying to claim/prove.” Without disclosure of data verifying the ATSB deductions from the Boeing simulation they should not be treated as gospel. I think I have said as much several times.

    Restrict your questions to points I have not addressed, please.

  135. Andrew says:

    @DrB

    Re you scenario: The aircraft wouldn’t be in level flight at 00:19 when the SDU transmitted the LOR/LOA because the autopilot would have disengaged at 00:17 when the right engine failed and AC power was lost. The left engine would continue running at its previous thrust setting and with no TAC available the thrust asymmetry would cause the aircraft to yaw and roll to the right in a descending spiral.

  136. David says:

    @Andrew. Thanks for your comments.

    “I fully accept that cavitation of the engine fuel pump might prevent the engine accelerating to a high RPM if it did relight.”

    It was not cavitation of the APU DC fuel pump I was addressing but its capacity, a small part of what your cases required.

    On the effect of engine suction on the APU fuel supply you said,”The APU I’m not so sure about, given the much lower fuel demand.”

    If the engine were demanding enough it might reduce the pressure in the fuel inlet manifold by suction. That might speed up the APU DC pump, being DC, fuel flow being increase some. That drop in pressure could affect the pressurisation in the APU fuel line, connected to the same pump. Earlier, with some analysis, I have alluded to vulnerability of the APU pump when trying to draw fuel from the fuel tank with the line unpressurised, which it would be after the tank runs dry.

    Expanding further still, there should be no problem with this after the left engine shut down from lack of relight before residual fuel (ie the nominal 30 lbs in the tank) exhaustion, that is after fully pressurised supply was restored to the APU. My speculation (yes) was and is that before that, during a relight at the maximum flow the engine could draw, the pressure at the APU DC pump outlet might drop beneath ambient air pressure due to engine suction, in which case the supply to the APU from its line would be vulnerable once more. I hope that is clearer.

    “I’m not sure what you’re seeing during the engine restarts in the simulator videos viz. ’embolisms’ and ‘flickers of relights’. All I can see is an engine that briefly restarts and then flames out again, presumably due to fuel exhaustion.”

    The point I was making is that is there are periods in which the left engine clearly still has fuel after an obvious fuel cut, that leading to a truncated relight. The residual fuel then obviously has not been exhausted. This is what I have observed in those simulations.

    “Frankly, I doubt that a simulator could emulate the scenario you described.”

    I am unsure what that situation is. If “emulate” and “embolism” are clues I think I have covered that in the above.

    Even so as I said earlier, “Commonly the relight is brief and it is followed by a higher EGT than cold. Noticeable have been flickers of relights following also.” That means there was fuel still, not exhausted.

    Also I have noted that aside from EGT, engine speed does not drop away to the windmilling speed (eg of the right engine) after those brief relights.

    “I doubt that any simulator could accurately model the movement of fuel within the tank”.

    That is why I have suggested that Boeing did not do this. Yes I agree that pitch up would most likely make all the fuel available.

    Of a restart above the horizon you note that, “The maximum thrust it might have developed and its effect on the flight path is questionable.”

    This was, and remains, the problem. It is relevant if trim would not initiate the descent; and for that matter even if it could: ie the effect then of a relight.

    Separately, you have said to ALSM, “During a normal APU shutdown on the ground, the APU shutoff valve closes when the APU is selected OFF. The APU keeps running at 100% for 15 secs after the APU is selected OFF and at a reduced RPM for a further 80 seconds to cool down before it shuts down. During that period, the APU continues to draw fuel from the APU fuel supply line, but the shutoff valve at the tank end is closed.”

    I do not see how that could happen, drawing fuel from a line blocked at the other end. Do you have a reference please?
    The AMM at 49-00-00 page 23 shows the fuel solenoid valve and the metering valves, both at the APU, as closing at 70% RPM after shut down on the ground. The APU fuel shut off valve, at the tank, closes later, at 15%. There should be no fuel consumption between.

    From other reading the fuel shut off valve is operated by the ELMS. The ELMS is informed by the APUC whether the engine is running. Hence I assume that shut down at 15% is initiated by the APUC.

    Finally, clearly I have not explained myself too well in recent times. To reduce the traffic of my explanations I aim to expiate that by keeping my responses to one word plus one sentence for the next few days, or to none.

  137. irthe turner says:

    @TBill, It’s all conjecture of course 🙂 None the less, IMHO, ZS planned this well in advance taking many aspects into account, including his simulator data. The aircraft was his coffin, the SIO his grave. He would not leave the terminus to fate. But this little titbit isn’t helpful in locating M9-MRO, I understand that.

  138. David says:

    @Andrew. Youu to Gysbreght, “The manuals don’t state what happens to the DC pump and isolation valve if AC power is subsequently restored by an engine restart (however brief) or APU autostart. In short: I don’t know!”

    A help I hope; AMM 49-30-00 page 13, “When AC power becomes available, the left forward boost pump turns on and the dc pump turns off.”

  139. Andrew says:

    @David

    RE: “I do not see how that could happen, drawing fuel from a line blocked at the other end. Do you have a reference please?”

    That was an error on my part; thank you for pointing it out. The APU bleed shutoff valve closes when the APU is selected OFF, to start unloading the APU. The APU fuel shut off valve closes at 15% RPM, as you noted.

  140. Don Thompson says:

    DrB

    You wrote:


    17:22 L AC Bus is isolated (L IDG is off and L AC Bus is not tied to R AC Bus)
    18:24 L AC Bus is tied to R AC Bus; SDU boots up
    xx:xx IFE is turned off
    00:17 R engine flames out, causing loss of power to both L and R AC Buses; L engine is still running
    00:18 APU starts and powers L and R AC Buses

    You state, at 00:17, the “L engine is still running“.

    The L engine Backup Generator will continue to supply power to the Transfer Busses via the Backup Generator Converter, therefore, the APU will not start until the second flame out occurs.

    Your scenario requires that, at least, the L BACKUP GEN switch be unset.

    Continuing the discussion of Inmarsat and its responsibilities for the Classic Aero network, you wrote “A more important consideration is that Inmarsat collects and processes years worth of data from thousands of aircraft“. That is true but the data records only the performance of the network, not why certain events occur. Consider the anomaly exemplified by the delayed transmission of a subset of LOA bursts, that is a characteristic of Racal’s SDU design. While we have derived a solution to correct the anomaly, the cause has not been identified, and that bothers me.

  141. Gysbreght says:

    @Andrew says June 11, 2018 at 12:47 am to @DrB

    ” Re you scenario: (…) The left engine would continue running at its previous thrust setting and with no TAC available the thrust asymmetry would cause the aircraft to yaw and roll to the right in a descending spiral. “

    Sorry, that is not what the Boeing simulations show. The simulations conducted in 2016 included four simulations “In an electrical configuration where the loss of engine power from one engine resulted in the loss of autopilot (AP), …”, i.e. essentially DrB’s scenario. The trajectories are shown in Figure 6 of ATSB’s November 2, 2016 report. If numbered from left to right at the top of the chart, trajectories 7, 8, 9 and 10 are in the abnormal electrical configuration. These trajectories show that the airplane is essentially in trim for the one-engine-inoperative condition when the AP is lost.

    Apparently the TAC sets the rudder trim for the thrust asymmetry during the rundown of the failed engine before AC power is lost. That trim remains when TAC is lost due to loss of AC and PFCS reversion to secondary mode. However, the increase of thrust asymmetry as the airplane descends to lower altitudes is not compensated, and the airplane is severely out-of-trim after the second engine failure.

  142. Andrew says:

    @Don Thompson

    Good point – I had assumed the backup gen was off, but clearly it’s not in the scenario DrB presented.

  143. Andrew says:

    @Gysbreght

    RE: “Apparently the TAC sets the rudder trim for the thrust asymmetry during the rundown of the failed engine before AC power is lost. That trim remains when TAC is lost due to loss of AC and PFCS reversion to secondary mode. “

    Actually, DrB’s scenario wouldn’t result in the loss of TAC & AP after the first engine failure because the left backup generator would continue to supply the transfer buses, as Don pointed out. The transfer buses would only lose power after the first engine failure if the left backup generator was selected OFF.

    I haven’t analysed the Boeing simulations as you have, but in all the airline B777 simulators I’ve used, the TAC input is removed if the PFCS reverts to secondary mode. The same thing was observed in the simulations that Mike observed in the UA simulator.

  144. Gysbreght says:

    @Andrew: “… in all the airline B777 simulators I’ve used, the TAC input is removed if the PFCS reverts to secondary mode”

    I suspect that what you are witnessing is the TAC reducing the trim to zero as the thrust asymmetry reduces to zero, before the PFCS reverts to secondary mode. Removing the TAC input at power failure would violate fail-safe design principles, considering that the main ‘raison d’être’ of the TAC is engine failure near V1 on take-off.

  145. Gysbreght says:

    In “TAC reducing the trim to zero” please replace “trim” by “TAC input”.

  146. Gysbreght says:

    @Andrew: RE “I haven’t analysed the Boeing simulations as you have, …”

    Please look at track #8 (the dark blue line in Figure 6). AC power is lost at the point where the track starts to deviate from the AP-controlled straight track, about halfway between the top of the chart and the 7th arc point. The airplane then initially turns slightly to the right, then slighly to the left, before settling into a wide left turn approximately at the 7th arc.

  147. Victor Iannello says:

    @Andrew: There were four Boeing simulations in which the left generator and left backup generator were isolated so that the flight control degraded to SECONDARY after the right engine flamed out. For each of these, there was a turn to the right, which means the TAC was not fully compensating for the engine failure, if at all. The difference in the behavior between the four simulations could have been due to residual trim. I believe two of the simulations had greater left residual trim, which resulted in a gentler turn to the right after the right engine failure, and a steeper bank to the left after the left engine failed. In the other two, the bank to the right continued into a tight spiral.

  148. Don Thompson says:

    @Gysbreght wrote “Removing the TAC input at power failure would violate fail-safe design principles, considering that the main ‘raison d’être’ of the TAC is engine failure near V1 on take-off.

    ‘Normal’ operation of the PFCs, where TAC is computed, requires power to be available on the Transfer Busses.

    Engine failure on take-off (or at any other phase of a flight) removes only 2 of 4 possible sources of power to the Transfer Busses, i.e. the IDG and Backup Generator of the failed engine. The functioning engine will continue to supply power from its IDG to both Main AC busses via the bus tie function while its Backup Generator and the Backup Generator Converter are available to take up supply to either or both Transfer Busses. Fail-safe operation is not compromised by loss of one engine.

  149. DrB says:

    @Andrew,
    @Don Thompson,

    Thank you for your astute comments on my proposed end-of-flight scenario.

    Consider the following modifications:

    1. When the L IDG is shut off, the L backup generator is also shut off.
    2. After R engine flame-out, the pilot manually descends rapidly shortly thereafter at 00:19 to a lower altitude so as to create the BFOs observed then. This occurs with the L engine still thrusting, providing another 6 minutes thereabouts for flying well away from the 7th Arc at a lower altitude before L engine flame-out.

    Does this modified scenario match all the observables?

  150. Victor Iannello says:

    @DrB: First, I’d say if you allow pilot inputs, the final BFOs and an impact far from the 7th arc are both definitely achievable and that scenario will be impossible to disprove. You’ve introduced one of many pilot input scenarios. For that manner, the left bus might have been deliberately power cycled, causing the reboot of the SDU. But why would the pilot in an EO configuration choose a 0.7g downward acceleration and reach a 15,000 fpm descent?

  151. Gysbreght says:

    @Don Thompson: The loss of one engine obviously does not result in the loss of the TAC function, so the question of whether the trim input required to compensate the thrust asymmetry should be removed or retained doesn’t arise.

    The considerations of the system designer are not limited to single failures like the loss of one engine. He must consider all combinations of multiple failures that can not be shown to be extremely improbable (10^-9 per flight hour). So consider a flight condition where there is a large thrust asymmetry, compensated by the TAC rudder trim input, and then a complex combination of multiple failures not only causes the loss of the TAC function but also a high workload for the crew. Should the TAC input be removed or is it safer to retain it, leaving to the crew to deal with it when done with more urgent tasks?

  152. TBill says:

    @DrB
    “Does this modified scenario match all the observables?”

    Without commenting on the equipment details, I am tentatively thinking yes, perfect BFO match after 22:41 (to match the DSTG BFO straight line) probably requires descent, probably as far back as 23:14…I am thinking MH370 is perhaps FL150 to FL200 when the fuel is gone.

    This is work in progress, but what where I am heading “in search of a perfect BFO match.”

  153. @Victor
    « Your new trajectory passing over Cocos Island »: A piloted trajectory up to 18h40 (and for you up to VOCX) appears now for granted 🙂

    Analyzing your trajectory from 18h25 (Arc1) to 19h41 (Arc2) it seems we are missing some information. May I ask you to complete the nice table you provided from 19h41 and after with Arc1 data also?

    Could you explain what is the trajectory of the flight and its speed between Arc1 and Arc2? We could not figure it out from your data.

    Thanks.
    Sorry for being late, I was not in the last two weeks.

  154. Gysbreght says:

    @Victor Iannello: You wrote to Andrew:

    “There were four Boeing simulations in which the left generator and left backup generator were isolated so that the flight control degraded to SECONDARY after the right engine flamed out. “

    Are you familiar with the scenarios and conditions that the ATSB has selected for those simulations?

  155. Victor Iannello says:

    @Gysbreght: No, I don’t know the scenario and conditions for the four simulations in which the left generator and left backup generator were isolated other than that. But based on the shape of the paths, as I said, I do believe that two of the simulations had greater left residual trim, which caused the change in bank from right to left after the left engine flame out.

  156. Gysbreght says:

    @Victor Iannello: Thank you for your reply.

    The three questions I’m interested in regarding those simulations are:

    (a) Was the right engine always the one that flamed-out first in all four simulations?
    (b) Did the remaining engine flame-out before the airplane crashed in any or all of those simulations?
    (c) If the answer to the previous question is yes, was that before or after the high rates of descent were observed?

    My interpretation of the shape of the paths differs from yours. I’m not sure what you mean by “residual trim”. Is that applied before the simulation starts?

  157. Victor Iannello says:

    @Jean-Luc Marchand: There are many possibilities for the path before 19:41. As such, I didn’t propose one, nor do I think it really matters. For instance, if we ignore the Lido Hotel radar image (which I have doubts represents MH370), and accept the radar position at 18:02, then there are paths over the Malacca Strait that satisfy the BTO and BFO data without the need for the lateral offset, including a straight path to Car Nicobar, as I described in a previous article. Or, the Lido Hotel image was valid, and there is the lateral offset that I depicted in the path shown above. If the aircraft was on a northwest trajectory at 18:40, then it was descending, as I described in another previous article. The paths towards Car Nicobar also require a loiter in that vicinity, as I’ve described in other places.

    I’m not sure it’s productive to choose among several possibilities for which there is little evidence and might not make a difference in determining the impact point.

  158. Victor Iannello says:

    @Gysbreght:
    a) Yes.
    b) I don’t know, although I believe the change in direction of the turn from right to left indicates the left engine flamed out.

    By residual trim, I mean the rudder trim that is applied in excess (or in deficit) of that required to cancel the yaw imbalance due to asymmetry. If the simulator has no provision to introduce yaw imbalance, than the residual trim is the rudder trim.

  159. @David

    Hi David, thanks for your reply re my thoughts that the Rolls Royce emblem may have ripped off the vortex generator fin. I enjoyed reading your breakup sequence, and you impressed on me its enormous complexity.

    However, I think the RR emblem would get the first shot at taking off the fin. If the engines hit first, it would be the first know about it and be saying, “Oh my gawd!” before it was torn off by previously undisturbed water, travelling ballistically, and possibly following the barrel curvature of the engine via the Coanda effect.

    Regarding the found piece, you said, “What you see is part of the base to which the fin had been mounted.” Do you know if that includes the severance cut of the fin? If it does then hopefully someone can check for matching damage on the found RR emblem. It’s a long shot, of course, but after the search failures, do we have anything that is’nt?

  160. Kelly Perazzolo says:

    For those here working on the drift analysis, can this be factored in? It seems to have been forgotten about.

    https://www.telegraph.co.uk/news/worldnews/asia/malaysia/11460544/MH370-investigators-look-at-Malaysia-Airlines-towelette-washed-up-in-Australia.html

    I can understand how these are common and impossible to trace back to 9M-MRO however it’s worth a try.

  161. Brian Anderson says:

    I’m hoping that at least some of Chillit’s twitter followers also read Victor’s blog.

    From time to time I am tempted to sign up to Twitter so that I can comment there, and point out the fallacy of Chillit’s ramblings over the BTO data. However, I know that would be a waste of time because he just blocks people who seriously challenge his views.

    The fact is that Chillit continues to mis-represent the BTO data, and denigrates all who have properly preformed the calculations to determine the location of all the BTO “arcs”, and particularly the location of the “7th arc”.

    Inmarsat released ALL the BTO data years ago. Chillit uses only part of the data and then misunderstands how to analyse it anyhow. He seems to ignore the fact that the ACARS data out to about 17:07 UTC allows the so called “bias” term, actually a constant otherwise known as “K”, to be calibrated. He seems not to understand that the satellite ephemeris is well known and well documented, and that this is fundamentally important to take into account in the calculations. Does he even appreciate that the calculations allow a determination of the total transit delay for transmissions from the Perth GES, to the satellite, and then to the aircraft AES, and for the return journey on the same path. From this the known line-of sight distance from the Perth GES to the satellite can be subtracted, leaving just [twice] the line-of-sight distance from the satellite to the aircraft.

    Of course the line-of-sight distance then has to be translated into a measurement over the surface of the WGS84 model of the earth’s surface to determine the precise location of the arc.

    Understanding the known rounding of the BTO numbers, and the accuracy of other variables, the location of the arcs can be determined to within about +/- 6km.

    Chillit can’t do this, and his ramblings are a complete nonsense.

  162. Andrew says:

    @Gysbreght

    RE: “I suspect that what you are witnessing is the TAC reducing the trim to zero as the thrust asymmetry reduces to zero, before the PFCS reverts to secondary mode. Removing the TAC input at power failure would violate fail-safe design principles, considering that the main ‘raison d’être’ of the TAC is engine failure near V1 on take-off.”

    Fair enough, I chose a bad example. Nevertheless, the TAC input IS removed if the TAC fails. The primary flight control system reverts to secondary mode if it loses information from other aircraft systems. The TAC isn’t available in secondary mode because the primary flight computers (PFC) may lack the information they need to calculate the correct rudder input. Consequently, the PFCs remove the TAC input when the TAC fails in case its erroneous. The same thing happens during an engine failure where the engine is surging. The TAC can’t calculate the rudder input while the thrust is surging, so it fails and removes any prior input. Believe it or not, but pilots are still trained to use the rudder to keep the thing flying straight!

  163. DrB says:

    @Victor Iannello,

    You said: “But why would the pilot in an EO configuration choose a 0.7g downward acceleration and reach a 15,000 fpm descent?”

    I haven’t the faintest idea, just as I cannot fathom a pilot murdering a planeload of passengers while committing suicide during or after flying until fuel exhaustion.

    I’m still looking for a reason to power up the L AC bus circa 18:24. Any suggestions? Would that be of any benefit if the aircraft was being re-pressurized?

    On the other hand, we have no other examples of this type of log-on event during normal operation or as a test event. If the LOR/LOA transmissions somehow occurred PRIOR to the OCXO temperature set point being reached, then one would get BFOs with negative errors, and this would lessen the RODs (but maybe not the acceleration).

  164. Victor Iannello says:

    @DrB: My point is if you allow the possibility of pilot inputs at fuel exhaustion, and don’t care about motivation, there are multiple scenarios that produce the observed BFO values and an impact far from the 7th arc. I thought you were proposing a scenario for which you assigned a motivation.

    Power up of the left AC bus? The simplest (but not only) answer is some SATCOM functionality was desired, such as the ability to initiate or receive calls, or receive a signal in the form of a ring. The fact that ACARS was disabled prior to the SATCOM boot up is clear evidence that the pilot was aware that the SATCOM link would be restored.

  165. David says:

    @Neville Macaulife. “Do you know if that includes the severance cut of the fin?”
    Mutual witness marks? I think not. The fin extended the base outwards into the airflow. Its outside extended over the sides of the base and covered all the bottom. There might have been marks on the fin but that was not recovered.

    If you wonder how a piece like that, encasing the base, came off so cleanly, so do I.

  166. Andrew says:

    @Victor

    RE: “There were four Boeing simulations in which the left generator and left backup generator were isolated so that the flight control degraded to SECONDARY after the right engine flamed out.”

    Thanks. Sorry, but my memory’s hazy on that point. I assume it was confirmed that Gysbreght’s trajectories #7-10 were in an abnormal electrical configuration?

  167. mash says:

    Corollary: Only certain Zigzag flights meet all the requirements of this Mystery mission.
    (see above June 10 9:23 pm)

    Therefore, most likely, the deciding factors for the target are:

    1. Flight’s content
    a. passengers
    b. cargo
    c. crew & fuel

    and

    2. Flight/plane’s technology
    a. control takeover possibilities (manned vs softwared).
    b. programmed flying mode options.
    c. landing/ditching/gliding capabilities.

    Btw, the first Satcom logon could also be triggered by a software exception, reportedly …

  168. Victor Iannello says:

    @Andrew: I don’t know which ones he is referring to, as I don’t know his numbering scheme. I disagree with his premise that there is TAC input in SECONDARY mode. What I do know is there were four simulations in which the left IDG and left backup generator were isolated. (I emailed you an image identifying these four.) In two of those simulations, there was a bank to the right leading to tightening spiral. In two of those simulations, the plane banked gently to the right after the first engine failure followed by a bank to the left and a tightening spiral. The gentle bank to the right followed by the bank to the left suggests to me there was left residual trim which partially offset the thrust asymmetry and became the predominant lateral asymmetry with both engines flamed out.

  169. Andrew says:

    @Victor

    Thanks. The colours of the tracks in your image are different, but the tracks appear to be those Gysbreght mentioned, assuming I understood him correctly.

  170. DennisW says:

    @Mash

    Btw, the first Satcom logon could also be triggered by a software exception, reportedly …

    Where has that been reported? Thx.

  171. irthe turner says:

    @TBill, I apologize if this topic has been covered ad nauseum before, but what is the probability of the FO’s phone detection over Penang on a 1st fly by at altitude ? Is the LIDO image reliable? Could ZS have taken his time to fully circle Penang, perhaps even at a lower altitude making a phone detection more probable and likely?

  172. Gysbreght says:

    @Andrew: If the TAC input is removed at power loss, how do you explain that the airplane continues eseentially straight for two minutes one engine out the remaining at CLB thrust, then turns several minutes at a rate not greater than in the dual failure cases?

  173. Gysbreght says:

    @Andrew: You explain that the TAC function is lost when primary flight computers (PFC) lack the information they need to calculate the correct rudder input. That doesn’t mean that any existing TAC input is necessarily removed at that time.

  174. MH says:

    @irthe turner : with the LIDO image in question and the radar data may not be of MH370, its highly unlikely the phone actually connected

  175. Gysbreght says:

    [Abrasive comments deleted.]
    …the numbering scheme here on June 11, 2018 at 4:20 am.
    https://www.dropbox.com/s/z02p174h4kdybad/BoeingSims.png?dl=0

  176. Don Thompson says:

    @Neville Macaulife

    While we worked to identify the vortex generator ‘chine’, originally attached to the engine cowl door, I reached a conclusion that the ‘fin’ part detached from its base due to the cowl door flexing. The base of the ‘fin’ shows delamination. FWIW.

  177. Victor Iannello says:

    @Andrew: To put some numbers to the page, in the simulations that Mike witnessed, when the right engine failed first, the TAC input was 2.5 units of left rudder trim. When the left engine failed first, the TAC input was 4 units of right rudder trim. Assuming the magnitude of the yaw produced by the thrust asymmetry was equal in both cases, that means the thrust asymmetry with an engine out is equivalent to (4+2.5)/2 = 3.25 units of rudder trim. It also means that even with no rudder trim, there was the equivalent yaw imbalance of (4-2.5)/2 = 0.75 units of left trim.

    So, in the Boeing simulations, with the left engine running, the left IDG and backup generator isolated, and in SECONDARY flight control mode, residual rudder trim of just a couple of units would have a significant impact on the flight dynamics. This again explains the two simulations with a gentle bank to the right followed by the bank to the left. There could have been a residual trim level of about one or two units of left trim, which reduced the bank to the right with one engine running, and resulted in a bank to the left after the second engine failed. On the other hand, the two simulations which increasingly bank to the right had little or no left residual trim.

  178. TBill says:

    @irthe turner
    “I apologize if this topic has been covered ad nauseum before-
    Is the LIDO image reliable?
    Could ZS have taken his time to fully circle Penang?”

    Nobody has suggested circle Penang which would put MH370 almost on top of Butterworth air force base. Also it is hard to fly circles.

    If anything MH370 might have been at FL400+ for the cell phone connect, which is somewhat bizarre. I don’t know if @ALSM thinks that is possible. We might postulate MH370 was at high altitude to prevent cell phone connects, yet one happened.

    Lido image confirmation or lack thereof is the type of info we could hopefully get with the new government in Malaysia.

  179. Victor Iannello says:

    @TBill said: If anything MH370 might have been at FL400+ for the cell phone connect, which is somewhat bizarre.

    The connection of cell phone to a tower at cruise altitudes is not really bizarre. The duration of the connection was very short. Essentially, the cell phone registered briefly on the tower. I’ve experienced the same during flights. For instance, in a recent flight to Europe, my cell phone connected to a Canadian tower while we were at around FL380. I received a text message welcoming me to Canada from the telecom.

  180. TBill says:

    @Victor
    “Power up of the left AC bus?
    The simplest…answer is some SATCOM functionality was desired, such as the ability to initiate or receive calls, or receive a signal in the form of a ring.”

    This reminds me another assumption to date, that the pilot was oblivious to implications of SDU pings and sat phone calls re: trail of crumbs.

  181. Victor Iannello says:

    @TBill: It’s possible the pilot thought by disabling ACARS, there was no means to track the aircraft. Certainly, there was no precise, real-time tracking.

  182. MH says:

    @VI, @TBill
    What is even more bizarre is that no other cell phones connected or registered
    For those on MH370 when from @VI’s experience that should have happened.

  183. Victor Iannello says:

    @MH: We don’t know whether or not any passenger cell phones registered on a tower.

  184. TBill says:

    @MH
    Obviously the cockpit has better windows, and to give you an inkling of my clandestine thinking process, I wonder if a window antenna could have been in use in the cockpit.

  185. DennisW says:

    @MH

    It is not clear how many passengers had a phone compatible with the Malaysian network. My guess is that most Chinese travelers rent a phone in Kuala Lumpur if they want a cell phone. My AT&T GSM phone does not work with a Verizon LTE base station.

    Actually I now have an LTE phone and installed a microcell (home base station at my beach house where there is zero coverage). Works great with an internet cable backhaul and it was cheap (~$250).

  186. mash says:

    @DennisW

    The log-on message sent from the aircraft at 08:19:29 was not immediately well understood. The 02:25 handshake was also initiated by the aircraft.:22 Only a few reasons that the SDU would transmit a log-on message exist, such as a power interruption, software failure, loss of critical systems providing input to the SDU, or a loss of the link due to aircraft attitude.:22

    copied from wikipedia:
    Analysis of Malaysia Airlines Flight 370 satellite communications

  187. MH says:

    @DennisW – “It is not clear how many passengers had a phone compatible with the Malaysian network”

    with an aircraft of over 250 pax, at least another cell phone should have made at least registration with tower on Penang other than the copilot plus if MH370 did fly back over Malaysia, there should have been many more registrations such as what @VI’s own experience. Far too much value has been put on the co-pilot’s registration at a tower on Penang, if it actually happened.

  188. @David,

    Thanks for another insightful note, David, re the engine fin damage. I just got a note from @Don Thompson (6/12, 3:30am), giving his conclusions that “the ‘fin’ part detached from its base due to cowl door flexing.” I’m expect you already know about that, but just in case.

    My RR emblem collision theory is now RRIP.

  189. airlandseaman says:

    I agree with Don that the fin separated from the base due to a bending force on the door (front to back) under the base. I built a 1/6th scale model to demo this.

  190. airlandseaman says:

    Altitude per se has little to do with the probability of a cell phone connecting. It is much more dependent on the radial speed (Doppler) and base station antenna side lobes. A connection from 40,000 feet is not unusual if the aircraft is flying nearly tangent to the radial vector, is within the range limit, and within a side lobe. All those conditions were easily met for the path at 17:52.

  191. DennisW says:

    @ALSM

    Yes. Several people have looked at the cell phone registration and concluded it was possible. The fact is, it occured. There is really not much else to extract from that event. It is useful relative to confirming the flight path and timing.

  192. airlandseaman says:

    Dennis: I should have noted I was answering TBill’s question: “If anything MH370 might have been at FL400+ for the cell phone connect, which is somewhat bizarre. I don’t know if @ALSM thinks that is possible. ”

    I agree it is not only possible, but fact.

  193. DennisW says:

    @MH

    with an aircraft of over 250 pax,

    You are talking gibberish including the number of passengers.

  194. TBill says:

    @ALSM
    Thank you for the technical explanation about minimal altitude effect on the cell phone connect. I wonder if the special windshield coating for the heater helps or hurts reception, or no impact?

  195. David says:

    @Neville Macaulife. Thanks, yes the fan cowl on which the vortex generator was mounted may well have buckled in compression. Aside from the bending that ALSM mentions, the position of the lost bolts, although ambiguous, suggests that there might have been torsion also.

    However while that might account for the fin (chine) separation to me it is also possible that that part of the cowl struck such as the wing which played a part.

    It is possible that unless Malaysia conducts (or sees to) a deeper damage assessment than done on other recovered items, and even if they do, we will be left to speculate.

    I mentioned the paint loss which differentiates it from the undamaged finish evident and also from the other fan cowl part recovered. At first glance shattered off but not at second. You will see that in the below.

    Because all of us might need a refresher on this whenever we get a Malaysian report here are some refresher URLs, including some good work done on this:

    https://www.dropbox.com/sh/gedw5unomsikbdn/AABk_yIbQsEwjp2F6lG0RIioa?dl=0&preview=Two+further+pieces+of+possible+MH370+debris+handed+over+to+Madagascar+Authorities.docx Note, photos unattached to this.

    https://www.dropbox.com/sh/gedw5unomsikbdn/AADFJE5–6XiLTHc3327Pfd4a/Possible%20MH370%20debris%20handed%20in%20to%20Madagascar%20August%202017?dl=0&preview=Madagascar+debris+handed+in+August+2017+24.jpg

    https://www.dropbox.com/s/ekdwjhui02xi4lk/MH370%20Debris%20Analysis.pdf?dl=0

    https://www.dropbox.com/s/t921zi4tqydwkcq/Compare.JPG?dl=0

    https://www.dropbox.com/s/3wy4dh3pox5gy49/Door%20debris%20%26%20VG%20analysis.pdf?dl=0

    https://www.airteamimages.com/pics/163/163785_big.jpg

  196. mash says:

    @Sabine Lechtenfeld
    @ all

    re: Maldives sighting

    Thanks for you inspiration/implication.
    As this could be a wonderful discovery.

    I say that the plane is exactly that, an impersonating plane.
    Does that make full sense?

    How do you explain the coincidences, ‘same’ time and location, as if they knew the real happened/happening event?

    This is a calculated deception, a sophiscated operation and a well-organised mission. Al least not a pilot suicide …

  197. Victor Iannello says:

    @mash said: How do you explain the coincidences, ‘same’ time and location, as if they knew the real happened/happening event?

    If the sighting was unrelated to MH370, I don’t think it really is an extraordinary coincidence. My guess is that at any given time during daylight, there are unidentified airliners that people see. For instance, there was a sighting in Australia of an airliner on March 8, and some remain convinced it was MH370. Look at all the witness sightings of planes at night, separated by distances and time that prove they could all not be MH370.

  198. DennisW says:

    @mash

    The Inmarsat data is very solid, IMO. It cannot tell us precisely where the plane terminated, but it can certainly tell us where it did not go. The Maldives, Indian Ocean, Diego Garcia, Australia… can definitely be ruled out. There is no question that it terminated “near” the 7th arc.

  199. ventus45 says:

    Page 39 of Larry Ston’e presentation shows his three areas (green yellow and red boxes) between latitudes 26S and 21S. Has Metron advised OI ?

  200. David says:

    @Don Thompson. Keeping away from simulations, a check please. Conventional wisdom is that transfer bus power loss would result in loss of A/P because of pitot heat loss. As I read it the ADIRU relies on the AoA sensor data. Heating power for that is via the left AC bus, Sec 1:(TM 34-20-00 p80,81).

    If an unpowered the left AC bus also results in reversion to secondary, the A/P would be lost with transfer bus power available still. Presumably that is not so?

  201. mash says:

    @DennisW

    Sabine Lechtenfeld says:
    June 6, 2018 at 6:43 am
    Since I mentioned the Kudahuvadhooan sighting of a large, low and very noisy plane over the tiny Maldive island on March 8, 2014, just after local sunrise, I may as well point out a very quirky fact: if the sat com hadn’t made a comeback at 18:25 UTC, we wouldn’t have any sat data. And if we wouldn’t have any sat data, the Kudahuvadhooan sighting would’ve been treated very differently. It would’ve been entirely plausible that MH370 might’ve continued on a route westwards after the turnaround near IGARI, even if the available radar evidence seemed to point more into a northwestern direction. But the Central Indian Ocean would’ve been searched high’n low for plane wreckage. Nobody would’ve looked towards the SIO for a long time.

  202. mash says:

    @Victor Iannello

    You can search say “mh370 maldives” to find out more and make you own judgment.

    For example Florence de Changy’s …

  203. Gysbreght says:

    @Andrew: I think we discussed it back in february:

    (a) If the TAC is switched off – any trim applied by the TAC remains in
    (b) If the TAC fails – any trim applied is removed (Trim centred)

  204. Gerald says:

    Coming back to cell phones, taking pics before take off is a usual habit of folks traveling on planes. But I haven’t heard of any pics received by NoK from their friends or relatives. Aren’t there any photos on the web? Very strange to me!

  205. Don Thompson says:

    @David wrote “As I read it the ADIRU relies on the AoA sensor data. Heating power for that is via the left AC bus, Sec 1

    When power to the pitot sensors heaters is lost the PFCs revert to secondary becuase fundamental air data cannot be assumed to be reliable, therefore the autoflight systems cannot be relied upon. Without reliable air data, the aircraft then must be flown using the sensors provided by the man-in-the-loop.

    I’ll offer that AoA, and TAT, are not regarded as fundamental air data terms required for continued operations of the AFDS.

    The ADIRS produces corrected and calculated AoA terms that are used by the GPWS system, in windshear detection, and the WES, for a number of its functions. The WES functions include stall warning/stick shaker initiation, calculation of speed tape parameters displayed on PFD, and signalling to FSEU to modify LE slat deployment.

  206. Andrew says:

    @Gysbreght

    RE: “@Andrew: I think we discussed it back in february:

    (a) If the TAC is switched off – any trim applied by the TAC remains in
    (b) If the TAC fails – any trim applied is removed (Trim centred)”

    If we did, I can’t find the discussion. That quote is taken from the following webpage, under ‘TAC Fun Facts’:

    The Boeing 777 Thrust Asymmetry Compensation (TAC)

    I assume the ‘Gysbreght van Aemstel’ who posted a comment at the bottom of the page is you?

  207. Gysbreght says:

    @Andrew: Yes, that’s me.

  208. @victor
    Thanks for your reply on June 11 at 3:36pm from which we could calulate the fuel consumption for your new path.
    From your reply, one can conclude that the path does not include extra specific maneuver between Pulau Perak and Arc2. But as this part of the flight which conditions very much the remaining legs cannot be ignored, we assumed a smooth flight for arriving at Arc2 on time with the characteristics given in your table. This means that a very long detour of additional ~350Nm took place (you called it a loiter didn’t you ?) between Arc1 and Arc2.

    Based on the above, the fuel consumption computed every second during the trajectory using Bobby Ulich’s fuel model and using the actual meteo data is summarized in this table:

    Waypoint Fuel On Board (Tons) distance from Zero Fuel locus(Nm)
    IGARI 42,2
    Pulau Perak 37,6
    Arc1 34,8
    VOCX – Nicobar 26,2
    Arc2 25,9
    Arc3 18,9
    Arc4 12,0
    PCCNE 8,3
    Arc5 6,4
    ATC-Phone-Call 3,6
    Exhausted fuel 0,0
    Arc6 -1,1 83
    Arc7 -1,8 127

    Would you agree with these figures ?

    This leads to these conclusions:

    a- According to Bobby Ulich’s fuel model, the Zero fuel location is (-20.27° /102.27°E).

    b- 1.1 tons of fuel is missing at Arc6 which thus cannot be reached by ~83 Nm nor Arc7 by ~127Nm. (as a comparison: Dr Ulich’s model indicates 187kg of fuel at Arc6 for CAPTIO trajectory and 0 at Arc7 , cf our report).

    c- The rationale of a large detour (a loiter as you call it) between Arc1 and Arc2 of ~ 350Nm is paradoxal for an aircraft which was speeding to get out of the « scene of the crime » and which behaved to stay undetected. It would have lost 40min by this detour instead of going direct to VOCX distant from Arc1 by ~130Nm.

    d- In addition, during the detour, the aircraft would have entered Nicobar radar detection while it tried to stay « invisible ». This is another paradox. It could have flown direct to stay out of the radar coverage.

    e- Except to the fact that it provides a potential path to Cocos compatible with the pings, the altitude 32 000 ft is not an intermediate level that a pilot – who entered the new route and targeting to land safely at Cocos according to your new scenario – would select. This is playing Russian Roulette when crossing the 5 encountered airways through their mid flight-levels especially considering the potential traffic from KL or Singapore to Europe on these routes for ex. No pilot without TCAS would take its chance. Passing below them is more probable, but then no more compatibility with the pings.

  209. Re previous post : figures with “,” should be read with “.” in English.

    Sorry for this cut and paste from European format.

  210. David says:

    @Don Thompson. I have now found my ADIRU reference, AMM 27-02-00 p36 which in effect says there will be reversion to secondary on loss of ‘….attitude sensor data’. If AoA sensing were fundamental to that the A/P would be lost at a left main AC bus de-power, not denoting total AC loss.

    However not being fundamental the ‘attitude sensor data’ can be calculated without, so that does not arise. Thanks for that.

  211. flatpack says:

    @Jean-Luc

    “…is paradoxal for an aircraft which was speeding to get out of the « scene of the crime » and which behaved to stay undetected”

    No. The aircraft was the scene of the crime.

    “This is playing Russian Roulette when crossing the 5 encountered airways … No pilot without TCAS would take its chance.”

    You are kidding aren’t you?

    Get real, this guy had already murdered everyone else on board MH370 and yet you continue to model his behaviour along the lines of “No pilot would blah blah blah”

    Maybe you are unintentionally correct and he was in fact playing Russian Roulette.

  212. Tim says:

    @all,
    As someone who believes this was just an accident caused probably by the rupture of the crew O2 bottle, I think it’s time we admit that it is impossible to determine the exact course this aircraft took.

    Trying to fit waypoints/tracks and headings to a pilotless autopilot off flight will not work. And as the aircraft wasn’t flying efficiently and assuming the 7 th arc is correct, a future search north along the arc is all we can do. When considering drift currents, maybe it is between 25-20S.

  213. TBill says:

    @Tim
    Well at least we can agree on search location strategy. I agree north of 25S tends to indicate either (1) active pilot or (2) a passive flight limp mode we have not yet examined. Most seem to feel passive-A/P off falls in the water.

  214. Bruce Robertson says:

    @TBill
    “Most seem to feel passive-A/P off falls in the water.”

    Not exactly. Once the plane goes beyond certain parameters (e.g. excessive roll or pitch), the envelope protection is triggered and the autopilot enables itself. Flight automation is *always* involved as this is a fly-by-wire aircraft. Envelope protection will return the aircraft to a more conservative attitude and wait for a human pilot to take over. With MH370, that never happens and the fuel runs out at Zenith Plateau, 20.8S 103.9E.

  215. Victor Iannello says:

    @Jean-Luc: I don’t agree with your fuel calculations. The ongoing descent at 18:40 could have been to holding speed at FL200, which would reduce the fuel flow to the minimum achievable during the “loiter” (which could have been an excursion or a racetrack holding pattern). After YPCC, the fuel flow at 210 KIAS would also be only a small amount more than the holding fuel flow rates.

    As for your “paradoxes”, it’s only because you are trying to shoehorn other scenarios into your chosen scenario. For instance, I see no demonstrated intent for the pilot to stay out of radar range. In fact, MH370 was captured by Malaysian civil radar and possibly military radar. As for your comments about altitude, “safety” was not a primary concern, or a diversion would not have been attempted. Why your “safe” pilot was so careless to run out fuel after passing Christmas Island makes no sense.

    I think a more relevant question is to ask why would a pilot choose FL320, as it does not maximize range, nor does it minimize fuel flow. It’s possible that that there are other altitude/speed combinations that satisfy the satellite data, as I have not systematically explored all combinations, as my primary motivation was to demonstrate that at least one automated flight exists that satisfies the BTO and BFO data, and to elicit comments about the scenario in which there was a left turn and deceleration at YPCC, something that I had not seen before.

    I learned long ago to not fall in love with any particular scenario as the probability of landing on the true scenario is small. I think that advice is too late for your group.

  216. Victor Iannello says:

    @Bruce Robertson said: Once the plane goes beyond certain parameters (e.g. excessive roll or pitch), the envelope protection is triggered and the autopilot enables itself.

    That’s not the way it works. While in normal flight control mode, if bank angle exceeds 35°, the envelope protection reduces the bank to 30°. The autopilot does not automatically re-engage. A plane with a bank angle held at 30° will fly in circles.

  217. @Don Thompson

    Thank you for sharing, I’m sure your right about the cowl door flexing. As I told @David, my RR emblem collision theory is now RRIP. Let’s hope those parts get the kind of examination they merit, with nothing behind closed doors, but from what I’m learning, MH stuff doesn’t always work that way.

    Keep on truckin’, I guess!

  218. mash says:

    re: Captio Trajectory

    There are at least two types of plan A/plan B scenarios, namely 1) Ransom and 2) Backup.
    For example, in pseudo lang: (if negotiated B else A / if detected B else A ; where A = water, B = land)

    Similarly, there are two types of fuel consumption methods, namely A) Saving and B) Wasting (in the sense to perform a no-fuel glide at the appropriate point).

    And the trajectory, especially the hard-to-explain arc6-7 part, looks more like a Backup and Fuel exhausing type. Otherwise why go towards land instead of to the middle of the deep sea initially …

  219. @David

    Wow David, A whole MH smorgasbord! and as you say, it provides a great refresher for anyone.

    I looked through the photos right away. I found something of the Vietnam Wall about the pieces – voices stilled forever, yet you wait to hear them speak. Then the pictures of the aircraft looked so beautiful, and I can’t understand anyone wanting to destroy such great work, anymore than I can understand random murder.

    I’ve been wondering if any of the unidentified pieces have ever been posted in 777 factories. The people who crafted them with their own hands would have the best chance of remembering something. -Just a thought.

  220. Sfojimbo says:

    @Tim
    “I think it’s time we admit that it is impossible to determine the exact course this aircraft took.”

    How exact do you mean? I can tell you within a few miles and a couple of minutes where 9M-MRO was for all the time between takeoff and the 18:28 ping ring; at 18:28 I know within two miles where the plane was. I know that 12 minutes after that first ping ring it was on a southbound course where it intersected each ping ring at the times recorded. I also know that the plane, being piloted by an experienced pilot, would have flown at mach 8.3 or a little above, and would have maintained cruising altitude.

    There is nothing mysterious about any of this unless you want to imagine that the plane flew around in circles for a bit because the pilot “felt guilty”. Or if you want to imagine that a long series of events which all began in the middle of FIR turnover were caused by a ruptured O2 bottle. Things get really really complicated if you believe in either of those happenings.

  221. Victor Iannello says:

    @Sfojimbo: Yes, your logic is why back in July 2014 we predicted an impact point along the 7th arc at 37.5S. Since you believe there is no mystery, where exactly do you think the debris field lies?

  222. Andrew says:

    @Sfojimbo

    ”I also know that the plane, being piloted by an experienced pilot, would have flown at mach 8.3 or a little above.”

    Wow, 9M-MRO must have been the fastest B777 on the planet!

  223. flatpack says:

    @Sfojimbo

    “I also know that the plane, being piloted by an experienced pilot, would have flown at mach 8.3 or a little above, and would have maintained cruising altitude.”

    Mmmmm, the plane must have been a waverider. He wasn’t ditching, he was surfing.

    On a more serious note, as I noted earlier today, it doesn’t seem prudent to expect the behaviour of a ‘normal’ experienced pilot, such as maintaining cruising altitude.

  224. Victor Iannello says:

    @Andrew: I gave him the benefit of the doubt that he meant to say Mach 0.83.

  225. Andrew says:

    @Victor: I’m not that kind…

  226. @Victor
    I think there is a missunderstanding: we try to help “challenging” and to avoid studying scenarii with very low probability and for which we have some experience as Cocos was indeed the first potential site for a safe landing of our many studies, simulations and computations.

    Coming back to the essential: the fuel consumption.
    At this time of the flight and at this weight, descending to a loiter at FL200 and climbing back to FL320 allows to save roughly ~400kg compared to the minimum consumption scenario previously computed for you at a constant altitude. In addition if you envisage a potential racetrack holding, this “saving” will be reduced because of the additional turns which are more demanding in fuel.

    In conclusion, about ~700Kg are missing to reach Arc6.

  227. Victor Iannello says:

    @Jean-Luc: Do you think your assignment of “low probability” might be a little subjective? Your favored scenario has the hijackers hidden in the MEC, climbing out, taking over the cockpit, yet “safely” navigating, expecting to land on Christmas Island, yet passing it and running out of fuel. Some here might also consider that “low probability”.

    I’ll try to put something together on fuel consumption. Basically, the time spent at holding speed and FL200 combined with the last two hours (!) spent at 210 KIAS significantly reduce the fuel consumption compared to cruise conditions. Also, bear in mind that at 17:07, the fuel load was 43,800 kg. Your 700 kg of missing fuel is only 1.6% of that quantity. How accurate do you believe your fuel model is, especially at speeds less than holding speed?

  228. TBill says:

    @Jean-Luc
    From looking at Flight Radar 24, there seems to be a time of day around 3AM that there is no air traffic to speak of along Indonesia. Until EK425 takes off from Perth 6AM and could have passed close to MH370 at 22 South on L894 (I’ve checked into EK425 on 8-March). In any case looks like holding +500 would be safe altitude, and as you , TCAS could be back on by that time.

  229. Sfojimbo says:

    “we predicted an impact point along the 7th arc at 37.5S”

    What has changed since then? I know Duncan Steel had predicted 37.5s, but that isn’t where the searches have taken place.

  230. Victor Iannello says:

    @Sfojimbo said: I know Duncan Steel had predicted 37.5s, but that isn’t where the searches have taken place.

    You are again making arguments without knowledge of the facts. The first 120,000 km2 of seabed searched was centered on the DSTG hot spot of 38S, which was very close to the IG’s prediction. (In Sept 2014, the IG issued a report that recommended searching at 37.7S latitude.)

    Our thinking has evolved since then based on the failure of the ATSB-led and OI-led searches, the recovery of debris, drift models, improved aircraft performance models, and better understanding of the BTO and BFO data.

    This blog is not the place to get you up to speed on the long history of MH370 events. I would advise you to at the very least read all the official reports issued by the ATSB, as well as the Factual Information issued by Malaysia in March 2015.

    In light of your complete ignorance of the basic facts, the strong statements and accusations you make are baffling. I have little patience left.

  231. @Victor
    The last two hours at KIAS 210 kt are indeed part of the computation in the table provided to you yesterday.
    We use Dr Ulich’s model and we compute the fuel every second according the aircraft parameters for any detailed flight plan that we enter as an input. Some times when required, we put some additional constrains on certain points if needed like time at the point, RoC, speed, track, meteo etc. Not necessarily all of them but the necessary subset.

    Today’s estimation for the holding was made via the flight simulator to get a first order estimate in order to avoid to design a new flight plan.

  232. Sfojimbo says:

    The Fugero Discovery did get almost to 39s but in a fairly narrow search path. Everything since then has been farther north.

    Is it my rejection of the sortie to the north and all the speed changes and altitude variations that distress you so Victor?

  233. TBill says:

    @Jean-Luc
    Do you consider the no bleed air case?

  234. Victor Iannello says:

    @Sfojimbo said: The Fugero [sic] Discovery did get almost to 39s but in a fairly narrow search path.

    Totally false. The search was widest on this part of the arc. Near 38S, it was about 77 NM wide.

    Alternative scenarios don’t distress me. What does distress me is how many times you’ve made false statements, even after being warned. For this reason, you are banned.

  235. Victor Iannello says:

    @Jean-Luc: It’s possible that you very accurately entered the input values into the fuel flow model. I am saying that you place a higher precision on the fuel calculations than is warranted, especially at speeds less than holding, which are outside of the database of fuel flows that was used to develop the model (LRC and holding tables). The fuel flow before fuel exhaustion might have been around 5500 kg/hr. A 700 kg discrepancy amounts to 7 minutes of endurance.

  236. @Jean-Luc

    “a- According to Bobby Ulich’s fuel model, the Zero fuel location is (-20.27° /102.27°E).”

    Interesting, that’s in the vicinity of where Ocean Shield detected pings at 33Mhz on April 5 and April 8, 2014.

    https://www.dropbox.com/s/7e5lv9i2ok8bdus/MH370%20%E2%80%93%20Flight%20Path%20Analysis%20Update%2030%20July%202015%20%28Fg12%2C%20Pg13%29.png

  237. Victor Iannello says:

    In a new article from Geoffrey Thomas, Chari Pattiaratchi says the next search should along the 7th arc between 28S to 36S, but wider than previously searched.

  238. Richard Godfrey says:

    @Victor

    Prof. “Charitha Pattiaratchi is emphatic about where he believes MH370 is lying.”

    The astounding new revelation is somewhere outside the previously searched area between 28°S and 36°S.

    This is simply the combination of Chari’s previous study result of 28°S to 33°S and David Griffin’s previous study result of 32°S to 36°S.

    Chari may be “emphatic”, but he is not very precise. That narrows it down to an Australian politically correct length of the 7th Arc, which is only 1,366 km long.

    If Chari really believes that the Drift Analysis is so imprecise, he should give up making such statements.

  239. Victor Iannello says:

    @Richard Godfrey: I think he believes that any impact site south of 28S would have delivered the debris too soon compared to the date of the reported discoveries. He is clearly in the camp of “wider” rather than “farther north”.

  240. Richard Godfrey says:

    @Victor

    The statement from Chari is political and not scientific.

  241. @Victor
    @TBill

    In order to progress systematically and avoid to compute patches over patches on the trajectory which brings confusion and will lead to unreliable results, may I suggest the following to Victor:

    It is possible you could…

    1- complement the description on top of this blog and present your complete scenario from Kuala Lumpur up to the End of Flight
    2- clearly indicate which data you use and which you decided to ignore (for ex the flight characteristics above Kota Bharu or which exit from radar coverage you selected as well as Arc1 crossing location etc.)
    3- clearly indicate along the trajectory where and when some parameters do change (like the air conditioning etc.)
    4- provide a complete 7D trajectory (4D + speed + RoC/RoD + track) on top of the key points provided in the introduction of this blog. This means the minimum set of waypoints and the important points where the flight characteristics do change like turns, start and end of climbing after IGARI or top of descent before the holding and its foot of descent, then the beginning of climb at the end of the holding) and possibly the gliding/falling at the EoF.
    5- specify the error margins that you consider now as acceptable with the subsequent rationale as you are relaxing them somehow (for ex 2 BFOs of the your proposed trajectory are now out of the so far considered acceptable +/- 7Hz range). This adresses all BFOs, fuel estimation and also the debris drift dates of arrival at la Reunion and others you can think of…

    It would be great because then, at each of your key points, we could help you in providing figures computed by our CAT (Constrains Assessment Tool) as a feedback on the assumptions.
    I do hope this is acceptable way to proceed, isn’t ?

  242. Victor Iannello says:

    @Jean-Luc: If there is general interest here, I will consider providing more data points to supplement the ones I provided above, as I compute all data each minute (and to the second at the handshake times). As for BFO error, I don’t think anybody knows what is an acceptable error. If you’ve been following along, you know that I’ve shown evidence that previously the bias changed after a re-boot due to oscillator retrace error. There also could have been some drift after reboot.

    Of course, if I were to allow pilot inputs along the path as you have done, all satellite data could be exactly matched.

  243. TBill says:

    @Victor
    George Bibel said “…if I tried to disappear, I’d change course a few additional times” to make the crash site hard to find.

    Here is a logical “George Bible” path to 22S. This path assumes a diversion to the northeast at the 23:14 sat call. In SkyVector:
    0698E 0795E 2591S 2895S 22S02 22S04

    Here is a similar path to 20S:
    0698E 0894E 2293S 2595S 20S03 20S05

    (These paths based on the IG’s orig paths, assuming however, after Arc5 we have less clarity that MH370 stayed on that straight path)

    >> For the future, I wonder if we should assign 50/50 probability to passive vs. active flights. I presume active flight possibility brings in locations north of 25S (per my post and your OP). Searching 20-24 South could be just a few weeks for OI. >At the moment, I don’t feel drift studies should completely trump flight path possibilities.

    @Jean-Luc
    I am struggling with prioritization of outside hijackers and/or active negotiation flight paths, thinking we may need some evidence to sway the search in that way.

  244. Victor Iannello says:

    @TBill: Why do you think you can predict the path of MH370 if the path was meant to be unpredictable?

  245. @TBill

    You said: ‘George Bibel said “…if I tried to disappear, I’d change course a few additional times” to make the crash site hard to find.’

    Another idea: I haven’t heard anyone discuss this before, but is there anything to suggest the pilot might have hot-dogged the aircraft? I don’t know if you sit in the Big Seat yourself, but after thousands of hours of flying by the book, pilots must sometimes fantasize about cutting loose, and however sickening the thought, MH370 provided the perfect opportunity.

  246. DennisW says:

    @all

    I am still thinking about what to do next. I agree with Richard that Chari’s statement reeks of politics. Searching wider is basically a non-starter from a funding point of view. Victor’s scenario has merit, IMO. But as he says, the probabilty of it cannot be defined.

  247. MH says:

    @Jean-Luc :after reviewing your videos a few times I have to wonder more in detail regarding the three tropical cyclone affects on the debris drift. I would think it’s possible if the debris was pushed further south making the actual sea impact much more north.

  248. TBill says:

    @Victor
    “Why do you think you can predict the path of MH370 if the path was meant to be unpredictable?”

    I only said the active paths to 20-25 South were logical derivatives of 32-38 South passive paths, not necessarily predictable.

  249. Victor Iannello says:

    @DennisW: At this point, I can find reasons to search many areas in the SIO. However, I don’t see a way to reduce those areas to a manageable size. There’s still time, however, for new data or new insights to surface.

  250. mash says:

    Is it possible to find a ‘simple’ programmed path by throwing away the PSR data completely?

    From day one the official statement is “It could have been…but we are not sure”. It is un-reliable, and worse, it could be mis-leading.

    There is no connectivity, or an intentional connectivity, just a few minutes after the SSR data and a few minutes before the Inmarset data …

  251. airlandseaman says:

    The military radar, civil psr data, 1752 cell pnone registration and Lido Hotel radar taken together are too consistent to all be wrong. They tie the path from 17:21 to 18:25 convincingly.

  252. Richard Godfrey says:

    @Dennis, @Victor, @Don,

    I am working on a new drift analysis. I have extended the database to cover floating debris tracks in the Indian Ocean from starting points on the 7th Arc from 10°S to 40°S. I have increased the number of drifter buoys used to build the database, to ensure a comprehensive coverage of all the Indian Ocean. The software model spatial granularity is 1° of Latitude and Longitude, the temporal granularity is 1 day and the seasonal granularity is 1 month. The forward path track is recalculated for each day and uses only the relevant position and seasonal data from the database. The software calculates the average track in each cell from the GDP historic data, which has a spatial granularity of Latitude and Longitude to 3 decimal places and a temporal granularity of 6 hours.

    I am building a library of tracks for all start points on the 7th Arc from 10°S to 40°S in increments of 0.5° of Latitude. I am building a table of results for when and where debris passes Rodrigues, when and where debris passes Reunion and when and where debris finally hits land.

    I am focusing in particular on the locations of Var-Brûlé Beach in Rodrigues after 753 days, St. Andre in Reunion after 508 days and Mossel Bay, South Africa after 655 days, as these are the only locations where MH370 floating debris has been found with barnacles still attached. Barnacles demonstrate a relatively recent arrival. Barnacles die within a few days, when out of the water. Also barnacles are cleaned off by crabs and other scavengers in a couple of days.

    There are already some interesting results. For example, I will be able to show that all floating debris reaching Tanzania, Mozambique and South Africa most likely took the long route around the Northern tip of Madagascar, due to a much better fit to the drift data at the time of passing Madagascar and beyond. I also hope to be able to explain how it took nearly 753 days for floating debris to reach Rodrigues, whereas floating debris to Reunion and even South Africa was much quicker. The delay in reaching Rodrigues was most likely due to floating debris being stuck in a mid ocean gyre for a while. There is usually such a gyre ESE of Rodrigues.

    Finally Don and I have been working on a comparison of the GDP historic data to the actual data from March 2014 from 17 GDP Drifters that passed the 7th Arc, as well as the SLDMBs drifter data, which were launched from aircraft during the aerial search, as well as David Griffin’s synthetic drifters from actual satellite data. Initial results show good alignment between actual and historic data.

  253. flatpack says:

    @Neville Macaulife

    “I haven’t heard anyone discuss this before, but is there anything to suggest the pilot might have hot-dogged the aircraft?”

    Yep, the initial extreme manoever to asphyxiate the passengers. Possibly followed by attempts to subdue those using oxygen bottles.

    I think there were also extreme manoevers later on, such as dumping fuel and maybe trying to ignite it.

    If he tried anything in his sim, he probably tried it in MH370.

  254. Victor Iannello says:

    @Richard Godfrey: That sounds like quite a lot of work. Thank you for this undertaking, and please keep us posted.

  255. TBill says:

    @flatpack
    ….fyi here is a fuel dumping screen grab from PSS777:
    Fuel Dumping PSS777

  256. Don Thompson says:

    Adding to Richard’s comment above.

    I recently linked, in a comment, a video clip depicting an animation of SLDMB drift tracks and CSIRO’s simulated particle drift. That clip was flawed in as much as Google Earth didn’t seem to keep up with simultaneous animation & rendering the recording to video file.

    I am preparing further animations using the SLDMB tracks and CSIRO drift simulations. Unfortunately, the SLDMB data set is limited, temporally and origination along the arc, but the combination of the recorded SLDMB and computationally simulated particle drift is useful. Stay tuned!

  257. DennisW says:

    @Richard/Don

    Thanks. That is a mountain of work. From time to time I feel guilty about not getting engaged with the drift work or the aircraft dynamics at the end of flight, but Andrew (and others) are so knowledgeable about the latter, and you guys are light years ahead of me in the drift analytics.

  258. mash says:

    This area was searched based on the primary radar data which indicated an unidentified aircraft had flown up the Strait of Malacca, thought to be MH370.

    (above statement copied from page 15 of the ATSB’s report – section Malaysian led surface search)

    Therefore, the PSR is not 100% reliable, as suggested above.

    In any case, the purpose of eliminating the PSR data is to explore the possibility of a ‘simple’ model, say

    a. A fully automated flight path.
    b. A more flexible 1st arc starting-point.

  259. airlandseaman says:

    Mash: What ATSB Report are you quoting? Regardless, the statement “…Therefore, the PSR is not 100% reliable, as suggested above….” is misleading at best. A lot more is known today from the primary radar data then was known early in the search when the first reports came out. It would be foolish to ignore this data when trying to determine the next p-lace to search.

    The primary radar data is for the most part 100% reliable. It tells us a lot, like the path between 17:30 and at least 1802, if not 18:22. We also have derived important speed and altitude information from the data. And we have determined that the plane was likely flown by hand between 17:30 and 18:02. The only thing that is now known to be “unreliable” are the military long range altitude estimates circa 17:38. The near range estimates circa 17:52 were much more consistent with the KB civil PSR derived altitude (~43,000 ft).

  260. Paul Smithson says:

    @ALSM.

    Would you agree that:
    1. the radar data indicates a decline in the high GS of ~530kt had already begun at some point before 17:51:xx as the aircraft was approaching the west coast, with RMAF Butterworth abeam?
    2. By 17:53:xx, south of the island, the slowdown to ~510 kts was already largely completed?
    3. By the time of the final cluster around 18:00:xx, ground speed has dropped a few knots further, to approx 506-7kts?

  261. airlandseaman says:

    Paul:
    1. No. ~530 kts at 17:51
    2. Yes. ~510 kts at 17:53
    3. No. Still ~510 kts at 18:02

    I believe there was very little change in TAS. Most of this change was due to the right turn from running mainly with a strong tail wind component to more of a cross wind. The ground speed remained very high (~500 kts) to 18:22, but changed circa 18:25 to 18:28 (slowing).

  262. @flatpack
    I asked if MH370 might have been hot-dogged, and you replied:

    Yep, the initial extreme manoever to asphyxiate the passengers. Possibly followed by attempts to subdue those using oxygen bottles.
    I think there were also extreme manoevers later on, such as dumping fuel and maybe trying to ignite it.
    If he tried anything in his sim, he probably tried it in MH370.

    I thank you for the info, but I’m beginning to feel sorry I asked!

  263. mash says:

    If one could accept an EoF glide scenario, and even more mysteriously, a ‘good’ intention one; the “suddenly awaken” pilot would probably glide in the direction of:

    a. land and civilisation, for SAR purposes.
    b. back along the coming path, perhaps instinctively.
    c. longest and safest according to condition (visibility, wind, sea surface, altitude, etc).
    d. depending on the availability of operable controls and circumstances.

  264. Niu Yunu says:

    mash says: If one could accept an EoF glide scenario, and even more mysteriously, a ‘good’ intention one

    maybe the hypoxia scenario – when the plane ran out of fuel and the alarms stated to go off, one pilot “woke up” and tried to “glide in the direction of land and civilisation for SAR purposes” as you said.

  265. David says:

    @Victor. About what should be expected of damage assessments in the final report, another item needing more work is the recovered part of the left outer flap, Malaysian item 10. So far, all that we have published is an ATSB summary of evidence of its origin:
    https://www.atsb.gov.au/media/5773388/debris-examination-update-5_amended.pdf

    In the AMM (27-51-00 page 64) the outer flap’s construction is described thus,”The skin is reinforced graphite/epoxy bonded to a core of Nomex Honeycomb”.

    Despite this, from the cross-section in the bottom right ATSB photograph it looks to me that the skin to the rear of the rear spar is brittle and non-fibrous. That can be seen better at the right outer flap part recovered here:
    https://www.atsb.gov.au/media/5772902/mh370_inboard-section-of-flap-1.jpg

    The skin in front of the spar is of carbon fibre. That will be stronger and less brittle than the skin to the rear if at least of comparable thickness, which it does appear to be.

    To me the carbon fibre looks to have been torn in tension along the fasteners on the rear spar bottom edge, the top skin then breaking in bending. Note that it has been hit very hard from the bottom judging by that hole.

    I do not pretend to have expertise in carbon fibre failures or information other than the photos and perhaps the failure was the other way round.

    Which ever way it was though, why would this flap’s trailing edge have failed at the carbon fibre when the skin behind the spar would be more vulnerable in tension or bending? Note the failure is to the spar’s rear in the right outer flap, though possibly that was in torsion.

    The left’s part is short and its ends quite perpendicular suggesting there was little torsion but that there was bending along its length.

    Put together with assessments of other recovered items by those well informed, with the objects to hand and access to undamaged parent items, I would hope that something comes from the overall damage assessment which might help define end of flight attitude, configuration and speed more confidently and closely. The ATSB does indicate the “Malaysian ICAO Annex 13 Safety Investigation Team” has carriage of that.

    In the meantime maybe others can throw more light on what the damage to this particular item might mean.

  266. flatpack says:

    @mash

    My take is that the pijacker possibly entered a fugue state.

    This state may have been ‘enhanced’ by liberal use of powerful painkillers. Remember the gliding accident that left him with severe back pain?

    I don’t think the “suddenly awaken” pilot was a ‘good’ intentioned one, merely that he made some attempt to preserve his own life for religious reasons. I don’t think that any probable glide direction can be reliably inferred.

  267. Don Thompson says:

    @David,

    It’s my understanding that the item that is the subject of the ATSB’s Debris examination – update No. 5 is predominantly the trailing edge wedge from the outer end of the outboard flap, and that wedge remains fixed to a fragment of the aft spar of the flap’s core torsion box. The trailing edge wedge is a full depth honeycomb form faced with GFRP skins, whereas the core torsion box is constructed as CFRP ribs, CFRP spars, and CFRP-Nomex honeycomb panels.

  268. David says:

    @Don Thompson. Earlier you identified the recovered MH 17 outer flap part. Here are some other MH 17 flap items identified. Of passing interest the inner carrier of the outer flap shown here didn’t break and come away with the inboard flap part like the MH370 right side.
    https://www.dropbox.com/s/1vnq3u0qki39tqa/MH17%20wreckage%20joostniemoller.nl%20P1460386.jpg?dl=0

  269. mash says:

    re: Oceanography 100 vs Glideology 100

    Niu Yunu : Maybe “hypoxia”, but still undecided whether it is plane hypoxia or pilot hypoxia.
    flatpack : Not necessarily no “probable glide direction” – it must be a purposeful glide, if any.

    Say assume it is a back-to-civilisation glide and the centre-of-civilisation is an invariant point. As a first approximation, let this point be the centre of the 1st Inmarset circle. It is then quite easy to determine the glided 7th arc based on certain gliding distance calculation.

    Similarly the centre-of-civilisation can be expanded to a circle to allow for margin of error.

    Anyhow, neglecting Australia, it is more likely a Big Crunch glide than a Big Bang glide.

  270. Gysbreght says:

    The plot of “Time Difference from Constant Speed” (Victor Iannello, May 1, 2018 at9:12 pm) offers a way to filter out quantisation errors from the Civil PSR data to determine the variation of speeds. The following chart shows this for a target height of 43,000 ft:
    https://www.dropbox.com/s/dbvvje8xg55t85f/PSR_43kft_speeds.pdf?dl=0

    Between 17:36 near Kota Bharu and 17:52 South of Penang the groundspeed is nominally 530 kts, with variations of +/- 12 knots. That variation is quite large and corresponds to altitude varations of +/- 500 feet.

    A competent pilot flying manually should be able to maintain a speed within +/- 5 knots, and an altitude within +/- 100 ft. (Reference: Continental 777 Flight Manual Sec. 3 Page 333, Acceptable Performance).

  271. Tim says:

    @Gysbreght, surely this civil PSR information is showing us that this was a pilotless, meandering flight. Vertically it was flying phugoids.

  272. Gysbreght says:

    I’ve added the Military PSR data from Bayesian Methods.

  273. Victor Iannello says:

    @All: It was just announced that Ocean Infinity has acquired a 10% interest in Kraken Robotics for $2.3 million [Canadian dollars]. Kraken offers sonar sensing systems for AUVs and tow fish.

  274. Ge Rijn says:

    [Abrasive comment deleted.]

  275. Barry Carlson says:

    @All: Ocean Infinity’s interest in Kraken Robotics is for a very good reason; Kraken’s Synthetic Aperture Sonar (SAS) AquaPix MINSAS is the new top-end replacement for sidescan sonar systems.

    The resolution available is now down to 3cm2 – check the specs in the above link.

    Don Thompson believes OI are likely using this equipment in their current gas field survey near the Pluto platform on the North West Shelf offshore NW of Dampier, West Australia.

  276. DennisW says:

    @VictorI

    @All: It was just announced that Ocean Infinity has acquired a 10% interest in Kraken Robotics for $2.3 million. Kraken offers sonar sensing systems for AUVs and tow fish.

    Just to be clear the $2.3M was Canadian dollars. Current exchange rate is 1C$ = 0.75$US

    At the time of purchase OI paid a 33% premium to Kraken’s market cap stock price.

  277. Victor Iannello says:

    @DennisW; Thanks for pointing out the prices were in Canadian dollars.

  278. mash says:

    http://youtu.be/Gt_F0tzjsI4 (reference)

    re: Human Interference vs Machine Interference

    “HCM queried about MH370 again, stating that radar contact was established over IGARI but there was no verbal contact. Ho Chi Minh advised that the observed radar blip disappeared at waypoint BITOD.” (circa @25:29 – youtube above)

    Does it mean that Capt Zaharie did select the “120.9” HCM radio frequency, flick the switch, and attempt to establish “verbal contact”. But something spectacular happened in that short moment between “radar contact” and “verbal contact”.

    If so, is it reasonable to suggest that that switch switched everything from order to chaos? Or even that it was not “human interference” but “machine interference” …

  279. lkr says:

    @mash

    No.

  280. airlandseaman says:

    Gysbreght and Tim: The KB and Bu civil PSR data indicate a hand flown path climbing from FL350 (~37200 ft) at IGARI to ~43000 ft by 17:37 at KB, remain at ~43000 ft and GS=530 KTS

  281. Gerald says:

    Taking a T7 to FL 430 and keeping her at that level handflown is probably a job you have to be concentrated on, right? So the pilot(s) didn’t want to be disturbed. Passengers and crew should be dead or unable to intervene. Maybe pilot(s) told them about an emergency. Overflying Malaysia without any cellphone call except Hamid is also very strange to me.
    There were so many strange things happening on this flight, even when you found out about the flight path over Malaysia, it is not getting better. Thanks anyway for all the work!

  282. mash says:

    Gerald says: “Overflying Malaysia without any cellphone call except Hamid is also very strange to me.”

    This is not too difficult to expalin if one can accept a “Machine Interference” scenario:

    1. Pilot suspected a “Remote Control” attack.
    2. Pilot told crew and passengers to turn off cellphone once again.
    3. In this life and death situation, no one dared to turn on cellphone. Everyone guarded against each other.
    4. Hamid’s decision to try making a call was a trade-off between risk and return; besides, he was in the cockpit and he probably had discussed that with Capt Zaharie.

  283. Scott says:

    After Ocean Infinity concluded their search, former Canadian investigator Larry Vance stated on a radio call in show that after the pilot climbed to FL450, he remained there for 30 minutes, descend to FL380, veered southward along the 7th arc, and descended to FL125 into breathable air, engaged the autopilot, and left the flight deck, and began reciting prayers. The plane quickly exhausted its fuel supply at this altitude. If the plane would have remained at FL380, it would have ended up much further south, and debris would have washed up in Australia and New Zealand.

  284. Victor Iannello says:

    @Scott: Do you have a link to this interview with Larry Vance? If he really did say what you claim, he is seriously mixing fact, fallacy, and conjecture.

  285. DennisW says:

    Yeah @Scott,

    Where is your response?

    @mash

    I have put you in the whacko category.

  286. Gerald says:

    @airlandseaman: Because of your in your last post published data: Is it possible to kill crew and pax in the cabin in this period of time by any means?

  287. airlandseaman says:

    Gerald: Yes, it would take only 1-2 minutes for everyone in the main cabin to become comatose after the pressure was raised to outside ambient pressure, and everyone would be dead within 10 minutes or so after that. I am not saying this is what did happen. We simply cannot know for sure. But it is certainly the most likely theory of those that fit the data.

  288. flatpack says:

    @ALSM

    ‘raised’ you say?

  289. Victor Iannello says:

    @flatpack: Raised to the ambient pressure altitude, which is how cabin the cabin pressure is displayed and monitored.

  290. mash says:

    re: … Machine Interference Theory
    (aka Cyber Attack, Remote Takeover, Softwared System Takeover, Malfunctioning Software? etc…)

    Notice how well the so-called “little mysteries during the flight” can be explained by this theory.

    Tinkering with the power supply circuits and data buses of the subsystems are, simply, symptoms of an on-going ‘trouble-shooting’ process for self-preservation.

    However, sadly, they have apparently achieved only 2 things, namely:
    a. reactivated the SATCOM
    b. isolated the IFE

  291. DennisW says:

    @mash

    ZS was really inspired to anticipate such an intervention by his simulated flight path to the SIO.

  292. ST says:

    @Victor/Andrew – I was re-reading the transcript from the cockpit in the early part of the flight and the wikipedia on MH370.

    “At 00:46, Lumpur Radar cleared Flight 370 to flight level 350[f]—approximately 35,000 ft (10,700 m). At 01:01, Flight 370’s crew reported to Lumpur Radar that they had reached flight level 350, which they confirmed again at 01:08”

    I am not sure if you have discussed this before in a different blog but was wondering if the confirmation in 7 seconds again of reaching FL350 is common as it is possible the flight might have been at a different pressure altitude at that time and the re-confirmation was to mislead. Any thoughts as the ACARS last transmission is noted as 1:06, two seconds prior to this 1:08 confirmation.

  293. William Shea says:

    The NTSB was participating in the initial search.
    Has there been a attempt to FOI the NTSB data? Not the report, but the raw data behind the NTSB finding/
    participation?

    That should not be overly difficult. If blocked, maybe that might be a talking point.

  294. Andrew says:

    @ST

    RE: “Any thoughts as the ACARS last transmission is noted as 1:06, two seconds prior to this 1:08 confirmation.”

    The time interval between the two calls to ATC was minutes, not seconds. The first ‘maintaining FL350’ report was made at 0101:17 MYT and the second at 0107:56 MYT (ie 6 minutes 39 seconds later). The last ACARS report from the aircraft occurred 27 seconds before the second ‘maintaining FL350’ call, at 0107:29 MYT.

    Some people have labeled the second ATC call as suspicious, simply because there was no need for another report. Although it’s certainly possible that something was happening in the cockpit around that time, I don’t think the second call necessarily indicates foul play. The pilot might simply have forgotten that he had already reported maintaining FL350.

  295. Andrew says:

    @William Shea

    RE: “Has there been a attempt to FOI the NTSB data? Not the report, but the raw data behind the NTSB finding/participation?

    That should not be overly difficult. If blocked, maybe that might be a talking point.”

    I think any such request would be blocked, the same as it was in Australia. The NTSB may use a number of exemptions to deny FOI requests, including ‘Information from an investigation that is ongoing.’

    Exemptions & Filing a FOIA Appeal

  296. ST says:

    Thanks Andrew. Appreciate the inputs.

  297. Gerald says:

    @airlandseaman and all other pilots here: After @ airlandseaman’s answer to my question a few post above, dio you think it is possible to do all these things like handflying to 43000ft in a sharp curve, staying at this FL handflying, doing all the preparation to kill crew and pax and so on , alone or do your experience as pilots tell you, the workload would be too high for one person?

  298. Bradam says:

    Hi All,

    First time posting about MH 370, so am hoping I am not repeating something that has been mentioned before: has there ever been or considered an attempt to replicate the flight by Boeing or Malaysian Airlines or anyone (obviously with a modified 777 to carry more fuel to Perth after the 00:11 event)? Miight help to verify BTO and BFO data.

    I also find it intriguing that this flight also disappeared on a moonless night as well – passengers would have no visible reference point that they were flying south.

    Is there someone who has all their calculations on an excel spreadsheet so I can peruse the calculations (would save me a lot of trouble). Thanks.

  299. Victor Iannello says:

    @Bradam: Welcome.

    One of the challenges in replicating the BTO and BFO data is that the satellite orbit has changed over time, as the I3F1 IOR satellite nears it end-of-life. The amount of inclination, which is increasing over time, will change the BFO values. The methodology used by official investigators was to validate the BTO and BFO models with data obtained prior to MH370, and then apply those models to reconstructed flights for MH370.

    That said, I think there are opportunities to gain additional knowledge with some limited testing. For instance, I don’t think we have any data from an in-flight power down / power off dwell / power up sequence for the SATCOM. For the log-on at 18:25, the duration of the power off was about an hour. For the log-on at 00:19, the duration of the power off was about one minute. Both cases, as well as intermediate power off durations, could be studied. These tests would be relatively inexpensive and could be conducted in a safe manner.

  300. TBill says:

    @Bradam
    Not sure what spreadsheet you are looking for, but DrB has a lot of his work well documented. At the end of the paper attached you will find a link to DrB’s shard directory of MH370 apers. The CBTO CBFO paper is a calculator for the BTO and BFO.

  301. ventus45 says:

    @Victor

    Re testing.

    The first step is to get representative equipment to test.

    The precise configuration of the satcom fit in 9M-MRO on the night should be able to be determined from manufacturing and maintenance records, down to individual component part number and serial number.

    If a determined effort was made to track down and obtain all “sister units” within +/- 5 serial numbers of each and every module, including the antennas, then “ten ship sets” could be built up, and rigorously tested.

  302. Richard Godfrey says:

    @Bradam

    You asked “Is there someone who has all their calculations on an excel spreadsheet so I can peruse the calculations (would save me a lot of trouble). Thanks.”

    Here is a sample of excel spreadsheets using a bearing between 168° and 185° after the final major turn southwards:

    https://www.dropbox.com/s/cf5tw90hdvvnbcg/MH370%20Flight%20Path%20Model%20V17.0%20168.xlsx?dl=0

    https://www.dropbox.com/s/k3rt3nuzi1j4004/MH370%20Flight%20Path%20Model%20V17.0%20169.xlsx?dl=0

    https://www.dropbox.com/s/inltq6c3amr2uub/MH370%20Flight%20Path%20Model%20V17.0%20170.xlsx?dl=0

    https://www.dropbox.com/s/xvljd0gukej351h/MH370%20Flight%20Path%20Model%20V17.0%20171.xlsx?dl=0

    https://www.dropbox.com/s/ip4mu6kda3qx4xc/MH370%20Flight%20Path%20Model%20V17.0%20172.xlsx?dl=0

    https://www.dropbox.com/s/qi7kqn0l4s6wqug/MH370%20Flight%20Path%20Model%20V17.0%20173.xlsx?dl=0

    https://www.dropbox.com/s/9yc6w1gkpo5bpye/MH370%20Flight%20Path%20Model%20V17.0%20174.xlsx?dl=0

    https://www.dropbox.com/s/0oyw0jdkrb1lrb8/MH370%20Flight%20Path%20Model%20V17.0%20175.xlsx?dl=0

    https://www.dropbox.com/s/rybzvugxmjufn85/MH370%20Flight%20Path%20Model%20V17.0%20176.xlsx?dl=0

    https://www.dropbox.com/s/r9kie8cs6p61ki2/MH370%20Flight%20Path%20Model%20V17.0%20177.xlsx?dl=0

    https://www.dropbox.com/s/qzvk30z5lk51gat/MH370%20Flight%20Path%20Model%20V17.0%20178.xlsx?dl=0

    https://www.dropbox.com/s/0l5yz3zcawm24mj/MH370%20Flight%20Path%20Model%20V17.0%20179.xlsx?dl=0

    https://www.dropbox.com/s/oychkr2jbetz2hw/MH370%20Flight%20Path%20Model%20V17.0%20180.xlsx?dl=0

    https://www.dropbox.com/s/yfn43l0mhfba7vf/MH370%20Flight%20Path%20Model%20V17.0%20181.xlsx?dl=0

    https://www.dropbox.com/s/81wfbe9v1u12bhu/MH370%20Flight%20Path%20Model%20V17.0%20182.xlsx?dl=0

    https://www.dropbox.com/s/knrp9byotszu855/MH370%20Flight%20Path%20Model%20V17.0%20183.xlsx?dl=0

    https://www.dropbox.com/s/ud3xtgz4tbjm0m8/MH370%20Flight%20Path%20Model%20V17.0%20184.xlsx?dl=0

    https://www.dropbox.com/s/ptfn1p5xr2wce1r/MH370%20Flight%20Path%20Model%20V17.0%20185.xlsx?dl=0

  303. Richard Godfrey says:

    @TBill

    “It is frustrating that everyone who develops a nice MH370 flight path model then proceeds to run their model based on their personal assumptions, generally assuming passive pilot flight.”

    Bollocks!

    I have run my flight model over 700 times from every possible start point, for every possible air speed and every possible bearing. How did my personal assumptions dictate the outcome?

    Do you even know what the outcome was?

    Do you even know what my personal assumptions are?

  304. TBill says:

    @Richard
    I was not directing that comment towards you. But more overall automated models like the Bayesian approach, which search for a “hot spot”. In general I do feel passive paths have gotten most of the attention. I know you have considered several non-passive paths in your work.

  305. Victor Iannello says:

    @TBill: What methodology would you use to select among all the possible actively controlled flights that satisfy the satellite data?

  306. mash says:

    re: … Glideology 100

    reference https://www.atsb.gov.au/media/5747317/ae2014054_mh370-definition_of_underwater_search_areas_3dec2015_update.pdf

    Homework exercise (rudimentary model only)

    Aim: To attract further seach (attractive areas found!?)
    Method: Change search assumption from Dive to Glide** (rationale – implication of previous search result, etc…)
    Result:
    (just example figures only; but guess that band characteristics should be the same in general)

    band 1 – 13.3%
    band 2 – 13.3%
    band 3 – 13.3%
    band 4 – 20.0%
    band 5 – 40.0%

    And after not found in band 1 and 2, probablity increased to:

    band 3 – 18%
    band 4 – 27%
    band 5 – 54%

    Glossary:
    Glide** – max range vector (straight line) glide in any direction, 100 NM in this example.
    Start of Glide line – an infinitely long line, to avoid end of line complications at this stage.
    band 1 – 0 to 20 NM from line
    band 2 – 20 to 40
    band 3 – 40 to 60
    band 4 – 60 to 80
    band 5 – 80 to 100

    Sine table:
    sin 11.54′ = 0.2
    sin 23.57 = 0.4
    sin 36.87 = 0.6
    sin 53.13 = 0.8
    sin 90 = 1

  307. Richard Godfrey says:

    @TBill

    It is a shame that you had not directed your comment toward me. I had arrogantly thought that your category of a “nice” MH370 flight path model had included mine. ;-))

  308. TBill says:

    @Victor
    “What methodology would you use to select among all the possible actively controlled flights that satisfy the satellite data?”

    I do not have a rigorous method, but I think about landing zone targets a pilot might have been thinking about, then I look at flying conditions and events that a pilot might have reacted to (wind, clouds, sunrise, sat calls, etc).

    Also I am generally thinking there was a straight flight period to about Arc5, but I like your new proposal with a turn even a little before Arc5.

    @Richard
    Yours and Victor’s flight path work is very important to me and appreciated very much.

  309. Bradam says:

    Thanks for the welcome and thanks TBill and Richard Godfrey for those links. Clearly I have a lot of reading and catching up to do.

    I came to this site as it is active in discussion and there seems to be a lot of great minds giving input. I do not profess to be in any position to provide any additional, or better answers or ideas – but I do have questions (which I am sure have been discussed in the past), so I hope I don’t open any old wounds or repeat too much stuff that has gone before. If so, could someone just simply provide the consensus of the group as to the outcome of the following queries I have. I have over the past 4 years made the assumption that the teams leading the investigation have the greatest minds and knowledge and all the facts available to them, and that MH370 would eventually be found. That fact that it hasn’t been found is indicative that some vital piece of information remains elusive.

    1. Given that the satellite is approaching end of life and that it is now producing an oval-like rotation with respect to its previous geostationary position, then would it not also be correct to conclude that the satellite is precessing as it rotates around its own axis as well as the ‘z’ axis of Earth? And would this be relevant in some way?

    2. Glide v Plummet. There seems to be a great deal of continuing discussion as to whether the plane glided or plummeted to the sea after ‘flameout’. Is it true that a B-777 in autopilot would glide after flameout? Or would the autopilot attempt to maintain S & L, eventually resulting in a stall and thus plummet into the sea? Surely Boeing would be able to provide such an answer or have conducted test-work to determine this point of contention.

    All for now …

    Cheers,

    Bradam

  310. Victor Iannello says:

    @TBill: Do you really think it is possible to devise a sensible search plan based on possible maneuvers after 19:41 caused by the pilot’s reacting to external factors and thoughts along the way? To cover all the possibilities, you might as well commit to searching the 7th arc all the way to Java.

    You’ve been frustrated that more attention hasn’t been devoted to routes with pilot inputs. The problem is that without some constraints on the reconstructed flights (such as passive flight after 19:41), the search area grows to an unmanageable size. Or, you can impose scenario-specific constraints, such as a failed attempt to land on Christmas Island.

  311. Victor Iannello says:

    @Bradam: Two quick answers.

    1) The satellite orbit degrades from a geostationary orbit due to second-order effects, such as the gravitational forces of the sun and moon. To limit the inclination, “station-keeping” corrections periodically occur. But all of this doesn’t matter because we know what the orbit was for I3F1 quite accurately during the MH370 flight.

    2) Here’s my take. (Others may disagree.) After the flameout of the second engine, electrical power is interrupted to the AC transfer busses, power is interrupted to the pilot heaters, and the flight control mode degrades from NORMAL to SECONDARY. This causes the A/P to disengage. In this mode, the only lateral stability is passive stability from the dihedral effect, which tends to level the wings. However, if the rudder is not almost perfectly in trim, the yaw and resulting bank will exceed the dihedral effect, and the plane will eventually enter a banked descent. External forces from gusts and turbulence will also disrupt the glide path.

  312. Shadynuk says:

    @Victor “The problem is that without some constraints on the reconstructed flights (such as passive flight after 19:41), the search area grows to an unmanageable size.”

    Exactly! And if the intent was to hide the aircraft, don’t you think that ZS would have realized that and conducted the flight accordingly?

  313. Victor Iannello says:

    @Shadynuk: Depending on how much knowledge you attribute to the pilot about how the satellite data might be used to reconstruct candidate flight paths, there is the small possibility that there was an intent to mislead searchers by following an erratic flight path and even faking a premature fuel exhaustion by cycling the SATCOM power and descending rapidly. If this occurred, there is practically no hope of finding the plane. On the other hand, I doubt the pilot had any clue about the satellite data and how flight paths might be reconstructed, and how to use that knowledge to mislead searchers.

  314. Shadynuk says:

    @Victor Looking again at my comment above, it seems this may be taken as an unfair statement or question to you. That was not the intent. You are clearly fully cognizant of the circumstances.

    It does seems a difficult task at this juncture to define a reasonable search area with enough convincing support evidence to attract another search expenditure.

    I continue to read and hope one of you smart guys will come up with something or get a lucky break!

  315. TBill says:

    @Victor
    You and I both tend to agree with George Bibel who said MH370 appears to be a criminal act of some kind. Therefore we may need FBI-style techniques not NTSB/ATSB techniques to assess flight paths/motives. I am only guessing at how FBI might rank the goals of a rouge flight.

    I just strikes me we could fairly easily make up several intentional flight paths to 12-25S. You have suggested a semi-active path above. Jean-Luc has one. I am suggesting possibly to Dordrecht Hole with direction change to Zenith at 23:14 sat call. Try to pick a set of 5 best active cases and work them through to what an active pilot did to meet BTO/BFO all the way to impact. Maybe the study tells us it was passive flight, I dunno the outcome.

    Give it a chance is all I am saying. I suggest let’s give 50/50 emphasis to passive vs. active flights.

  316. TBill says:

    rogue

  317. Victor Iannello says:

    @TBill: First, the YPCC path ending at 22S latitude requires no pilot input from before 19:41. I’m not sure why you refer to it as “semi-active”.

    I don’t stop people from suggesting active flight paths. I only point out that the probability of choosing the correct one out of the many possibilities is small. For that reason, I advocate progressive searches over a range of latitudes. Now, if you advocate searching from 12S to 25S because there exist “unconstrained” reconstructed flight paths satisfying the BTO and BFO data, the next questions are how wide to search, and why is this better than searching wider at previously searched latitudes? We also can’t forget that it is unlikely that a path with multiple maneuvers after 19:41 would replicate the BTO and BFO signatures of flights with no maneuvers after 19:41.

    I falsely get accused of not wanting to consider the possibility that there were pilot inputs after 19:41. A more accurate statement is that I don’t know how to deal with this possibility in recommending a search area.

  318. TBill says:

    @victor
    Active cases look more like what Jean-Luc is trying to communicate, we can use BTO BFO to tell us what the pilot did. So I think it basically assumes BTO BFO are quite accurate.

    But I too would be open to suggestions. More like orig ATSB approach assuming Arc2 is correct and moving forward from there.

    If we could re-do the Bayesian study, I do not have any problem with the assumptions they used, except that is just one of many cases. I would do another case where FMT was after 18:40 and allowed maneuvers after Arc5 etc. Check out various sets of assumptions.

  319. Victor Iannello says:

    @TBill: If the DSTG study was repeated ignoring the BFO value at 18:40 and allowing the possibility of a hold, the results would be very similar to the calculations I did for the great circle paths. If you further remove constraints by assuming the probability of a path is independent of the number of maneuvers, then I suspect the posterior distribution would be fairly flat over a large range of latitudes. Without some other assumption, such as the pilot was trying to reach Christmas Island, or the pilot was aiming for a particular location in the ocean, the search area becomes very large.

    Without additional forensic evidence that prioritizes some scenarios over others, I just don’t see a way to limit the search area for actively controlled flights with an arbitrary number of maneuvers. We are left with following “hunches”, and the probability of finding the debris would be a function of the accuracy of the hunch.

  320. @Victor
    @TBill

    If you add the fuel constraint to be exhausted at Arc7 you will reduce by far the number of possibilities in addition to fly over waypoints. We have done quite a lot of simulations at high FL. We could not reach Australia and were always short of fuel with the simulator (which we found always optimistic by the way:-) and with our Constraint Assessment Tool and always overshooting Arc7. And the timing at the Arcs was basically impossible to match without changing the speed which is not making sense… A descent was justified by the BFOs, the airspace structure and the timing between the arcs… and during this the speed had to be managed only by the aircraft itself to get the best performance at any time.

    Arc2 does not appear as a real issue: because of the 58microsec uncertainty, Arc2 is in fact a curved stripe through which the a/c flew almost tangentially. As the distance between the “entry” point of this tangent into the stripe and the “exit” point is potentially larger than 120 NM, there is no real constraint as it can be almost “anywhere” on this segment.

  321. @all

    To help grasping what is the rationale in full words for a piloted trajectory (50/50 as says TBill :-), we have produced a document including answers to the questions received (from you and others) about CAPTIO MH370 trajectory because we think that some important aspects of the analysis were overlooked.
    This document provides more substance and also additional information justifying why the End of Flight location near Christmas Island should be searched.

    http://mh370-captio.net/wp-content/uploads/2018/06/Questions-Answers-1d.pdf

  322. Victor Iannello says:

    @Jean-Luc Marchand: If you are implying that only your Christmas Island route satisfies the constraint of fuel exhaustion at the 7th arc, I would disagree.

  323. @Victor

    I agree with you, other trajectories might exist, so your mission, if your accept it Mr Phelps… 🙂

    I am just saying that in spite of our efforts, we could not find another concurrent one that fits all the constraints.

    We started be analysing trajectories in the same vein as yours i.e. at high FL and going south and turning wide in the south of Cocos towards Perth (and up to Learmonth)… we gradually converged to this trajectory, at lower FLs to Xmas, as it fits all the constraints: timing at the arcs with BFOs accordance, navigation using known, clear waypoints, airspace structure explaining the need for descent, fuel consumption, automatic flying mode selection by the aircraft itself including full automatic speed management by the aircraft and also the matching with the intermediate BFOs (those without corresponding BTOs), reachable traget with adequate runway…

  324. flatpack says:

    @TBill

    “… MH370 appears to be a criminal act of some kind. Therefore we may need FBI-style techniques not NTSB/ATSB techniques to assess flight paths/motives. I am only guessing at how FBI might rank the goals of a rouge flight.”

    The flight went rouge(sic) when the pijacker burned off fuel! (maybe as seen by Kate Tee)

    We can make educated guesses such as this one of mine re FIR boundary:-

    “Most accept that the early flight along a FIR boundary was deliberately done to reduce radar operator ‘attention’. So, actively reducing eyewitness attention and/or probability of identification should be considered in my view.” such as ‘contrail management’.

    My overarching guess is that the overall ‘plight plan’ was to simulate a wounded bird. Thus any primary radar operater that actually noticed would perceive an unresponsive aircraft possibly heading towards the next available diversion airstrip.

    Each national operator would only see part of the overall picture. Their individual segment would make sense and be essentially unthreatening. Only in hindsight, with all available returns plotted, would repetitively lining up for a possible emergency landing look deeply deceptive.

  325. Victor Iannello says:

    @Jean-Luc Marchand: I won’t repeat why I think your scenario is not the most probable. That said, I don’t completely dismiss it. I appreciate all your hard work, as well as your willingness to civilly discuss various aspects.

    Of course, if you want your proposed area searched, I am not the person you ultimately need to persuade.

  326. flatpack says:

    @TBill

    “Therefore we may need FBI-style techniques not NTSB/ATSB techniques to assess flight paths/motives”

    For the assessment of paths (rather than motives) there may be another set of techniques available, that is those used in air defence.

  327. TBill says:

    @Jean-Luc
    Very nice write-up on the Questions and Answers. Re: A3 I was going to ask “why” Christmas Island, but D20 later gives that rationale (proximity to Arc7).

    So if the goal was instead crashing at sea somewhere to hide the crash, I assume that opens up more possible landing zones.

  328. David says:

    @HB, Ge Rijn. In case of interest I have tidied-up and embellished what I posted earlier about Larry Vance’s book, and added some notes to my own interpretation of the evidence.
    Amended version:
    https://www.dropbox.com/s/ilpmwzem3q7lmb9/PERUSAL%20OF%20THE%20BOOK%20BY%20LARRY%20VANCE.docx?dl=0

  329. Bradam says:

    @Victor

    “1) The satellite orbit degrades from a geostationary orbit due to second-order effects, such as the gravitational forces of the sun and moon. To limit the inclination, “station-keeping” corrections periodically occur. But all of this doesn’t matter because we know what the orbit was for I3F1 quite accurately during the MH370 flight.”

    But does the precession of I3F1 affect the data, particularly the BTO?

  330. Bradam says:

    Hi All,

    What is the consensus of the group as to the failure of the search team to find the wreckage? Is it due to the size of the area, the type of equipment used, or that they were searching in the wrong location?

    If the search team were searching in the wrong location, then is that because there is an error in the calculations (formulas) or an error in the data (BTo or BFO)?

  331. DennisW says:

    @Bradam

    As Victor stated we know the position and velocity of the satellite at any given time during the flight, and our models account for it.

  332. Victor Iannello says:

    @Bradam: The most likely explanations for the failed search are either the debris field is along the 7th arc but further north, or the debris field is farther from the 7th arc than what was previously searched. Opinions differ as to which explanation is more likely.

  333. Niels says:

    @VictorI
    The third option, which imo is not much more unlikely as the other two, is that the wreckage somehow has been missed in searched area. All three options fell a priori in the low probability category.

  334. Victor Iannello says:

    @Niels: Yes, in my comment to @Bradam I did neglect to mention the probability that the debris field was scanned and missed (listed as possibility #5 in my post above). I don’t view that nearly as probable as the other two possibilities, although I do agree that we are now considering impact locations that were previously considered to be low probability, so many possibilities remain on the table.

  335. lkr says:

    @Niels: Good point, and yes, the search seems to have ended with three not-so-good possibilities. I wonder about the representation of false-negatives in the 5% or under range. [Meaning that on the face of it, the possibility of debris fields in the area searched by Fugro and OI is much lower than that limit]. If I remember correctly, the stated capability to identify debris fields was represented as near-100%, so that false-negative was computed just for tracking errors by the submersibles [towed and AUV, respectively].

    Is that correct, and does anyone know to what degree the stated capability is theory-laden, eg, based on fragment-size and distributions from modelled or actual ocean impacts? Would debris detection be compromised by any reasonable scenario of impact, from Sully-style ditch and sink to Germanwings full-on oblique impact, or powered dive? I recall discussion of worst-case examples, eg, nothing bigger than an engine core. Is that what Fugro and OI used for their detection capability?

    I take it from Victor, that there’s a live possibility of OI self-funding a completion to the north. But hard to imagine anyone attempting a wider-and-wider lateral search without new evidence narrowing it latitudinally [leaks from Indonesian radar or Malaysian govt come to mind]. And specific-scenario foci like Jean-Luc and the infamous Rob-at-daybreak might eventually attract a wild-catter like Bill Gates. Otherwise, surely, someone will audit the data already in hand. IMO, it wouldn’t be the strangest thing in the world if debris field was there all along, missed by machine and human eyes..

  336. lkr says:

    @Niels: Good point, and yes, the search seems to have ended with three not-so-good possibilities. I wonder about the representation of false-negatives in the 5% or under range. [Meaning that on the face of it, the possibility of debris fields in the area searched by Fugro and OI is much lower than that limit]. If I remember correctly, the stated capability to identify debris fields was represented as near-100%, so that false-negative was computed just for tracking errors by the submersibles [towed and AUV, respectively].

    Is that correct, and does anyone know to what degree the stated capability is theory-laden, eg, based on fragment-size and distributions from modelled or actual ocean impacts? Would debris detection be compromised by any reasonable scenario of impact, from Sully-style ditch and sink to Germanwings full-on oblique impact, or powered dive? I recall discussion of worst-case examples, eg, nothing bigger than an engine core. Is that what Fugro and OI used for their detection capability?

    I take it from Victor, that there’s a live possibility of OI self-funding a completion to the north. But hard to imagine anyone attempting a wider-and-wider lateral search without new evidence narrowing it latitudinally [leaks from Indonesian radar or Malaysian govt come to mind]. And specific-scenario foci like Jean-Luc and the infamous Rob-at-daybreak might eventually attract a wild-catter like Bill Gates. Otherwise, surely, someone will audit the data already in hand. IMO, it wouldn’t be the strangest thing in the world if debris field was there all along, missed by machine and human eyes..

  337. Victor Iannello says:

    @lkr: I won’t speak for OI, but based on public comments, there seems to be a willingness to search again if an area of reasonable size can be justified. In my opinion, that has not yet occurred.

  338. ST says:

    I was thinking yesterday about when we could expect the Phase 2 of the Australian geo science sea floor mapping would be released to public and found in a strange coincidence that it was released today…

    http://www.ga.gov.au/about/projects/marine/mh370-data-release

  339. PaxLambda says:

    About the release: Very interesting!

    But there are some strange data (errors?):
    On this page : https://geoscience-au.maps.arcgis.com/apps/Cascade/index.html?appid=038a72439bfa4d28b3dde81cc6ff3214, in the debris section (about 5/6e of the page), if you click the yellow point just above the A of “SWAZILAND”, you get a window saying “Left Wing Trailing Edge Panel. Debris piece was found at Macenta Peninsular, Mozambique”. But the picture name is “https://d3q3olqy33wgar.cloudfront.net/Item9_RiakeBeach.jpg” (Riake beach is the place where BG found many debris in Madagascar east coast).
    The link to Malaysia about those debris (http://www.mot.gov.my/en/aviation/air-incident-investigation/MH370) seems invalid.

    Another remark: many of these debris are “Unidentified Part”: Nobody at Boeing may have an idea from where they are coming from? At least interior/exterior?

    Thank you for maintaining this blog!

  340. Niels says:

    @lkr
    Figure 73 and the discussion around it in the 3 Oct 2017 ATSB report may be helpful. Data holidays are attributed to equipment failure, terrain avoidance, shadow zones and off-tracks. Looking at the figure shadow zones seem to occur quite frequent, however their total area is limited: The total overall data gap occurrence is in the order of 1%.
    Note also the rather large “lower confidence coverage” area in the “orange” region. This region includes the “high probability” latitudes S35 – S36 as indicated by CSIRO.

    It is mentioned at p. 96 that “filtering out data gaps and LPD areas less than 200 x 200 m decreases the individual data gap count by 78 percent, and LPD count by 60 percent.”….”This highlights that many of the data gaps and LPDs are smaller than 200 m by 200 m”….
    ….”Historic analysis has shown that aircraft debris fields typically cover areas larger than 200 m by 200 m.”

    Finally it says on p. 96/97:
    …”Therefore, areas less than 200 m by 200 m can be discounted with a moderate to high degree of confidence as they are not large enough to fully contain the aircraft debris field. However, this reduction only results in a small overall coverage increase of 0.1 per cent (data gaps) and 0.2 per cent (LPDs) respectively, hence the decision was made to maintain a conservative approach by using the 100 m by 100 m area metric.”

    This latter “However,..” I do not fully understand (at least not in the light of the previous paragraph). It seems to suggest that although the total number fraction of smaller data gaps is large they do not considerably contribute to the total data gap area. It would be interesting to see the total numbers and area histogram / distribution.

  341. ST says:

    @PaxLambda- The Phase 2 data was referred several times as challenging to consolidate and put together as it contains a large number of data sets from the side sonar scans. There are probably some errors in uploading information as several folks have worked on this together to present the data in a way that can be useful to scientists/researchers and public who are interested.

    There are certain very interesting aspects to this data that for sure will be put to good use by the scientific community. It will probably go through some updates and corrections but we have to be thankful to have this kind of access to deep sea research from our couch!

  342. sk999 says:

    Here’s a thought exercise, which may or may not be of any value. The following link is a plot showing the BTO and BFO values that would have been generated during the turnback, from the last ACARS point (17:06:43) to the last radar point (18:22:12). The flight path is the one reconstructed from Fig. 4.2 of Bayesian Methods, so it won’t be accurate in detail, but the major trends are evident and real. The BFO calculations assume no change in altitude.

    https://drive.google.com/file/d/1dPs8wQRuLpSBvN5-qqJjeeZMtPdATuSQ/view

    The black points are the BTO, and the blue points are the BFO. Obviously, if one were to sample these curves at an hourly interval, you would only have 2 points, and it would be impossible to determine what the plane was doing with any degree of reliability. Suppose one were to sample the curves more frequently so as to end up with 7 points, spaced similarly to the 7 arcs. How well could one reconstruct the flight path? In particular, supposed one used the flight dynamics model of Davey, Gordon, Holland, Rutten, and Williams. What would that model predict?

  343. @Niels,

    You Quote ATSB, 3 Oct, 2017, as saying, .”Historic analysis has shown that aircraft debris fields typically cover areas larger than 200 m by 200 m.”

    In case you missed it, Fig 35 shows and describes the AF 447 debris field as being 600 x 200m, however, it looks like 90% of the debris is in an ~400 x 100m area.

    In case it helps.

  344. DennisW says:

    @sk999

    Why would you simply not integrate the velocity values in Figure 4.2 along the track in Figure 4.2? Converting to BFO and BTO and then converting back to velocity and track makes no sense to me.

  345. DennisW says:

    @sk999

    Sloppy of me. “Velocity” above should be “speed”.

  346. lkr says:

    @VI, @Niels: Thanks for input — I’m not sure that I’m framing my original question very effectly — I’d agree that the small data holidays as such wouldn’t credibly account for missing the debris field. My thought is whether a debris field at either the highest speed conceivable or at the ditching end would defeat the search image they use for debris. [In particular, what a debris field would look like at very high impact speed for varying angles of incidence. Further, I’m curious how much of the highlighting data for human inspection is automated — were all of the tens of thousands of hours of data acquisition viewed? Whether machine or human-eye filtering — I wonder if a future generation of data analysis might find the debris hiding in plain sight? Not an expert, but it would seem to have more bang for bucks than ever-widening circles at the 7th arc.

  347. Victor Iannello says:

    @lkr: I believe they are searching for a debris field as opposed to individual parts. The debris field should be >200m long and discernable even if the parts are small, as the acoustic “hardness” of the metallic parts should be in contrast with the seabed. As far as the review of the data, I believe that today, multiple viewing by humans provides the best probability of detection. Perhaps at some point the data could be again be reviewed using improved pattern recognition software. I agree that automated review of the data could be a cost effective way to proceed.

  348. mash says:

    re: Constructive Interference

    ref:
    https://www.atsb.gov.au/media/5747317/ae2014054_mh370-definition_of_underwater_search_areas_3dec2015_update.pdf
    (page 9 – Reponse of aircraft systems to engine flame-outs)

    Assuming aircraft “un-controllable” after a major “system failure”, the above “system response” could perhaps un-intentionally release some other subsystems (say “cockpit input units”) …

    Thus explaining a surprising “potential controlled glide” after that initial “rapid descent”?

  349. Victor Iannello says:

    @mash: I don’t fully understand your comment, but those Boeing simulations provide evidence that with no pilot input, the crash should have been close to the 7th arc. The fact that the plane was not found suggests to me that likely the plane was actively controlled in an extended glide, or the plane crashed further up the 7th arc.

  350. mash says:

    @Victor Iannello

    Proposed event sequence:

    1. Plane was uncontrollable before engine flame-outs.
    2. Plane became controllable after engine flame-outs (as a result of “system response” and/or electric power restart).
    3. Pilot discovered plane controllable again (say from cockpit output unit signals).
    4. Pilot regained control of plane just in time (circa SATCOM logon).
    5. Pilot performed “controlled glide” afterwards …

  351. Victor Iannello says:

    @mash: Why would a plane continue to fly until 00:17 and then become uncontrollable before the engine flameouts?

  352. Richard Godfrey says:

    Since the search by Ocean Infinity terminated without success, there have been a number of ideas as to why MH370 has not been found.

    Some of these hypotheses are wild, to say the least.

    In my view, the reason that MH370 is not that complicated.

  353. Richard Godfrey says:

    Correction: In my view, the reason that MH370 has not been found, is not that complicated.

  354. Victor Iannello says:

    @Richard Godfrey: So that we all understand your thoughts, can you please explain what you think is the most likely reason for why MH370 has not been found?

  355. mash says:

    @Victor Iannello

    No, the plane had been uncontrollable (by the pilot) since the “system takeover” or “system faliure”.

    Or, more simply and wildly, could the statement (after engine flameouts – page 9 above)

    “Half of the 6 cockpit display units would be lost, with the Captain’s PFD, Captain’s ND and the upper EICAS display remaining powered”

    be changed to the statement (after “system failure”)

    [All of the N cockpit ‘input units’ would be lost … ]?

  356. Richard Godfrey says:

    @Victor: My apologies that I was vague.

    In my view, the plane is either north of 25°S up to 20°S or outside the ±22 NM search width up to ±40 NM. The 7th Arc has been correctly defined. The plane simply ran out of fuel.

    I do not believe that:
    1. The plane is outside the 40 NM search width.
    2. Was subject of a long glide.
    3. The debris field has been missed.
    4. The BTO data set is wrong or incorrectly interpreted.
    5. The BTO and/or BFO data has been spoofed.
    6. The plane is south of 39°S.
    7. The plane is further north than 20°S.
    8. The plane permanently or intermittently malfunctioned.
    9. The plane had a fire.

  357. David says:

    @Richard Godfrey. The First Principles Review Report of 20th December, 2016 at p.17 listed reasons for its participants’ conclusion that, “… the distance required to be searched from the arc could be reduced to 25 NM from the 7th arc…”.

    For a new search to the old 40 NM to be warranted would suggest that the (then) new information which led to that reduction should be set aside now, or some of it, or its relevance adjusted.

    Can you be more specific as to why you think it might be outside 25 NM yet within 40?

  358. TBill says:

    @Richard
    It is significant that you are allowing up to 20 South with respect to drift studies.

    I tend to agree for due diligence, they should check up to 20S, and maybe check Xmas Island (if that is not too time consuming). Right now I am not too hopeful. We must now consider the worst case scenario of a “false” fuel exhaustion event that left enough fuel to go maybe another 250-nm.

  359. DennisW says:

    @TBill

    We must now consider the worst case scenario of a “false” fuel exhaustion event that left enough fuel to go maybe another 250-nm.

    I am not going to consider that.

  360. Richard Godfrey says:

    @David

    You asked “Can you be more specific as to why you think it might be outside 25 NM yet within 40?”

    Actually I said that the MH370 might be outside the search width of ±22 NM. The search width of ± 22 NM was a compromise made by Ocean Infinity to allow the search to be extended further north to 25°S. This was the high probability area.

    Since MH370 was not found in this high probability area, we should first consider going further north to 20°S and then consider going further out from the 7th Arc. 40 NM represents an absolute limit for me. We should consider extending the search width in stages, for example, first to ±28 NM, then to ±34 NM and finally to ±40 NM. I agree, that each time we extend the search width in such a staged approach, the chances of finding MH370 reduce. It was just a case of being thorough.

    I also agree that MH370 is most likely to be found within 25 NM of the 7th Arc, but there maybe some factor that we have not considered that places the MH370 end point just outside 25 NM.

  361. Richard Godfrey says:

    @TBill

    You stated “It is significant that you are allowing up to 20 South with respect to drift studies.”

    I am in the process of redoing the drift analysis. The timing of floating debris reaching the shores of Africa does not appear to fit start points north of 20°S. As has been noted by many analysts, floating debris arrives far too soon.

    However, jus as there is a hot spot that can fit the timing of the Flaperon arriving in Reunion and other debris arriving in Madagascar and mainland Africa around 30°S, there is another hot spot approaching 20°S. This would allow scenarios, such as presented by Victor in the current post to fit the drift analysis.

    More details to follow, when I finish the new drift analysis. Unfortunately, it will still take a few weeks to complete.

  362. David says:

    @Richard Godfrey. Thanks Richard. Yes I recall the adjustment to 22 NM in OI’s last sector. If the outer extension limit is in the eye of the beholder, that beholder needs also to weigh what priority should be given to extending from 22 NM, vs the 25 NM area, or even extending the earlier 40 NM, and possibly allocating preferences to outwards and inwards.

    My view has been that a search width assuming there to be a pilot would be narrower than without if the final descent rate is valid, leaving the logic for extending the width to the vagaries of flight subsequent to the ATSB/Review phugoid possibility. However those are compounded by unknown variations which might arise from relight (including such as the hung relights of the ALSM simulations) and APU start, if any, effects on trajectory, again if any.

    You say,”…there maybe some factor that we have not considered..”. I have suggested that OI could get together directly with Boeing to see to filling some gaps in information, along with more research into the SDU and the systems that integrates with at reboot. I think without a lead from that work the case to extend width would be too weak.

    Consistent with your priorities an extension along the arc looks more promising, particularly should you find a drift anomaly which would help with timing.

  363. mash says:

    re: Input Technology (An Analogy)

    Intermission – a fun experiment

    Equipment req: An old-fashioned Nokia mobile phone (text mode)

    Preparation: Turn “Automatic Keyguard” on. Keypad locked after say 10 secs.

    Question: How to unlock keypad?

    Ans 1: Standard way. Press “Unlock” followed by “*” immediately.

    Ans 2: Ask someone else to call you through other means (if you are not alone in a desert); when the phone is ringing, keypad unlocked automatically.

    Ans 3: Please provide … (assuming you are alone in a desert).

    [Hint: You cannot even switch off the phone by pressing the “red” button, because the phone is not ringing and the keypad is still locked.]

  364. DennisW says:

    @Richard

    “Far too soon” is irrelevant. Good grief. Who knows when the debris is dsicovered versus when it is in the area?

  365. Richard Godfrey says:

    @Dennis

    You asked “Who knows when the debris is discovered versus when it is in the area?”

    Fortunately there are 3 items of MH370 floating debris that were discovered with barnacles attached. Barnacles die within a few days, when out of the water. Also barnacles are cleaned off by crabs and other scavengers in a couple of days.

    The locations of the Flaperon found at St. Andre Beach, Reunion after 508 days, the Engine Panel found at Klein Brak River, Mossel Bay, South Africa after 655 days and the Cabin Divider found in Var-Brûlé Beach, Rodrigues after 753 days, are the only locations where MH370 floating debris has been found with barnacles still attached. Barnacles demonstrate a relatively recent arrival.

    When Schalk Lückhoff discovered the MH370 Engine Panel, known as Roy, he reported the barnacles smelt terribly, a clear sign that they were dying off. This item of MH370 floating debris was discarded by Schalk, because of the smell. We are very lucky that the same item was found almost 3 months later on 21st March 2016 by Neels Kruger, about 100 m further inland, on the same beach at the mouth of the Klein Brak River in Mossel Bay, South Africa.

    Goose Barnacles (Lepas Anatifera Striata) have feeding tentacles called Cirri. When the Cirri are out of the water, they die off quickly. This results in feeding damage and headless stalks.

    None of the 3 items of MH370 floating debris, found with barnacles, show photographic evidence of feeding damage or headless stalks. This would suggest that the items were discovered and photographed within at least a few days of beaching, but not much longer than 5 or 6 days. Prof. Charles Griffiths, a marine biologist from South Africa, claimed to be able to see evidence of cirri projecting from some animals in the photographic evidence of the Flaperon in Reunion. It should also be noted, that there were no complaints of smell from rotting barnacles, either in Reunion or Rodrigues. The timing of MH370 floating debris arriving in Reunion after 508 days, Mossel Bay after 655 days and Rodrigues after 753 days is therefore assumed to be correct to within less than 10 days before the discovery.

  366. Victor Iannello says:

    Reuters reports that the Former Prime Minister of Malaysia, Najib Razak, was just arrested on corruption charges for embezzling money from the 1MDB fund. This comes after a previous investigation cleared Najib of any involvement, despite evidence that a billion dollars of 1MDB money was diverted from 1MDB accounts to Najib’s personal accounts.

    The former Inspector-General of Police, Khalid Abu Bakar, is himself now under investigation for misconduct in handling the 1MDB case.

    For those that are sure that the conclusions of the RMP report exonerate the captain, one only needs to observe how the same cast of characters used a criminal investigation to whitewash the guilt of Najib. Because of the political implications, a true criminal investigation looking into MH370 probably never was and probably never will be conducted.

  367. lkr says:

    @RG: Nice summary of the barnacle evidence. Presumably you can specify that debris with scars from Lepas/all of body and stalk missing as having been on shore at least 2 weeks..

    As I keep saying, based on BGs success in Madagascar a couple of years ago, there should be composite fragments on less-patrolled beaches around the IO even now. Badly weathered, of course. I’d also wager that you could rapidly rule in or out portions of the northern 7th arc by presence or absence of these fragments on specified coastlines. And as BG showed, survey by ATV on the beach is a lot less expensive than ocean floor by AUVs.

  368. ventus45 says:

    @Richard Godfrey
    What are your thoughts on the Pemba Flap and it’s barnacles ?

  369. mash says:

    Re: Further Analysis (based on presumption)

    Presumed fact:
    “rapid descent” + “controlled glide” (in time sequence)

    Necessary condition for “controlled glide”:
    1. conscious pilot
    2. controllable plane

    No necessary condition for “rapid descent” (four combinations):
    A. conscious pilot + controllable plane
    B. conscious pilot + uncontrollable plane
    C. unconcious pilot + controllable plane
    D. unconcious pilot + uncontrollable plane

    But C and D can be ruled out since a pilot cannot suddenly become conscious.
    And A can be ruled out basically – unless one still insisted that it could be a deceptive action (presumably required knowledge of Inmarset maths and SATCOM logon timings).

    So the only plausible answer is B (implicating the plane changed from uncontrollable state to controllable state).

    Conclusion:
    According to the presumed fact, the only plausible and consistent theory so far is the System Takeover/Failure Theory, aka Machine Interference Theory.

  370. flatpack says:

    @mash

    “…a pilot cannot suddenly become conscious”

    No? Upthread you suggested a ‘suddenly awaken (sic) pilot’.

    Here is my reply to you then:-

    “@mash

    My take is that the pijacker possibly entered a fugue state.

    This state may have been ‘enhanced’ by liberal use of powerful painkillers. Remember the gliding accident that left him with severe back pain?

    I don’t think the “suddenly awaken” pilot was a ‘good’ intentioned one, merely that he made some attempt to preserve his own life for religious reasons. I don’t think that any probable glide direction can be reliably inferred.”

  371. Richard Godfrey says:

    @ventus45

    You ask “What are your thoughts on the Pemba Flap and it’s barnacles ?”

    I was not able to find any photographic evidence of barnacles in the pictures of the Outboard Flap from Kojani, Pemba, Tanzania discovered 20th June 2016. The ATSB report dated 15th September 2016 also makes no mention of barnacles.

    If you have any evidence of barnacles attached to the Pemba Flap, please share it.

  372. David says:

    Those are not identified in the Operational Search Final Report of 3rd Oct 2017, Appendices F & G, which make no mention of the Pemba flap part, though
    Appendix G Table 2, p31 onwards, illustrates some creatures under general discussion about other recovered items.

  373. mash says:

    @flatpack

    So he must be conscious …

  374. Victor Iannello says:

    @mash: You have curious logic. First, your “presumed fact” that there was a rapid descent followed by a controlled guide is debatable. It’s possible that there were no pilot inputs and the plane crashed near the 7th arc (but further north than already searched). Second, many believe that the pilot first rapidly descended and then flew an extended glide. That’s not my favorite theory, but it is possible.

  375. DennisW says:

    @Richard

    The locations of the Flaperon found at St. Andre Beach, Reunion after 508 days, the Engine Panel found at Klein Brak River, Mossel Bay, South Africa after 655 days and the Cabin Divider found in Var-Brûlé Beach, Rodrigues after 753 days, are the only locations where MH370 floating debris has been found with barnacles still attached. Barnacles demonstrate a relatively recent arrival.

    Yes, relative to when the debris beached. In the case of the flaperon there are reports that people saw it in the water earlier than the beaching date. I am very suspicious of arrival times other than times that are too early for a given terminus. Probably not a big deal relative to other variables in the drift analytics.

  376. TBill says:

    @mash
    I am at your: A. conscious pilot + controllable plane

    You argue the aircraft must have been under remote control of outside agents, because it is impossible the pilot might have comprehended that satellite signals could help locate the aircraft. On the other hand, I increasingly feel the pilot might have comprehended the need for maneuvers to move away from where the satellite signals were sent.

  377. Victor Iannello says:

    Update on July 4, 2018.

    I have expanded the post above to include data for a new but similar path that assumes a speed of M0.8 at FL320 after 19:41. The BTO and BFO errors are slightly improved compared to the previous path of M0.819 at FL320. I also include the results of a fuel analysis, which predicts fuel exhaustion very near 00:17. The fuel analysis inputs and outputs are tabulated from 17:07 until 00:17 so that others can make their own comparisons.

    In the post above, I said that Ultimately, the decision where to search must consider other aspects such as end-of-flight dynamics, drift modeling, surface search efforts, and fuel consumption, none of which were considered here. Unless somebody finds a material error in the fuel model results, an impact point of 22S seems to be consistent with the loaded fuel. I am also working on end-of-flight dynamics, including more analysis of the Boeing simulations. This work should complement Richard Godfrey’s new work related to his drift analysis, which considers more northern impact points. The additional work, when considered in sum, should give us an indication as to whether a search further north should be considered versus a wider search at previously searched latitudes.

  378. Victor Iannello says:

    To all the readers in the USA: Happy Independence Day! Have fun, and be safe.

  379. mash says:

    @Victor Iannello

    re: Probability vs Possibility

    Before the search:

    pr(plane in area X) = 90% (roughly)
    pr(plane not in X) = 10%

    pr(plane in narrow corridor) = 90% (say)
    pr(plane in remaining wide corridor) = 10%

    After the search:

    Plane not found in narrow X.
    It could be argued that (based on previous expert analysis) there should be no change in the above probabilities , so

    pr(plane in wider X) = 90%

  380. ventus45 says:

    @Richard

    There appears (to my eyes at least) to be barnacles (or the remnants of their shells at least) here.
    https://www.jamiiforums.com/attachments/image-jpeg.359675/
    https://www.jamiiforums.com/attachments/image-jpeg.359674/

    Source article here.
    https://www.jamiiforums.com/threads/bawa-kubwa-la-ndege-laokotwa-kisiwani-pemba.1070044/

    If the flap had been “beached” for some time before discovery, (in that hidden little cove, under a ledge, accessible only by a small fisherman’s row boat remember), you would expect that it had most likely been “picked clean” by birds or crabs etc, just like the ROY fragment was, when found “the second time”.

    The ATSB’s own “internal” photos (that David so kindly posted the links to above – thanks) also clearly show, what appear to be, the remnants of barnacles, which suggests, if indeed not proves, that barnacles had been growing inside the structure.

    Absence of any discussion of the barnacles, or their remnants, on and or in the Pemba Flap, by the ATSB, does not prove anything.

  381. Victor Iannello says:

    @mash: The probabilities change as our knowledge evolves.

  382. Richard Godfrey says:

    @David, @ventus45,

    Many thanks for the photographic evidence of barnacles found on the Outboard Flap at Kojani, Pemba, Tanzania.

    I stand corrected and will include this item of floating debris in my new drift analysis.

  383. Niels says:

    @Richard Godfrey
    Can you explain a bit more about your drift analysis approach; is it based on the drift data of (undrogued) GDP drifters? How do you deal with the effect of wind on the different debris items you are considering in your analysis?

  384. Richard Godfrey says:

    @Niels

    I am currently writing a paper on the subject of drift analysis. In the paper, I explain, amongst many other issues, the difference between drogued and undrogued GDP drifters and the issue of windage. For the windage of the Flaperon found in Reunion, I use the results of the CSIRO analysis.

    Unfortunately, it will take another couple of weeks to finish the paper.

  385. Richard Godfrey says:

    @Dennis

    You raise the issue of timing between floating debris arriving off shore, beaching and being discovered at a particular location. There is also the possibility of debris re-beaching.

    In all locations where debris has beached there is an on shore drift greater than 0.5 knots. The fact that a debris item has beached at least once, means that it is quite likely to beach again if the tide takes it back out to sea.

    In many of the locations where debris has beached, there is a relatively small tidal difference. Some beaches are steep like Réunion, others are flat like Rodrigues. Some beaches are even behind a reef and once an item of floating debris has passed the reef, it is effectively trapped.

    The land and sea breeze system will also provide a daily point where wind, tide and drift align. If we allow a further 10 days of beaching and rebeaching from the arrival time in the area, then we are on the safe side. The bigger unknown was the time lapse between beaching and discovery, which forces me to use items with barnacles.

  386. Victor Iannello says:

    @Richard Godfrey: Because of the potential for a long delay between the time a part first arrives near the shore and when it is discovered and reported, it may be that the early arrivals are better than late arrivals in determining whether to eliminate a particular impact point as viable. Therefore, within reason, the early arrival of “Roy” in South Africa might be more useful to eliminate more southern impact points than the late arrival of the flaperon in eliminating more northern impact points.

  387. Niels says:

    @Richard Godfrey
    Looking forward to your paper. For the flaperon and engine panel indeed CSIRO provided useful information on how to deal with wind effects. What would you assume for the other two items?

  388. mash says:

    @TBill

    Remote control could be a possibility but not the only possibility. Similarly, pilot (or even experts on plane) might have suspected the remote control possibility.

    If the purpose is to disappear, why not just turn off the data source?

    Assuming Rogue Pilot Theory, please explain the SATCOM [off/on/(no off?)] sequence (no reaction even after two ‘reminder’ calls).

  389. Victor Iannello says:

    @mash: A remote control scenario requires that there is an undisclosed feature in the B777 AIMS in which all flight deck inputs are ignored, an identified communication link is established with the aircraft, and commands are sent remotely over that link which control the aircraft. That there is a secret feature that completely disables control and communication from the flight deck is very unlikely.

    If the SATCOM was powered down by isolating the left bus (which most believe is the most likely explanation), EICAS messages appeared which indicated that the SATCOM DATALINK and SATVOICE were unavailable. When the SATCOM logged on at 18:25, the datalink was unavailable, as there was no ACARS activity. We believe this was from deselecting the SATCOM link in the ACARS Manager screen. It’s possible that the reason to re-power the SATCOM was to re-enable SATVOICE capability to possibly monitor incoming call attempts, and/or to receive incoming calls, and/or to initiate outgoing calls.

    We can speculate about why the incoming call attempts were not answered, and why there were no outgoing calls.

  390. TBill says:

    @mash
    The explanation for SATCOM being turned off at approx. 17:10 is intentional turning off per Victor. The explanation for turning back on at 18:25 is intentional turning on. There are many operational/strategic reasons why the SATCOM/left electrical bus might have been turned back on.

    As @Rob has pointed put, one possibility is that the SATCOM strategy was to fake a crash in the South China Sea. Once safely out of Malaysian airspace there was no longer any need to maintain absolute stealth.

    You are assuming a rogue pilot would have kept the SDU off to hide the fact that the aircraft was still in the air, but your assumption is probably a weak one. My concern now is that the rogue pilot may have been using the sat call feedback to alter headings to hide the crash site.

  391. Trip says:

    When the satcom was turned back on at 1825 there was no aircraft id. How do you delete the id if it was a legitimate transmission?

  392. Victor Iannello says:

    @Trip: @Don Thompson believes that if the SATCOM link is de-selected from the ACARS Manager screen, there will be no Flight ID provided when the AES logs on to the Inmarsat network.

  393. DennisW says:

    @Victor

    @Don Thompson believes that if the SATCOM link is de-selected from the ACARS Manager screen, there will be no Flight ID provided when the AES logs on to the Inmarsat network.

    That should be verifiable. Strange that it is still and open item.

  394. Victor Iannello says:

    @Jean-Luc Marchand: In the past, you have disputed whether the flight path that overflies YPCC and crosses the 7th arc at 22S latitude would be allowed from fuel considerations. Do you still believe this to be true? I have provided the inputs and outputs to the fuel analysis, so you should be able to compare results.

  395. flatpack says:

    @TBill

    “My concern now is that the rogue pilot may have been using the sat call feedback to alter headings to hide the crash site.”

    By ‘sat call feedback’ you mean that, upon receiving (but not answering) the first sat call, the rogue pilot could infer that MAS ops were now ‘aware’ and trying to make contact.

    MAS ops were probably following some SOP decision tree. Presumably the rogue pilot was fully aware of internal procedures and could also infer that they would not yet be in panic mode because the satphone actually rang rather than was not able to connect (as indeed was the case).

    That there was only one subsequent satphone call attempt made is regrettable. Had frequent attempts been made there would have been more Inmarsat data available to constrain flight path reconstruction.

    Based on the sat call information available, I cannot see any reason for the putative rogue pilot to have altered his heading.

  396. Don Thompson says:

    @Trip wrote “When the satcom was turned back on at 1825 there was no aircraft id. How do you delete the id if it was a legitimate transmission?

    The 18:25 Log On did not include the ‘Alphanumeric Flight ID’, nor was it followed by an ACARS Link Test msg. The absence of both conditions indicates that SATCOM had been deselected on the MFD ACARS Mgr screen, with the consequence that the Data Comms Management Function was no longer effecting any control over the SDU. That is not to say the ARINC 429 interface between DCMF and the SDU would be totally inactive, but only periodic ARINC 429 status messages would be exchanged.

    Note that the ‘Alphanumeric Flight ID’, when optionally defined in the Log On, is used by an Inmarsat network management function, it is not related to ACARS or ISO-8208 comms.

    @DennisW, it is possible that absence of the ‘Alphanumeric Flight ID’ parameter was a consequence of the Flight ID field being cleared from the FMS. That it’s possible to clear the Flight ID from the FMS has been verified. However, as I have set out above, the simplest reason for the absence of the ‘Alphanumeric Flight ID’ and the subsequent ACARS Link Test msg is that SATCOM was deselected.

    Taking this further…

    I don’t hold idea that the perpetrator had a deep and thorough knowledge of the SATCOM system, and that the GES Log records were in some way intentional breadcrumbs. I doubt that isolating the L AC Bus was specifically intended to disable the AES (and TCAS). Isolating the L, and the R, AC Bus would remove power for most of the cabin services, I’ll suggest that disabling electical power to the cabin was the intent during the period ending at 18:25.

    Deselecting SATCOM using the MFD ACARS Manager checkbox ensures that the aircraft would not initiate any ACARS messages. However, the aircraft documentation does state that ACARS messages may be received over a medium that has been deslected.

    While @Victor has described that various SATCOM statuses are reported via the EICAS there is an important nuance of ACARS: an ACARS communication session is aircraft originated. That is, an aircraft must initiate the correspondence with the ACARS server on the ground. While the SATCOM Log On with the GES implicitly opens a datalink for ACARS, more correctly a ‘logical channel’ over the datalink, the core GES Log On does not imply any ACARS correspondence.

    If there was any notion that the perpetrator in control of 9M-MRO restored main AC Bus power with the expectation of receiving a ground-to-air ACARS text message via SATCOM, that expectation would never have been fulfilled.

    Recall that the FI’s ACARS Traffic Log shows that at 18:05 MAS OCC originated a message to 9M-MRO and the message delivery failed. 9M-MRO’s AES failed to respond on the SATCOM datalink. The failure resulted in the ACARS ground segment clearing its knowledge of any path to 9M-MRO.

    While 9M-MRO’s AES renewed its Log On at 18:25, its Data Communications Management Function did not restore the ACARS session. The ACARS server remained unaware of any path to 9M-MRO. Had MAS OCC, or any other party, originated an ACARS message to 9M-MRO after 18:25, the ACARS server on the ground would have rejected any such message back to the originator.

    Its notable that the FI describes that MAS OCC continued to receive message rejections after 18:25.

    While there seems to be a tacit narrative abroad that the perpetrator had specific and detailed knowledge of ACARS & SATCOM messaging, I tend to an opposite view. That person’s knowledge was incomplete.

    Imagine that a protocol had been planned between the perpetrator & an associate on the ground, and that the protocol required a ground originated ACARS text message after 18:25 so as to initiate a particular phase of the plan. Had this been the case, as a consequence of the perpetrators’ misunderstanding the operation of ACARS, and a message was initiated but not forwarded to 9M-MRO, then a tragic farce ensued.

  397. ventus45 says:

    @Don
    “Imagine that a protocol had been planned between the perpetrator & an associate on the ground ……… ”

    I, (and I presume many others) have thought long and hard about that in the past.

    Long story short, I concluded that you would not use satcom or acars, specifically because they would be traceable through the system logs.

    You need an untraceable link, and there is only one, HF radio.

  398. Don Thompson says:

    @ventus45 wrote “I, […] have thought long and hard about that in the past.

    Me too, but the topic is wholly speculative.

    I agree that a conversation over HF radio might be ‘untraceable’ in as much as identifying the remote party or their location. However, HF is an open broadcast and tramsmissions may have been received and recorded by any member of the DX’ing community, or other MAS crews.

    MAS did install its own HF radio base at the airline operations control centre during 2013. Long range operational communications with their 737-800s while operating on MY-IN routes was cited as the requirement for this HF base. Presumably, by installing their own HF base MAS wished to avoid the costs of using the ARINC LDOC HF service that includes a station at Hat Yai, Thailand.

  399. Victor Iannello says:

    @Don Thompson, @ventus45: With SATCOM call capability, the cockpit can instantly be in communication with anybody with a telephone connection.The logistics of using HF are more complicated.

  400. David says:

    @Victor. You commented to the effect that the early arrival of “Roy”, “might be more useful to eliminate more southern impact points than the late arrival of the flaperon in eliminating more northern …”

    “Roy” has been treated as an outlier hitherto and the flaperon could have been missed discovery for some time while still bouncing about in the vicinity. The same could applying to others and I note the water line of the Pemba flap was clearly defined as if it had floated undiscovered in a sheltered cove, as its retrievers thought possible.

    Also, the part of tide cycle at the “Roy” first beaching together with offshore/onshore wind variations might account for its eventual retrieval further higher up that same beach. Elsewhere shore steepness and a lengthy bumpy arrival before beaching and site remoteness might have added to such complications, the aggregate not necessarily entailing stripping of barnacles.

    Could offering datum status to “Roy” have the potential of offering powerful support for a search north?

  401. Victor Iannello says:

    @David asked Could offering datum status to “Roy” have the potential of offering powerful support for a search north?

    I think so. Certainly Richard G. will consider the timing of the (first) “Roy” discovery in his drift studies.

    I think different floating orientations of the flaperon should also be considered. Can we be sure that the buoyancy measurements made by the French authorities and CSIRO are representative of how the flaperon floated across the Indian Ocean? For instance, if the flaperon was lower in the water and only lightly influenced by winds, this could help explain the absence of a water line, the barnacle distribution, and the timing of the arrival to Reunion Island with a more northern impact along the 7th arc.

  402. DennisW says:

    @David

    Could offering datum status to “Roy” have the potential of offering powerful support for a search north?

    Good question. “Early” arrival time should carry a lot of weight.

    @Don

    Thx for the details. I have long been an outside the camp relative to the left bus depower scenario. I don’t have an alternate explanation for the 18:25 log-on, but the accuracy of the BFO data at that time makes me question the removal of power to the AES.

  403. MH says:

    If using HF comms maybe a ham radio (or other such HF gear) was installed in the comms bay and they used telemetry However messages were encrypted before transmission

  404. TBill says:

    @flatpack
    “That there was only one subsequent satphone call attempt made is regrettable.”

    Not so sure…sat phone calls take away the BTO readings. Not sure if it is good or bad we have the sat calls from a flight path mapping perspective.

    I do not think the pilot knew BTO/BFO would be calculated, but I do think he might not have been completely oblivious to the potential for satellite/phone call data to provide position information.

  405. ventus45 says:

    @Victor, @Don.

    “Security”, “anonymity” and “un-traceability” are obviously paramount considerations for the “ground accomplice”, so the LAST thing you would use is SATCOM, regardless of ease of use, for obvious reasons.

    The advantages of using the HF are many.

    The aircraft equipment can operate on any frequency between 2 and 30 Mhz, and there are two of them.

    This means that the “ground accomplice” could use virtually any readily available civilian HF radio set, from any band in the 2 to 30 Mhz range, with SSB, owned by anyone. It could be HAM gear, marine gear, even some good old CB sets would do quite nicely thank you. Indeed, you would not use frequencies in the main bands that you know are used and monitored by many, you would deliberately use “obscure”, or better yet, “garbage” frequencies (like the HF CB bands), that “good” radio people, avoid like the plague, for rather obvious reasons. In other words, you would “hide in the noise”.

    Moreover, for good measure, since the aircraft has two HF sets, you would use them both, ie go duplex, ie, use two frequencies, one for Tx and one for Rx, with one on each radio. That means the ground party needs either two radios or just one Tx capable radio and one Rx radio. There are plenty out there, readily available for very little money, no questions asked.

    Furthermore, the transmissions themselves, both ways, would not be “plain language” that any eves dropping third party would pick up on, and they would only hear one half of the conversation anyway. That conversation would only need to be very simply coded in a way that was useful to the two parties concerned. Only hearing one half of that conversation, would just be gibberish to anyone else listening.

  406. Victor Iannello says:

    @ventus45: Yes, I understand the advantages that HF offers. I’m a ham myself, and I am active on the HF bands. (By the way, you mention CB. In the US, that would be 27 MHz – 11 m. If I wanted to guarantee that there would be suitable propagation conditions, that would not be a very good band to choose.) And yes, a satvoice conversation would leave behind evidence. However, if you wanted to quickly organize a conversation between the cockpit and one or more high-ranking official in Najib’s office, what would you choose? I’m not proposing this occurred, but I am saying that there are advantages and disadvantages to both satvoice and HF communications, depending on what communication needs were anticipated.

  407. DrB says:

    @flatpack,
    @TBill,

    To expand, a satphone call provides BFO data but no BTO data, and it resets the 1 hour timer. Thus, if MAS had made calls more frequently than every hour, we would have missed all the BTOs after 18:40 except for the last one at 00:19 triggered by the SDU being repowered. An unexplained anomaly is the 56 minute spacing between the 23:15 phone call and the 00:11 message. Why wasn’t it one hour?

  408. DennisW says:

    @DrB

    Good points.

  409. Niels says:

    It has been suggested that the “ROY” piece find (dec 2015) could be indicative for more northerly crash locations. I’ve been looking more in detail and based on CSIROs first report, fig. 3.1.1. I tend to disagree. I don’t think it is impossible (based on CSIROs calculations) that the debris originating from the yellow region (which includes S32 degrees) would reach Mossel Bay as early as Dec. 2015. It is important to note that there is strong current along the South African east coast in south-westerly direction which could take pieces from near Durban into the Mossel Bay area in perhaps as little as 1 – 2 months.

    http://www.marine.csiro.au/~griffin/MH370/br15_MH370_IOCC_tp3l1p2d_arc7_4422/20151223.html

    See also:

    https://www.youtube.com/watch?v=oNppEPKt52E

    To me this video also nicely illustrates the complexity of modeling ocean surface currents.

  410. Victor Iannello says:

    @Niels said: I don’t think it is impossible (based on CSIROs calculations) that the debris originating from the yellow region (which includes S32 degrees) would reach Mossel Bay as early as Dec. 2015.

    I’m not of aware of anybody making this claim.

  411. @victor

    I am in the middle of another analysis but I had a quick look at your request 🙂

    Thanks for the new data in the Excel file.
    To estimate correctly the fuel consumption we have to have few more points along the path from 17h22.
    Please find below the list of info we need at the minimum:
    – position at 17h22 where it descends to FL340
    – 3 points in the Major Turn 1 after 17h22
    – Time and position at the South of Penang
    – position and heading at 18h38 where it descends to FL250
    – position and heading at 18h41 where it starts the holding

    – Position and headingat Nicobar at 18:59
    – characteristics of the holding : position of the 2 summits points
    – position and heading at 19h30 where it climbs to FL320
    – position and heading at 19h32 when flying mode is switched to M0.8
    – position and heading at 22h12 when “passing the target”

    With that and the updated table and the Excel file we should be able to provide you with our estimate.

    The position of the holding, and the climb to FL320 vis-a-vis the sourrounding routes is of importance: if it descended to FL250 it was to pass underneath the routes I guess and thus the climb to FL320 should be safely done away from them.

    Do you have a more detailed description of your model of fuel consumption? what are your assumptions on the aging of the engine ? and on the over-consumption of the right engine ?

    Thanks.

  412. ventus45 says:

    @Victor
    I think the idea of conducting “negotiations” – from the cockpit – is utterly ridiculous.

    The “negotiations” would have been very simple, and conducted by the ground accomplice(s), NOT the pilot.

    Moreover, they would not have been “negotiations” in the conventional sense as such, but would have been a set of one or more “ultimatums”, with “a clearly stated and inflexible deadline” for compliance, “or else”.

    The only communications the pilot needed were:
    (a) Pilot to Ground – I have the aircraft – the plan is a “go”.
    (b) Ground to Pilot – Roger.
    That would have occurred very early, either over KB, or over Penang.

    Ground accomplice(s) then place the demands.
    The time frame to “deadline”is one hour.
    Assume accomplice is in Penang (17:50).
    Deadline is now 18:50.
    Deadline for a “suitable response” expires.
    The die is cast.
    Pilot executes the “or else” – fly to the SIO.

    From the pilot’s viewpoint, sitting in the cockpit, by now west of Sumatra, lack of a “suitable response” by “the deadline” could be in one of two forms:
    (a) no further Tx from ground at all, which could be either:
    (i) government is “stonewalling” as per “standard policy” – we don’t negotiate with terrorists – etc – etc.
    (ii) one or more accomplice(s) were already arrested, or otherwise thwarted.
    (b) Tx from ground accomplice(s) advising of a definitive “no deal”, per (a)(i).

    That is all that would be required for the pilot to decide to execute the: “or else”.
    It is as simple as that.

    Najib was very specific when he made that “deliberate action – by someone on the plane” statement. He knew.

  413. Victor Iannello says:

    @Jean-Luc Marchand: I supplied you with the data (minute by minute altitude, speed, rates of climb/descent, percent engine thrust, whether in racetrack pattern, temperature, weight, PDA, and fuel flow) so you can independently check the fuel calculations. Rather than do this, you are asking for more data that is not required to do the fuel calculation. If you are busy with other work, that’s fine. I wasn’t really making a “request”.

  414. Victor Iannello says:

    @ventus45: You seem quite certain about what kinds of communications might have been anticipated with the pilots in the cockpit. I don’t know, and you don’t either. There could be reasons to use satvoice beyond “negotiations” and “ultimatums”. At this point, we are speculating.

  415. ventus45 says:

    @Victor

    Nothing is certain, obviously, that is why we are speculating, is it not ? All we can do is speculate on what his objective(s) and requirements may have been, then devise scenarios that could be feasible, and test them for reasonableness. There may be clues deduced in that process.

    Frankly, I can’t think of any sensible, operational reason, to re-power satcom, since obviously he had no intention of actively using
    it for any outgoing communications, (since he did not), and the only incoming calls were not answered.
    But having said that, we do not know what he really knew about satcom.
    Perhaps he had a sneaky plan for re-powering satcom.
    we know about his discussions with an avionics engineer, but not the substance of those discussions.
    I think it is a mistake to assume that he did not know about the BFO’s and BTO’s, just because the rest of us did not know.

    What if he knew that a succession of incoming frantic phone calls from MAS Ops (which he would have expected) would continuously reset the hourly ping timer without BTO, thus ensuring that they would know he was still in the air, but without any way of ever deducing where he was ? When you think about it, where would we be without the BTO’s ?

    Perhaps the reason for re-powering the left bus, could have been either specifically to get back power for the left HF, or it could also have been to deliberately re-power satcom so that the “expected” stream of phone calls from MAS Ops would in fact arrive, or perhaps both.

    Could you imagine him sitting there, and getting the first phone call right on que, and being very happly, initially, then “nothing more”, for hours and hours ?

    He probably would have thought that they had assumed he had already crashed, so all good from his point of view. Only hours later, when the second call finally arrived, he may have thought that they weren’t sure he had crashed, but by then, it did not matter, even if some BTO’s had been recorded in the interval (which we have), because he still would have been comfortable in the knowledge that they could not work out where he was, even with a few BTO’s, and let’s face it, we still can’t.

    Looking at it from his point of view, keeping satcom alive serves the purpose of proving that the aircraft did not crash early in the flight, and by extension, if we presume that he had a pretty good idea of how the government would respond, and did, you could make the case that he re-powered satcom deliberately, to ensure that he politically damaged the government.

  416. David says:

    @Victor. “Can we be sure that the buoyancy measurements made by the French authorities and CSIRO are representative of how the flaperon floated across the Indian Ocean?” To the contrary I think we can be confident that they are not.

    I have raised the possibility of water infusion due to damage to the honeycomb (structure or skin adhesion). In the flaperon, stresses included those of trailing edge loss and collision with the adjacent flap. Infused water might have dried out ere the flaperon was re-floated in France. Supporting this, note that the description of item 26 in the Malaysian Debris Examination, the right aileron part, includes, “The non-metallic honeycomb was soaked with water resulting in the core material to swell”, and; “the non-metallic honeycomb core was wet”.

    Besides, the recovered flaperon was more draggy in water due to the barnacles. Their amount and size on the trailing edge indicates this had been awash for some time or they would not be there. Thus aside from drag its windage also would be different to that trialled by the CSIRO. Also I doubt it would have tumbled as did the CSIRO’s given the mass and submersion at its rear.

    With the unavailability of the recovered flaperon to trial one indicator of the effect of honeycomb damage would be a flotation trial by the Malaysians of the part outer flap (which they now hold) for like water absorption, though that would take some time. However I do think the barnacles on the rear end of the recovered item are sufficient evidence that the CSIRO flaperon drift trial suggests a higher transit speed, and probably drift angle, to that realised.

    On a related matter, at page 7, (318/440) Appendix G of the final ATSB report of the Operation Search, dated 3rd October, 2017, it states, “Macrofaunal analyses of Items 2 and 3 showed that these aircraft pieces were colonized with opportunistic and cosmopolitan species originating from the tropical Indo-Pacific Ocean. No evidence of cool or cold-temperate molluscs or annelids was found.” Items 2 and 3 are the No. 7 flap fairing part and the right horizontal stabiliser panel, both recovered in Mozambique. These were the only two items included in the Geoscience macro and micro fauna analysis. (Both items also had evidence of barnacles having been attached).

    Looking into this in more detail, at page 29 (340/440)), one mollusc in particular (Petaloconchus renisectus) is found in the tropical Indo-Pacific Ocean only. The specific example on the panel was 8-12 months old and had, “encrusting (growth) on the under-surface consistent with (that) fouling the aircraft debris” which means therefore that the panel, “had been in tropical waters at some early stage in its drift”.

    However, “….some early stage….” is questionable. It was recovered on 27th February, 2016. If it had beached shortly before this, that mollusc would have begun life between the end of February, 2015 and the end of June that year. So while the panel might well have been in the tropics before then if time for the mollusc’s initial attachment is included, if that was brief the panel may not have been in the tropics until around a year after the crash. However, I mention this in case it should be of use to @Richard with drift analysis.

    As to there being, “no evidence of cool or cold-temperate molluscs” of the above, Geoscience does add that this does not mean necessarily that the panel had not been there.

    On another topic, while naturally there has been little interest in the detail of internal damage to the MH370 right outer flap part, nevertheless I have updated the below again, that now (5th last para) discussing the fracture mode of MH17’s outer left flap support-track carriage, which I illustrate. This track was at the outboard end of that flap. Whereas the track in MH370’s case was at the inboard end, its carriage, missing, is similar and might have failed in the same way.

  417. haxi says:

    @DrB,

    Regarding the one hour timer, I asked Inmarsat’s experts the same question. Here is their answer.

    Whenever an AES is active, it resets a timeout counter (to 60 minutes as specified by a configuration parameter TDlv). Whenever the GLV process runs (every 5 minutes as specified by another configuration parameter TGlv), the counter is decremented and any AESs for which the counter has reached zero are queued for log on verification (LOV).

    So, it is possible to schedule a Log On Interrogation approximately 55-60 minutes after the last activity. If the last activity was a Log On Verification, then the interval will likely be close to 60 minutes since the LOV was initiated on a GLV tick. If the last activity was something else (e.g., receipt of a C-channel SU), then it could be up to TGlv ie 5 minutes less.

    In the example from MH370, the last activity at 23:15:02 would have reset the count to 60, but GLV likely ran 2 minutes later and decremented the count by 5 minutes (even though only two had passed).

  418. Victor Iannello says:

    @David: Thank you for the comment on flaperon buoyancy and marine biology. As is often the case with MH370, the analysis remains inconclusive.

    You said, there has been little interest in the detail of internal damage to the MH370 right outer flap part. I don’t agree. I have not weighed-in with an opinion because I believe that the ATSB is in a much better position than me in determining whether or not the flaps were extended in a ditching.

  419. mash says:

    @all

    re: SATCOM [off/on/(no off?)] sequence

    Aha! If the “1st off” above was a ‘software-off’ (in cockpit) and the “2nd on” was a ‘hardware-on’ (not in cockpit), this would almost immediatlely invalidates the (pure) Rogue Pilot Theory …

    Is ths correct?

  420. Victor Iannello says:

    @mash: You are confusing the sequence. The theory is that the SATCOM was powered off by isolating the left bus (using overhead switches in the cockpit) after 17:07 but before 18:02. While powered down, the SATCOM link was de-selected in the ACARS Manager screen via a MFD. Then, around 18:23, the left bus (again, using overhead switches in the cockpit) was re-powered, and the SATCOM was re-started, but without ACARS capability.

  421. Victor Iannello says:

    @TimR: You said in the past: On reaching cruise altitude Captain Zaharie signed off from Malaysian ATC, turned off some of the equipment, turned back over Malaysia and proceeded up the Strait of Malacca out of Malaysian radar contact then turned the power back on to receive confirmation the demands had been agreed to.

    We know that the SATCOM call at 18:40 was not answered. If the captain was awaiting confirmation about demands made by accomplices on the ground, why was this call not answered?

  422. Donald says:

    @Ventus

    You said “I think it is a mistake to assume that he did not know about the BFO’s and BTO’s, just because the rest of us did not know.

    People in the industry knew about BFO/BTO. Gerry Soejatman knew all about BTO/BFO. He even claims to have firsthand information from Indonesian engineers to the effect that various govts. (usual suspects…US, Israel, China, Russia)had spoofing capabilities and that this was demonstrated in front of said engineers. Of course, he made this claim after initially stating that he had no knowledge of such a capability?

    I happen to think he’s full of shit, but if you take him at his word it’s not at all far-fetched to believe that Z would have some insight and would have figured this into his plan.

    I imagine Victor has had private correspondence with him so perhaps he has an opinion on Gerry’s claim?

  423. TBill says:

    @ventus45
    I agree with you.
    Here is an FAQ by Inmarsat dated 21-March-2014 from their website.
    Inmarsat_FAQ
    I think it is reasonable to assume a tech savvy pilot *might* have been aware of this basic info.

  424. Victor Iannello says:

    @Donald: I believe Inmarsat when it says that the relationship between the BFO and aircraft flight dynamics, position, and satellite orbital mechanics was not known until after the MH370 disappearance.

    I’ll add that I am not in favor of theories that require an omniscient, omnipotent bad actor with accomplices.

  425. DennisW says:

    @Victor

    I agree with you which is why I also discard spoofing scenarios. That would take the “omniscient” and “omnipotent” to the next level.

  426. Victor Iannello says:

    @DennisW: Yes. It’s also impossible to disprove a theory in which any conflicting evidence can be discarded because an omniscient and omnipotent actor would have the ability to alter or fabricate evidence.

  427. flatpack says:

    @TBill,
    @DrB,
    @haxi

    Thank you all for pointing out that sat calls would reset the LOV timer and thus additional calls would be a ‘bad thing’ from a BTO perspective.

    The irony is that I had perceived the sole subsequent satphone call attempt as bordering on negligence by MAS but actually it was a ‘good thing’.

  428. NEW PAPER ON FINDING MH370 VIA CARGO.
    @All

    I’m not a member of the aviation community, but after all the search failures, I’m sure there’s plenty of people thinking we’ve got to find a Game-Changer that doesn’t involve Same Searches – Same Result.

    Acting on the possibility that MH370 may have had a tonnage of lithium on board, I’ve attempted to show that biology such as long-lived sea sponges may serve as witness plants revealing the debris field via lithium intake. Li may also substantially change the nature of the breakup.

    I consider the idea to be more possible than likely, but I’m hoping it might bug better ideas out of others.

    https://docs.google.com/document/d/12EH8fWOSuIbV5TbVP1WwDCb8x9v9OZPKv-xGtWVT9Cg/edit?usp=sharing

  429. TimR says:

    @Victor
    “We know that the SATCOM call at 18:40 was not answered. If the captain was awaiting confirmation about demands made by accomplices on the ground, why was this call not answered?”

    I do not know why the SATCOM call was not answered. Possibly communication was only to be conducted by duplex HF.

  430. Victor Iannello says:

    @TimR: Can you please ask your source for more details about how, what, and when communications were planned with the flight deck in that scenario?

  431. mash says:

    @Victor Iannello

    According to “the theory” it was an [in-cockpit power-off, in-cockpit power-on] sequence, but it could also be an [in-cabin power-off, in-cabin power-on] sequence, right?

    Notice also the 1st half-hourly ACARS report was also the last, so the “software-off” could also be a more specific “software-report-off” instead of a “software-SATCOM-off”.

  432. TimR says:

    @Victor
    ” Can you please ask your source for more details about how, what, and when communications were planned with the flight deck in that scenario?”

    My source was not involved in that sort of detailed planning.

  433. flatpack says:

    @Victor Iannello

    Was there a CLI display in the cockpit?

    If so, it is possible that the SATCOM call at 18:40 was not answered because the call was clearly from MAS, rather than another party.

  434. David says:

    @Victor. About internal flap damage discussion you say, “I have not weighed-in with an opinion because I believe that the ATSB is in a much better position than me in determining whether or not the flaps were extended in a ditching”.

    While the ATSB’s principal intent was to rebut flaps-extended ditching it also offered the opinion, derived from flap damage, that the flap and flaperon separations are “consistent with” an aircraft breakup. While that is an opinion I share, I believe the flap evidence per se does not exclude other possibilities; that belief being consistent with “consistent with”. An open question is whether the flap damage sequence and interpretation, undisclosed by the ATSB, damage to the flaperon (the ATSB’s photograph from France of the damaged flaperon rear spar is undecipherable) and any parallels from MH17 could rebut or reinforce breakup as cause of the separations.

    Rather like Boeing simulations and the lack of information on them, that to me has been insufficient reason to leave this alone. Yet with the final report bearing down, we hope, maybe that will discuss final aircraft speeds, configuration and attitudes so best await that.

  435. Don Thompson says:

    @flatpack

    No, calling line identity is not signalled from the GES to the AES during the C-channel setup.

  436. Don Thompson says:

    @ventus45, others

    The intent of my comment, above, was more to counter any notion that the perpetrator of 9M-MRO’s disappearance had a deep and comprehensive understanding of SATCOM system operations, rather than to explicitly speculate on methods for comms with the ground. Unfortunately, to explain that counter idea it was necessary to describe the issues involving ACARS session operation.

    Responding to @Donald: Soejatman makes a number of entirely bogus assertions in this blog post while wrapping the post in an air of plausibility.

    @Haxi, thank you for relaying the description for the Log On Verification timer. That is consistent with an explanation I had set out some, very long, time ago.

  437. mash says:

    re: Communication vs Pressurisation

    Latter ‘implicit’ communication would also invalidate former aircraft depressurisation.

  438. mash says:

    re: Physics 100 (Simple Harmonic Motion – Probability Density Function)

    Application: Probability of locating plane in certain band width
    Model: Only for restricted max-range straight-line controlled glide
    Method: Derive SHM density function from x = a sin(theta)
    Function: Integral of [dx/(pi*sqrt(sq(a)-sq(x)))] = (1/pi) * arcsin(x/a)
    Example: Probability between +0.5 and +1.0 is (normalised)
    (1/pi) * [arcsin(1) – arcsin(0.5)] = 1/2 – 1/6 = 33.33%. By symmetry, between -0.5 and -1.0 is also 33.33%. That is 66.67% outside half of maximum glide range.
    Comment: Similar result as to above ‘trigonometry” method but conceptually more interesting and perhaps more flexible (e.g. Damped SHM).

  439. Victor Iannello says:

    David said: Rather like Boeing simulations and the lack of information on them, that to me has been insufficient reason to leave this alone.

    I am trying to get more information about the simulations to share in order to better understand the initial conditions and the results. If we find, for instance, that there is a very low probability that, without pilot inputs, the aircraft flew a distance greater than 22 NM after the final SATCOM log-in request, that would provide additional justification to search further north rather than wider.

  440. Gysbreght says:

    In all simulations the airplane flew a distance greater than 22 NM after the final logon request. “In some instances, the aircraft remained airborne approximately 20 minutes after the second engine flameout.”

    In one simulation it flew 95 NM after the final logon. However, the radii of the turns was such that the tracks stayed within about 20 NM of the 7th arc, and the endpoint was always within 15 NM of the 7th arc.

  441. mash says:

    @DennisW
    @all

    re: Maximal vs Minimal Power Outage Time

    DennisW says
    “… I don’t have an alternate explanation for the 18:25 log-on, but the accuracy of the BFO data at that time makes me question the removal of power to the AES.”

    Since the power outage (if any) could be as short as 22 minutes or as long as 1 hour and 16 minutes; in order to fit the BFO data, it is more likely that it was a minimal outage (equipment plausibly still in ‘working’ condition) than a maximal one.

    Therefore, a theory that does not need too long an outage time (or maximal outage time) is more favorable, especially if the BFO deterioration rate can be verified (to some extent).

    (Second thought)

    Actually, an even better way is to find out if there is another explanation for the unsuccessful “handshake’ (18:03) in that case the power outage time can be as short as a second …

  442. Victor Iannello says:

    @mash, @DennisW: The large BFO excursion is more consistent with a long outage, not a short one. Mike Exner has shown that the first BFO value is in error due to a packet collision, and should be ignored, as Ian Holland recommends. The remaining BFO values in the cluster show a sequence consistent with an long outage.

  443. Victor Iannello says:

    @Gysbreght: When I was referring to the distance flown after the log-on request, I was referring to the straight-line distance to the impact, not the length of the curved path. However, ultimately, we care about the distance from the 7th arc, which of course is an even shorter distance.

  444. TBill says:

    @sk999
    Question for you- when you worked up sat data for the 9M-MRO Beijing to KLIA leg, did you see any data for the C-channel BFO’s to help calibrate the sat call BFO’s to the regular ping channel BFO’s? Probably not but just in case.

  445. DennisW says:

    @Victor

    Mike Exner has shown that the first BFO value is in error due to a packet collision, and should be ignored, as Ian Holland recommends.

    Yes, we have debated this topic rather extensively.

    The 18:25:27 BFO value is consistent with the 8 BFO values logged during 18:28:xx time period. The 4 BFO values logged between 18:25:34 and 18:27:08 are the outliers.

  446. ventus45 says:

    Seabed Constructor has just sailed from Dampier.
    https://twitter.com/Ventus_45/status/1017236625749757952

  447. ventus45 says:

    Seabed Constructor is heading for Capetown – South Africa
    https://twitter.com/Ventus_45/status/1017245442310979584

  448. sk999 says:

    TBill,

    No C-Channel transmission on MH371.

  449. Don Thompson says:

    @DennisW wroteI don’t have an alternate explanation for the 18:25 log-on, but the accuracy of the BFO data at that time makes me question the removal of power to the AES.

    A Log On Request made by the AES includes a 6-bit data field intended to define the ID of the GES with which the last Log On session was maintained. In the case that the AES executes a Log Off, or is power cycled, prior to a Log On this field does not define a valid GES ID, all bits = ‘1’.

    During the 18:25 Log On this data field was set to all ‘1’s, and no prior Log Off is recorded. Therefore one has to assume the SDU was restoring service after a power off.

  450. TBill says:

    @sk999
    Should we ask Inmarsat to search for a Channel C comparison in a prior flight? In other words, would that help? Or would any offset between channels fluctuate with time. Or alternately does Inmarsat have an opinion on the magnitude of the offset if any?

  451. DennisW says:

    @Don

    During the 18:25 Log On this data field was set to all ‘1’s, and no prior Log Off is recorded. Therefore one has to assume the SDU was restoring service after a power off.

    Or a LOG OFF was not received by the GES. The fact that the 18:25 BFO value is essentially correct given the position and velocity of the aircraft at that time flies (no pun intended) in the face of a “let’s discard the value” position. It would truly be extraordinary of the 18:25 BFO was “accidentally” correct, but it could be.

  452. Gysbreght says:

    @TBill: I recall that ATSB stated that tests have been done to establish any BFO offset for the C-channel and the result was no difference.

  453. Don Thompson says:

    @DennisW wroteOr a LOG OFF was not received by the GES.

    If the AES was unable to complete a Log Off with the GES, and yet its power was maintained, then a subsequent Log On Request would define that 6-bit ‘Satellite ID (prev]’ field to identify the previous region/GES* with which a Log On had previously been maintained.

    An explicit Log Off is initiated by the AES with a Log-Off Request burst followed by acknowledgement from the GES.

    An implicit Log Off is recognised by the AES if it loses sync with the GES originated, continuously broadcast P-ch. An implicit Log Off is recognised by the GES if its periodic Log On Interrogation of an idle AES fails.

    The Stratos Log recording 9M-MRO’s traffic during the MH371 service shows many Log On renewals where the AES has experienced an implicit Log Off. The 12:50 Log On follows an aircraft power-up. As the crew prepares the aircraft for the MH370 service, just prior to 16:00, the AES executes an explicit Log Off from POR and a Log On with IOR.

    In each recorded case, the AES configures the 6-bit ‘Satellite ID (prev)’ field as expected: only following an explicit AES Log Off or loss of power is that field defined as ‘111111’.

    \[*\] I did write GES ID, above. The field is defined as ‘Satellite ID (Prev)’ but given the satellite region-GES configuration existent in 2014, only one GES existed per region.

    Holland described that the demodulation and decode of the 18:25:27 LOR burst resulted in “a non-zero bit error rate (BER) associated with the log-on request at that time. The associated received signal level and carrier-to-noise density ratio (C=N0) were also unusually low. As such, the first BFO was deemed untrustworthy“. The condition has been attributed to a ‘collision’ on R-channel 36E1, that the GES CU had ‘locked onto’ 9M-MRO’s burst while another AES transmitted a burst on the same channel, later in the 40ms BTO window, and with a sufficient freq offset for the CU to complete its decode of 9M-MRO’s burst.

  454. DennisW says:

    @Don

    BFO does not have anything to do with BER. It is simply a frequency measurement.

    In any case, it is not important. Discarding the BFO value at 18:25 has no impact on terminus calculations. I don’t even know why I objected to it. Probably because of Victor’s use of the language “Mike Exner has shown that the first BFO value is in error due to a packet collision” annoyed me greatly. The reality is that no one knows if the first BFO value is even in error much less the cause.

  455. Don Thompson says:

    @DennisW

    My quote from Holland’s paper introduced BER, and the idea that the 18:25 LOR burst from 9M-MRO had been affected by a collision with a burst from another AES. MRO’s burst won out, the other lost. I understand that the interpretation of RxPwr, C/N₀, and Estimated BER to suggest collision came from Holland, et al.

    The GES demod has 250msec to ‘lock’ onto an unmodulated carrier; another 123msec to extract/sync the symbol clock; 53msec to decode the Unique Word, and then 533msec to decode the scrambled & FEC’ed i-frame. I’d presume the decoder part recovered an error while processing the i-frame causing the ‘Estimated BER’ metadata item to be recorded as non-zero.

    I am curious to understand how the GES CU derives BFO. It appears to be reported as each interleaver block of a burst is processed. An R-channel burst comprises only one interleaver block, whereas a T-channel burst comprises many interleaver blocks and a BFO is recorded for each (a T-ch burst can be regarded as an extended R-ch burst).

    Is the BFO an instantaneous measure or an average? Given the period of the interleaver block (533msec at 600bps or 266msec at 1200bps) the average might offer more confidence? The 00:19 BFOs are the key indicators for RoD at that time, more confidence, the better.

    The GES CU is built around Analog Devices TigerSHARC DSP, should any readers have experience of how that device might typically process the MSK-like A-BPSK burst, do please contribute.

  456. DennisW says:

    @Don

    I am curious to understand how the GES CU derives BFO

    Good question. I have never seen any description whatever or an estimate of the accuracy expected.

  457. sk999 says:

    TBill, Gysbreght:

    The ATSB report of Dec 3, 2015 provides the following (p. 21):

    ———

    C-Channel Burst frequency offset bias (BFO)

    The use of the C-Channel BFO information was key to the search as it provided the first frequency value that implied a southerly direction of travel. This relies on the C-Channel bias being similar to the R-Channel bias. Initial analysis performed on similar transmissions indicated that it was highly likely that this was the case.

    Due to the importance of this information it was deemed favourable to confirm these results via a flight test. Using a sister ship to 9M-MRO, several phone calls were made to the aircraft under controlled conditions. Each call produced a large number of C-Channel measurements which were analysed. The C-Channel BFO bias was found to be equivalent to the R-Channel BFO bias.

  458. TBill says:

    @sk999 @Gysbreght
    Interesting, thank you.
    Certainly that is the assumption Holland seemed to make in his paper when he plotted BFO versus time. He accepted 23:14 BFO as reported.

  459. Victor Iannello says:

    A China Air B737 co-pilot mistakenly turned off the air packs in attempt to prevent recirculating smoke into the cabin to hide his vaping. This led to a decompression event followed by a rapid descent from 35,000 ft to 10,000 ft. After the air packs were turned on, the plane climbed to 26,600 ft and continued, with depleted [emergency] cabin oxygen.

    I doubt we’ll see the pilot and co-pilot flying again…

    https://www.cnn.com/2018/07/12/asia/air-china-smoking-pilots-intl/index.html
    https://www.reuters.com/article/us-air-china-investigation/air-china-plane-loses-cabin-pressure-mid-flight-idUSKBN1K20UT

  460. TBill says:

    @Victor
    “…depleted cabin oxygen”

    On my soapbox, but it would be news to me that anyone is monitoring O2 content of aircraft cabin, or even temperature. I know in the wake of MH370 there are some calls for better ELT devices which could monitor confined space life parameters and signal alarm to outside world. Just nuts that is the current state of affairs, but I am speaking as a chemical engineer who has to go into confined space reactors, not as an aeronautics engineer.

  461. Victor Iannello says:

    @TBill: I didn’t properly word that sentence. When I said the cabin oxygen was depleted, I was referring to the remaining emergency oxygen. I’ll change the wording to make it less ambiguous.

  462. TBill says:

    @Victor
    That makes more sense if the article is trying to say the cannisters were depleted, but the article does speak in terms of oxygen content.

  463. Victor Iannello says:

    @TBill: You are placing too much faith in CNN. Here is an excerpt from the Reuters article (second link above). It’s clear they are talking about the remaining emergency oxygen, not the oxygen in the cabin air.

    The plane descended from 35,000 feet to 10,000 feet in 10 minutes as is standard practice in a decompression event, according to FlightRadar24 data, then ascended and continued the flight at a peak altitude of 26,600 feet rather than heading to a nearby airport to land.

    Industry experts said the decision to climb and continue the flight was unusual given the oxygen masks had already been deployed and there was a risk another decompression event could occur after the one-time supply of 12-20 minutes from the oxygen masks was used up.

    “The crew would not have been able to accurately assess the amount of emergency oxygen available,” said a former Qantas head of safety, Ron Bartsch. “In my opinion the pilot in command should have landed at the nearest suitable airport.”

  464. Niels says:

    @TBill,sk999,Gysbreght
    Recently I tried different bias frequencies for the C-channel BFO in my path generation tool. It looks like 150 Hz gives the best result when attempting to minimize path curvature after 19:41.

    (The tool has just been updated to WGS84 coordinates. I’m still checking; will try to document the mathematical procedure in future. It is rather “pragmatic” as compared to the analytical approach for spherical earth model so don’t expect too much)

  465. Andrew says:

    @Victor
    @TBill

    The two pilots in the Air China incident clearly deserve to have the book thrown at them for the events that precipitated the depressurisation. In my opinion, the decision to continue the flight at a lower level was wrong, but not without some rationale.

    According to the regulations in most countries, the drop down masks in the cabin are not required if the aircraft remains at or below FL250, provided the aircraft can safely descend to FL140 or below within 4 minutes of depressurisation at all points along the planned route. However, there must be sufficient supplemental oxygen available to provide 10% of the passengers with oxygen for 30 minutes. The crew in the Air China incident should have been able to comply with the first requirement, but I doubt they would have had sufficient portable oxygen bottles to comply with the second.

    RE: “Just nuts that is the current state of affairs…”

    The times they are a-changin’. You might be surprised at just how much data new-generation aircraft such as the A350 and B787 send to the ground. I attended a forum recently where we were told that the aircraft send about 5 TB of data during a typical flight.

  466. Victor Iannello says:

    @Andrew: What is the logic behind the requirement for “sufficient supplemental oxygen available to provide 10% of the passengers with oxygen for 30 minutes”?

  467. Andrew says:

    @Victor

    Most passengers won’t be adversely affected if a depressurisation occurs at FL250, provided the aircraft immediately descends to a lower altitude. However, some (eg the elderly, those with chronic lung problems) might need assistance during and/or after the descent.

  468. airlandseaman says:

    Andrew: There is not enough bandwidth in all the communications paths combined to send 5 TB to the ground. Maybe 5 MB, but definatly not 5GB or 5TB.

  469. Victor Iannello says:

    @Andrew: Got it. Thanks.

  470. mash says:

    flatpack says:
    http://mh370.radiantphysics.com/2018/06/08/an-mh370-flight-path-ending-further-north-on-7th-arc/#comment-16913

    “The irony is that I had perceived the sole subsequent satphone call attempt as bordering on negligence by MAS but actually it was a ‘good thing’.”

    Actually, a better call strategy would be (assuming everyone then knew as much as one does now!), based on estimated fuel exhaustion time, say 10 – 20 minutes prior to that (margin of error):

    “Repeat
    Make say 1 minute ‘polling’ calls
    Until call unsuccessful”

    Then following data will be available:
    a. more BFO data (say for backtracking and RoD calculation)
    b. time of impact
    c. energy of impact?!
    d. search width limit, etc.

    More importantly, the realisation that perhaps one of simplest explanation for maintaining the live SATCOM link is to facilitate the long-awaited SAR. But unbelievably and unexplainably not forthcoming after so many hours of flight time and two unanswered [unanswerable?] satellite calls …

  471. flatpack says:

    @mash

    Better still would have been to request Inmarsat ops to write a simple shell script to pop the MH370 AES identifier back onto the LOV queue say every five minutes.

    But that is with the benefit of hindsight plus the expertise of posters on this forum.

    I simply do not understand why MAS ops made only a single subsequent call attempt after such a long interval.

  472. Don Thompson says:

    @Andrew/airlandseaman

    Somewhile back, I looked for the source of an oft quoted ‘500GB data per flight’ and it seemed to originate with Virgin Atlantic IT director >a href=”https://www.computerworlduk.com/data/boeing-787s-create-half-terabyte-of-data-per-flight-says-virgin-atlantic-3433595/”>David Bulman in 2013.

    I can’t imagine any airline downlinking this most recently quoted 5TByte volume of data over an air-to-ground datalink, even if 5TByte is a source volume, prior to compression. Transfer over a terrestrial WLAN ‘gatelink’ connection might even be time prohibitive, more likely that a high capacity storage device, swapped out of a QAR, will be exploited if collected data are in the 100s of GBytes range.

    The B787 certainly steps up the volume of data avaiable for operational analysis: it implements a ‘continuous parameter logging’ regime to its QAR and Boeing offers a range customer selectable off-loading options. I expect the current generation Airbus models will, also.

  473. flatpack says:

    @Don Thompson

    It is an old story which, by necessity, ‘inflates’ to stay current. It may have started with a TV interview with a BA chap where he waved around a tape cartridge (maybe a DAT) and extolled the benefits of recording just about everything for later analysis.

  474. TBill says:

    @Niels
    Thank you…so what BFO value(s) are you using for 23:14 sat call? Looks like Ian Holland used 218 or 219 from his graph.

  475. DennisW says:

    The ascii storage of the King James Bible (Old and New Testaments) is 4.13 MB.

  476. Andrew says:

    @ALSM
    @Don Thompson

    My knowledge is limited, so I’m only going on what we we were told during the presentation. The figure that was quoted was 5 TB, using IP technology over mediums such as Swiftbroadband HDR, gatelink, AeroMACS, etc. It’s possible that not all the data is downlinked, as Don suggested.

    Separately, the following reference suggests the A350 generates 4 GB of system data per hour, while the B787 generates 500 GB per flight:
    https://www.inmarsat.com/wp-content/uploads/2016/01/White-Paper-SwiftBroadband-Safety-The-Future-of-Airline-Communications-1.pdf

  477. Andrew says:

    Further to my comments above, perhaps the presenter meant 0.5 TB, not 5 TB.

  478. Niels says:

    @TBill
    For 23:14 UTC I use a BFO value of 217 Hz resulting in a D = fup+fcomp of 12 Hz, based on table 9 of the Inmarsat paper.

  479. DennisW says:

    @Andrew

    Yes, I see that Inmarsat claims that the B787 generates 500GB per flight. This description is simply not even plausible. A one hour flight generates as much data as an 8 hour flight? Inmarsat marketing should have their publications checked by engineering.

  480. airlandseaman says:

    This is the current top of the line Inmarsat SwiftBroadband (HGA) Serive:

    “Class 6 terminal. It delivers the full capabilities of our top-of-the range communications service for general aviation. Up to 432kbps per channel.”

    At 432kbps, it would take 134 days to send 5TB, not including all the packet overhead, which probably adds 100%. So, figure most of a year.

    Much of the literature from Inmarsat and 3rd party papers tend to conflate the amount of data collected locally on the aircraft (for download to a hardwired device on the ground) and the amount of data sent to the ground via satcom. Only a small fraction of all the data collected is sent to the ground in real time, even with advanced satcon services, like Inmarsat SwiftBroadband. The new I4 K band satellites will have higher speed options, but still nowhere near capable of supporting anything like a 5TB download per flight.

  481. airlandseaman says:

    …and it should be emphasized, something like 50 16 bit parameters transmitted every minute or so (800 bits/minute) would be enough to pin point where a plane went down and in most cases, why. More would be nice to have, but not really necessary to solve 99% of the cases.

  482. mash says:

    re: spoofed data

    I don’t quite understand the north/south path reversal logic.

    If so, why not the up/down reversal logic; in that case all the rate of descent calculation should be rate of ascent instead …

  483. Andrew says:

    @ALSM
    @DennisW

    I regret that I quoted the number that was mentioned in the presentation. The point of my original comment to @TBill is that new generation aircraft (A350, B787) downlink a LOT of data, much of which is monitored in real-time. The airlines that operate them have a much greater ability to keep an eye on their aircraft in-flight.

  484. Victor Iannello says:

    @mash: The BFO is not representative of pure Doppler frequency shift. Tracks north have different signatures than tracks south, and climbs (descents) always increase (decrease) the BFO. There are ATSB, Inmarsat, and DSTG reports which present models for calculating the BFO. You can either dig in and try to understand the BFO models, or you can trust the many people here who do understand the BFO.

  485. DennisW says:

    @Andrew

    No need to be regretful. Your opinion/comments are highly respected here.

  486. Andrew says:

    Thanks Dennis. I only regret mentioning it because the ensuing discussion clouded the intent of my original post.

  487. David says:

    A curiosity? Compare the orange and green circles on the recovered flaperon outboard end.
    • Orange. The rear spar damage of the French photo utilised by the ATSB in its flaperon/flap contact assessment is obscured in the as-recovered photo?
    • Green. Convex vs concave outboard end of spar forward? Camera spherical aberration?

    https://www.dropbox.com/s/oc2zinmgtp9eyde/Flaperon%20rear%20spar%20damage%2C%20front%20spar%20end%2C%20concave..jpg?dl=0

    https://www.dropbox.com/s/ymhoabj6dn9t0d7/Flaperon%20on%20recovery%2C%20rear%20spar%2C%20convex%20forward%20spar..jpg?dl=0

    https://www.dropbox.com/s/tie2e2qmnm6gwqs/Flaperon%20as%20recovered%2C%20photoshopped%3B%20rear%20spar%2C%20convex%20forward%20spar.jpg?dl=0

  488. Gysbreght says:

    @David: RE ” Convex vs concave outboard end of spar forward?”

    I think it is perspectivic distortion. As manufactured, the main spar is probably flat with a straight edge. It appears that impact pressure on the front face of the spar end have pushed it rearwards so that it is now concave towards the front. Viewed from the front quarter the originally straight edge appears as convex, and viewed from the rear as concave.

  489. TBill says:

    @Niels
    “For 23:14 UTC I use a BFO value of 217 Hz resulting in a D = fup+fcomp of 12 Hz, based on table 9 of the Inmarsat paper.”

    OK that (217) is good.
    It is hard to go from 204 BFO at 22:41, which is on the high side, to 217 at 23:14, which is on the low side. I theorize the “winning” path meets both BFO’s and probably is not a straight, level path.

    Either that or the data scatter kills the argument, but I am focus on the “what if the BFO data are good?” scenario.

  490. Don Thompson says:

    @Andrew,

    I agree with DennisW, there’s no cause to regret your comment.

    There is no doubt that the latest generation of aircraft, including the B787 and A350, generate large volumes of data during operations.

    The airframers, operators, and aeronautical comms providers are suffering a disconnect between modern avionics generating data and the ability to get it to the ground, in real-time. The ‘safety’ considerations, and cost, remain significant issues.

    The ICAO definition of ‘Safety’ communications has, until recently, been satisified by only VHF data, HF data, Inmarsat Classic Aero, and Iridium SBD. Now Inmarsat SwiftBroadband-Safety has attained ‘Safety’ accreditation and became an addition to that list. As ALSM commented, even SwiftBroadband-Safety is constrained by a throughput of 432kbps per channel (max 4 channels). Ka and Ku band satcom operators are not yet accepted as performant for ‘Safety’ services.

  491. airlandseaman says:

    Andrew:

    I agree with Dennis and Don. No worries. I got your original point, but felt it important to correct the record on satcom data rate capability. Many people hear and read the marketing hype (like 5TB) and come away with the same impression. The marketing people tend to conflate the amount of data produced by the latest transport aircraft (which is huge) with the tiny fraction of that data that can be transmitted to the ground in real time with currently available AMSS(R) or AMSS services. The Ka band services on the horizon will expand the capability, but still not provide the capability to download all the data produced in real-time. Again, we should not see this as an operational limitation. Only a tiny fraction of the super fine-scale detail is needed in real-time. Current ACARS via L Band is currently capable of filling the bill, if carriers were willing to pay the high cost per bit.

  492. Niels says:

    @TBill
    If you look for example at fig. 15 of the “Inmarsat” paper (the Amsterdam flight BFO graph) you can see that a small (few Hz) scatter around the “expected” BFO values is not uncommon. I agree with you that we should not just put aside the BFO data as being “inaccurate” as some tend to do, however on a level of 2,3,4 Hz perhaps one should not “read” too much in the data.
    As we can fit a trendline consistent with level and straight path at speeds close to common cruise conditions (with just a few Hz RMS error) I currently don’t assign much probability to scenario’s involving sharp turns or strong curvature as needed to reach the lower to mid-20s latitude.

  493. Victor Iannello says:

    @Niels said: As we can fit a trendline consistent with level and straight path at speeds close to common cruise conditions (with just a few Hz RMS error) I currently don’t assign much probability to scenario’s involving sharp turns or strong curvature as needed to reach the lower to mid-20s latitude.

    I didn’t either until the previous search as far north as 25S latitude failed.

  494. Niels says:

    @VictorI
    I agree that probabilities have changed after the OI search, however really difficult to put any numbers. It is a complicated situation where imo we should go step by step. A logical next step would be an in depth analysis of scan data (including a re-assessment of Fugro’s data north of S36.5) to estimate effective coverage figures. I really hope OI is doing that and will publish the numbers (as detailed as possible)

  495. DennisW says:

    @Neils

    One of the reasons I like Victor’s path above is that it is an LNAV path with a very plausible reason for a turn at the end.

  496. David says:

    @Gysbreght. “I think it is perspectivic distortion”.
    Tom Kenyon’s Exhibit 13b. appears to show distortion from a whack under, though 13c. makes it look planar and concave. At Exhibit 15 it again is convex.
    https://drive.google.com/file/d/0B461FEILFXxacDJwVTZxVVFIZHM/view

    Yes, the distortion you mention is most likely the cause though I am surprised the ATSB has not mentioned the apparent whack, leaving unanswered what in turn caused that.

    It would be nice to see other photographs of the flaperon rear spar damage, to assist making out that of the ATSB’s.

  497. TBill says:

    @David
    @Gusbreght
    Concave?
    Flaperon looks to me like it somehow got positioned in the jet engine exhaust flow.

  498. TBill says:

    @Victor
    @Niels
    @DennisW

    So checking Victor’s path above for my criterion of hitting 204/217 for 22:41 and 23:14 and Victor is about 202.5/216.5…not too shabby. Dogleg left and slow down is how I see meeting that, but could be other ways.

  499. Victor Iannello says:

    @TBill: Yes, as you can see in the tables I provided in the post, the BFO agreement at 22:41 and 23:24 is good. However, the agreement at 00:11 is out by almost 12 Hz.

  500. flatpack says:

    @Victor Iannello

    I like your path above for the reason given by DennisW and more generally because I am in the ‘further north’ camp.

    The graphic depiction implies a continuous path, so my question is:-

    Do you still advocate even a short period of racetrack or other more random loiter manoeuvres prior to the 19:41 starting point?

  501. Victor Iannello says:

    @flatpack: Yes, I proposed a racetrack pattern near Car Nicobar. The timing of the descent, deceleration to holding speed, entry into racetrack, climb, and exit from racetrack are all included in the fuel analysis file I provided above.

  502. DennisW says:

    @Victor

    I am not troubled by the ~12Hz BFO discrepancy. A small rate of descent at 00:11 is needed to match the BFO in most scenarios. You maintained a 0 ROC at that point.

  503. Victor Iannello says:

    @DennisW: A climb of about 665 fpm would be needed to eliminate the BFO error at 00:11.

  504. flatpack says:

    @Victor Iannello

    Thank you.

  505. DennisW says:

    @Victor

    The satellite is approaching the aircraft rapidly at 00:11. I disagree.

  506. TBill says:

    @Victor
    …maybe there was climb at 0011…is what I am thinking (unfortunately that could imply a preparation step for a very long “glide”

  507. Victor Iannello says:

    @DennisW: I’m not sure what you disagree with. For level flight at 00:11, with GS=354 kn, a track of 144.8°T and a position of (-21.3731,103.0915), the calculated BFO is around 240 Hz and the measured BFO was 252 Hz.

    @TBill: I doubt there was a deliberate climb at 00:11. It’s possible that there was a change in BFO either due to drift, or the plane was in a descent and the BFO had a higher variation than level flight, as @sk999 has shown occurs. Or the speed and/or track and/or position are not correct at 00:11.

  508. DennisW says:

    @Victor

    OK. So I ran the numbers instead of just voicing an opinion. My apologies. You are right.

  509. Victor Iannello says:

    @DennisW: No problem.

  510. DennisW says:

    @Victor

    Yeah, well I am embarrssed. I should have checked before commenting. Most previous tracks I worked on had track angles closer to 180 degrees and they were much farther South at 00:11.

    I’ll be OK after a couple of beers.

  511. DennisW says:

    BTW, Victor.

    I knew we had a sign difference in BFO error, but I did not pay attention to it since I relegated to the realm of incidental.

    Your error is referenced to the observables i.e. 240Hz predicted is -12Hz relative to the observable 252Hz.

    My convention has been 252Hz is +12Hz positive relative to a predicted value of 240Hz.

    My convention is the accepted way of doing things, but I won’t ask you to change.

    https://en.wikipedia.org/wiki/Forecast_error

  512. TBill says:

    @Victor
    “@TBill: I doubt there was a deliberate climb at 00:11”

    Well I hope you are correct. I am not advocating a wide search unless we have some inkling where to look wide. I would advocate continuing to to 20S +-25nm, but I am worried about a really long fight extension.

  513. DennisW says:

    @Victor

    I hope you realize I was joking with you.

  514. David says:

    @TBill. “Flaperon looks to me like it somehow got positioned in the jet engine exhaust flow.”

    From the below it seems to me that the flaperon and flaps to some extent would be designed for buffeting by the by-pass flow. I do not know what cooling of the exhaust flow there would be through mixing of the two before they reach the wing rear but either way would not expect the skins to be exposed to the exhaust temperatures directly. I expect a late go round would be the design point for aerodynamic loading.
    https://www.dropbox.com/s/2zxxvjbhzp219k3/dihedral%20and%20flap%20hinge%20angles.docx?dl=0

    Hard to visualise how the flaperon would enter the exhaust flow unless separated and at the end of flight both exhaust and by-pass flows should be lower than such a design point even during a controlled ditching.

  515. TBill says:

    @Andrew
    Thank you for pointing out the progress on aircraft data monitoring.

    @Victor
    I do not want to dwell on this, but please let me state more clearly that the Air China depressurization incident to me is very much like the MH370 tragedy. We had a “rogue” pilot again, but the outcome was thankfully not as terrible as MH370. The Air China pilots just wanted to privately grab a smoke.

    >The current status quo allows rogue pilot to (aside from secretly turning off all all communications and intentionally crashing aircraft) to intentionally depressure aircraft to neutralize the PAX. All without any external warning signal being sent by say a more sophisticated ELT under consideration or other design/procedural deterents.

    The “rogue” activity is enhanced by 9/11 security measures which gives pilots impervious cockpit doors for complete privacy.

  516. Victor Iannello says:

    @DennisW: Actually, I did at one point consider which sign convention to use for the BFO error. Ultimately, since we are testing the “goodness” of various paths, I used the measured values as the “reference”, even though those values are noisy. I won’t try to defend that decision. As you know, in the end, it really doesn’t matter, as long as the definition is known and is used in a consistent manner.

  517. Victor Iannello says:

    @TBill: I understand. Under the current system, it is difficult to circumvent the actions of a determined rogue pilot. Luckily, that situation doesn’t occur very often.

  518. mash says:

    re: Possibility of a cabin-initiated takeover (from time constraint perspective)

    According to “CAPTIO trajectory”, to ‘switch off’ the transponder in cabin, left and right bus power must be disabled (at the same time).

    After removing power at 1720:36, the “people in command” can then take control and re-navigate the plane before 1721:31 (see quote from “MAS FI” below)

    1. “At 1719:30 UTC [0119:30 MYT], MH370 acknowledged with “ Good night Malaysia Three Seven Zero ”. This was the last recorded radio transmission from MH370.”

    2. “The Mode S symbol of MH370 dropped off from radar display at 1720:36 UTC [0120:36 MYT], …”

    3. “At 1721:13 UTC [0121:13 MYT] the Military radar showed the radar return of MH370 turning right but almost immediately making a constant left turn to a South Westerly direction.”

    The question is, operationally, is it possible to accomplish all these so smoothly in such a short time (say < 45 secs) – without any resistance?

    a. switch off power
    b. move to cockpit door
    c. unlock door
    d. overcome pilot
    e. re-program flight path
    f. then plane started turning, and radar recorded movement

    [g. reverse power-off process (presumably in cabin) after say one hour]

    Therefore, given that the elapsed time between "good bye" and transponder "off" is only 1 minute, and transponer "off" and plane "turning" is only say 45 second, I still (after simple-minded BFO "reversal" logic above!) maintain that any cabin-initiated takeover (esp starting after "good night") is very unlikely …

    [Further thought]
    Assuming eavesdropping cockpit radio conversation, why not turn off power immediately, since during that one minute pilot could have started voice communication with HCM.

  519. Victor Iannello says:

    @mash said: According to “CAPTIO trajectory”, to ‘switch off’ the transponder in cabin, left and right bus power must be disabled (at the same time).

    No. The transponder switch is simply set to standby.

    But I do agree that the timeline is very compressed for the scenario in which hijackers need to take control of the cockpit and turn back.

  520. ArthurC says:

    Apparently, and I don’t want to throw a wrench, Boeing has a remote control system that can’t be disabled: https://en.wikipedia.org/wiki/Boeing_Honeywell_Uninterruptible_Autopilot

    Would that not have been a factor?

    (Sorry if this was brought up before)

  521. Don Thompson says:

    @ArthurC

    BHUAP is nonsense.

  522. Victor Iannello says:

    @ArthurC: To further to Don’s point, the main proponent of this idea seems to be Field McConnell. Go peruse the site Abel Danger and see the collection of theories that he is espousing.

  523. lkr says:

    VI: re Abel Danger. Thanks! — now I need eye bleach!

  524. Victor Iannello says:

    @lkr: It’s not comforting to know that at one point he was a 747 captain for Northwest.

  525. ArthurC says:

    Makes sense, sounded too much like another conspiracy theory.

  526. Victor Iannello says:

    @ArthurC: If by “conspiracy theory”, you mean a complicated theory that is not consistent with the evidence, I agree that BUAP falls into that category. However, I tend to not use this term because crimes involving conspiracies commonly occur.

  527. Andrew says:

    @Victor

    RE: Abel Danger

    The Australian colloquialism “a few ‘roos loose in the top paddock” seems quite appropriate!

  528. mash says:

    re: ACARS messages sent & SATCOM log discrepancy

    A. Messages sent (more than 2 according to “FI MAS” report)

    “The first message sent to the aircraft cockpit printer from the MAS ODC was at 1803:23 UTC. The ACARS message requested the crew to contact the HCM ATCC immediately. The incoming downlink message at 1803:24 UTC showed the message failed to reach the aircraft. Messages are auto transmitted every 2 minutes and the message was retransmitted until 1843:33 UTC but all messages failed to get a response.” (page 47 “FI MAS”)

    B. But only 2 in SATCOM log

    “7. 1803:41 – GES initiates a DATA-2 ACARS transmission (uplink), but receives no acknowledgement from the SATCOM…
    8. 1805:11 – GES initiates a DATA-2 ACARS transmission, but receives no acknowledgement from the SATCOM, indicating that there is still no SATCOM link at this time.
    9. 1825:27 – SATCOM Log-On, initiated from the aircraft terminal…”
    (page 54 FI MAS)

    So why such discrepancy in report, does that make one raise the SATCOM power on-off timing question again?

  529. William Shea says:

    IGARI. South China Sea that is being monitored by several entities.

    These multiple airborne assets, from several entities, fly without notifying civilian Air Traffic Control.

  530. Don Thompson says:

    @mash

    You raise a question about the 18:05 ACARS message delivery attempt.

    First, it’s necessary to understand that the messaging system operates on the ‘store and forward principle’, and multiple delivery paths (VHF, SATCOM) may be possible but only one is active at any given time.

    MAS OCC submitted the message for delivery at 18:03Z, the message was forwarded through SITA’s ACARS server to the GES for relay to the aircraft over the SATCOM datalink. The datalink protocol (not ACARS) made two unsuccessful attempts to relay the message to the AES. Upon that failure, the GES signals failure back to the ACARS hub and, in turn, the MES is failed back to the originating system in MAS OCC.

    The MAS OCC originatoring terminal continues to resubmit the message, but the ACARS server immediately rejects/fails the message back to the OCC until it, ultimately, times/counts out on retries.

    If you examine the message submit and reject times in the SATCOM Log and the ACARS Traffic Log you’ll see that the message is first failed (MAS-F/message assurance – failed) 3 minutes after it is submitted (18:06Z). The SATCOM attempts occur in those 3 minutes. Subsequent message retries fail within seconds.

    An outcome of this is that the ACARS server, after 18:06Z, has no known path to the aircraft: SATCOM delivery has failed, and VHF was earlier disabled prior to departure from WMKK (an ACARS Media Advisory message indicates ‘LV’ – Lost VHF). While 9M-MRO makes two later SATCOM Log Ons, its data communications management function does not initiate ACARS correspondence over the available datalink.

  531. mash says:

    @Don Thompson

    Thanks.

    So to receive the ACARS message from MAS OCC, 9M- MRO has to do two things:
    a. SATCOM logon
    b. Init ACARS path/datalink (to SATCOM)

    And no re-init after logon means no message – if I understand correctly …

  532. Don Thompson says:

    @mash

    To be clear

    a. SATCOM Log On
    b. initialise correspondence with the ground ACARS server. That initiating message from the aircraft is an ACARS ‘Media Advisory’ message. It’s not enough for the network path to be available, the aircraft must initiate correspondence over that path using the ‘Media Advisory’ message.

    Just to preempt another possible question, the ISO-8208 satellite sub network (SSN) used by the IFE applications is entirely separate from ACARS. The ISO-8208 SSN uses additional logical channels over the SATCOM datalink to support switched virtual circuits to remote systems on the terrestrial ATN.

  533. William Shea says:

    The aircraft may have been “crippled .”

    BFO May be a distraction.

    Too much time expanded on minutae.

  534. William Shea says:

    What happened at IGARI…

  535. William Shea says:

    No claim of responsibility. No suicide.

    Too much effort for minutae.

    Time to re-evaluate.

  536. Don Thompson says:

    @William Shea

    What you call “minutae” is all that we have of any substance.

    While that “minutae” may yet be beyond the comprehension of many, it does indicate that the aircraft was not “crippled“: the time of the final SATCOM exchange indicates that the aircraft flew to its expected endurance; that the final SATCOM exchange occurred indicates that all the significant avionics systems required for sustained flight and navigation were operating as expected.

  537. Victor Iannello says:

    @Marijan said: Final MH370 report to be made public on July 30

    Word on the street is that there will be nothing of any significance that is new. That’s not surprising.

  538. Victor Iannello says:

    [Comments here are closed. Please continue the discussion under the new article.]